Download PGI PG PGI 2018 May Solved Question Paper

Download PGIMER (Post Graduate Institute of Medical Education & Research, Chandigarh) PGI 2018 May Solved Question Paper


1. Which of the following statement(s) is/are
FALSE aboutgenitor-urinary anatomy?
a) Superficial dorsal penile artery is a branch of
bulbourethral artery
b) Penile skin is supplied by superficial external pudendal artery
c) Penis receives three venous drainage systems
d) There are 5 urethral sphincters in children
e) Nerve supply of penis includes pelvic splanchnic nerves
Correct Answer - A:D
Ans. is 'a' i.e., Superficial dorsal penile artery is a branch of
bulbo-urethral artery & 'd' i.e., There are 5 urethral sphincters
in children

Vasculature & Innervation of penis
A. Arterial supply: mainly by branches of the internal pudendal
artery (branch of anterior division of the internal iliac artery).

Dorsal Arteries of penis supplying the fibrous tissue around corpora
and skin of penis.
Deep arteries of penis (cavernous artery or artery to crura of penis):
they pierce the crura and run within the corpora cavernosa, thus
supplying the erectile tissue.
Artery of bulb of penis (Bulbo-urethral artery): supply posterior part
of corpus spongiosum + Cowper's glands
Superficial eir Deep branches of external pudendal artery (branch of
femoral artery): supply penile skin
B. Venus drainage: The penis is drained by three venous
systems: superficial, intermediate, and deep.


SUPERFICIAL veins: drains into the left saphenous vein. Veins from
more superficial tissue may drain into the external superficial
pudendal veins.
INTERMEDIATE system contains the deep dorsal and circumflex
veins, lying within and beneath Buck's fascia.
DEEP drainage system consists :
. Crural veins
. Cavernosal veins
. The internal pudendal veins
C. Lymphatic drainage:
Glans drain into the deep inguinal nodes.
From rest of the penis lymph drains into the superficial inguinal
nodes.
D. Nerve supply
a) Somatic supply:
Skin of the penis is supplied by pudendal nerve via dorsal nerve of
penis and posterior scortal nerve.
A small area on the dorsum of proximal penis (root) :ilionguinal
nerve.
The muslces,bulbocavernosusand ischeocavernosus:perineal
branch of pudendal nerve.
b) Parasympathetic :It is responsible for erection and is derived
from pelvic splanchnic nerves (S2S3S4).
c) Sympathetic :It is responsible for ejaculation(initial part) and is
derived from Ll segment via superior and inferior hypogastric plexus.


2. 4th Aortic arch is responsible for the
formation of?

a) Arch of aorta
b) Pulmonary artery
c) Pulmonary vein
d) Subclavian artery
e) Subclavian vein
Correct Answer - A:D
Ans. is 'a' i.e., Arch of aorta & 'd' i.e., Subclavian artery
Aortic arch IV : The right and left side develop differently :
. Left aortic arch IV forms part of arch of aorta which lies between
left common carotid and left subclavian arteries.
. Right aortic arch IV forms most proximal part of right subclavian
artery (distal part is formed by right dorsal aorta and right 7th
cervical intersegmental artery). The left subclavian artery is formed
by left 7th cervical intersegmental artery.
. Pulmonary artery -5th aortic arch
. Pulmonary vein - pulmonary veins develop independently (during
the formation of septum primum.
. Subclavian vein - Subclavian veins are formed by 7th
cervical intersegmental vein


3. All are true regarding renal anatomy

EXCEPT?

a) Left kidney is related to both 11th & 12th ribs
b) Long axis is lateral and upwards
c) Supplied by anterior segmental artery
d) Supplied by rectal plexus
e) Both kidneys move in opposite direction during respiration
Correct Answer - B:D:E
Ans. is 'b' i.e., Long axis is lateral and upwards, 'd' i.e.,
Supplied by rectal plexus & 'e' i.e., Both kidneys move in
opposite direction during respiration

LOCATION
Arterial Supply:
Each kidney is supplied by renal arteries, left and right, which
branch from left & right phrenic artery which branch directly
from


the abdominal aorta.
Posterior, apical, upper anterior, middle anterior and lower are
5 segments of vascular supply in each kidney
Kidneys receive approximately 20% of the cardiac output.
Renal arterySegmentalarteriesInterlobar arteries (penetrate
the renal capsule and extend through the renal columns between
the renal pyramids)
Interlobar arteries supplyArcuate arteries (run through the
boundary of the cortex and the medulla) Interlobular arteries
Afferent arterioles (supply the glomeruli).
Branches of renal artery are end arteries.
Right renal artery is longer and passes behind IVC.
NERVE SUPPLY:
Kidney is supplied by renal plexus, an offshoot of coelic plexus.


4. True statement regarding anatomy of
nasolacrimal apparatus is/are?

a) Nasolacrimal duct opens in inferior meatus
b) There is upper & lower canaliculus
c) Most people have common canaliculus
d) Canaliculus is lined by Ciliary stratified columnar epithelium
e) Nasolacrimal duct is fractured in most head and neck injuries
Correct Answer - A:B:C:E
Ans. is 'a' i.e., Nasolacrimal duct opens in inferior meatus,
'b' i.e., There is upper & lower canaliculus, 'c' i.e., Most
people have common canaliculus & `e' i.e., Nasolacrimal
duct is fractured in most head and neck injuries
Nasolacrimal (drainage) apparatus consists of:

. Lacrimal canaliculi
. Lacrimal sac
. Nasolacrimal duct
Lacrimal canaliculi
There are two lacrimal canaliculi - superior and inferior on each side.
They unite to form a common canaliculusand drain via the sinus of
Maier into the lacrimal sac posterior to the medial palpebral
ligament and anterior to the orbicularis oculi muscle.
It is lined by stratified squamous epithelium supported by elastic
tissue.
Lacrimal sac:
The lacrimal sac lies in the lacrimal fossa on the inferomedial aspect
of the bony orbit between the posterior and anterior lacrimal crests.


The sac receives the lacrimal canaliculi before it drains via the valve
of Krause into the nasolacrimal duct.
Nasolacrimal duct
The nasolacrimal duct is the inferior continuation of the lacrimal sac
2 parts:
. intraosseous part (12 mm): lies within the nasolacrimal canal of
the maxilla
. membranous part (3-5 mm): runs in the nasal mucosa;
terminates below the inferior nasal meatus as a slit-like opening
where it is covered by a mucosal fold called the valve of Hasner


5. Posterior interosseous nerve supplies?

a) Extensor carpi radialis longus
b) Extensor carpi radialis brevis
c) Extensor carpi ulnaris
d) Brachioradialis
e) Extensor pollicis longus
Correct Answer - B:C:E
Ans. is 'b' i.e., Extensor carpi radialis brevis, 'c' i.e.,
Extensor carpi ulnaris & `e' i.e., Extensor pollicis longus

The posterior interosseous nerve is a pure motor nerve
and innervates supinator & extensor carpi radialis.
It supplies:?
. Extensor carpi ulnaris
. Extensor digitorum
. Extensor digitiminimi
. Abductor pollicis longus
. Extensor pollicis longus and brevis
. Extensor indicis


6. Cavernous sinus receives blood from?

a) Superior ophthalmic vein
b) Superior petrosal sinus
c) Inferior petrosal sinus
d) Spheno-parietal sinus
e) Basilar plexus ofveins
Correct Answer - A:D
Ans. is 'a' i.e., Superior ophthalmic vein & 'd' i.e.,
Spheno-parietal sinus

Tributaries (incoming channels) of cavernous sinus
. Superior ophthalmic vein
. A branch of inferior ophthalmic vein or sometimes vein itself.
. Central vein of retina (it may also drain into superior
ophthalmic vein).
. Superficial middle cerebral vein.
. Inferior cerebral vein.
. Sphenoparietal sinus
. Frontal trunk of middle meningeal vein (it may also drain
into pterygoid plexus or into sphenoparietal sinus).


7. Inner Waldayer's ring includes?

a) Jugulo-diagastricnodes
b) Jugulo-omohoid nodes
c) Palatine tonsil
d) Tubal tonsil
e) Pharyngeal tonsil
Correct Answer - C:D:E
Ans. is 'c' i.e., Palatine tonsil, 'd' i.e., Tubal tonsil & `e'
i.e., Pharyngeal tonsil



8. Structure(s) passing through aortic
opening into the chest include?

a) Thoracic duct
b) Azygousvein
c) Hemiazygous vein
d) Esophagus
e) Inferior vena cava
Correct Answer - A:B
Ans. is 'a' i.e.,Thoracic duct & 'b' i.e. Azygous vein
The aortic hiatus is a hole in the diaphragm. It is the lowest and
most posterior of the large apertures.
It is located approximately at the level of the twelfth thoracic
vertebra (T12).
Through it passes the aorta, the azygos vein, the thoracic duct, and
hemi-azygos vein passes through the left crus.


9. Constituents of Rotator cuffincludes all

EXCEPT?

a) Teres major
b) Teres minor
c) Supraspinatus
d) Infraspinatus
e) Subscapularis
Correct Answer - A
Ans.is 'a' i.e., Teres major
Each one of these muscles is part of the rotator cuff and
plays an important role:

Supraspinatus. This holds your humerus in place and keeps your
upper arm stable. And helps lift your arm.
Infraspinatus. This is the main muscle that lets you rotate and
extend your shoulder.
Teres Minor. This is the smallest rotator cuff muscle. Its main job is to
assist with rotation of the arm away from the body. Subscapularis.
This holds your upper arm bone to your shoulder blade and helps you
rotate your arm, hold it straight out and lower it.





10. Not TRUE statement regarding parotid
gland is/are?
a) Deep lobe contains deep lymphatics
b) Divided into superficial and deep lobes by facial nerve
c) Parotid duct opens opposite to the second upper molar
d) Ectodermal in origin
e) Auriculotemporal nerve is the main sensory nerve
Correct Answer - A
Ans. is 'a' i.e., Deep lobe contains deep lymphatics
Development:
Parotid gland is the first salivary gland to appear, in early 6' week.
It is ectodermal in origin and develops from the buccal epithelium
just lateral to the angle of mouth
Structures emerging from parotid
The following structures emerge from the parotid gland:
Anterior border:
Parotid duct
3 Terminal branches of facial nerve:
The zygomatic and buccal branches: toward the temporal region,
eyelids and cheek, respectively.
Mandibular branch : Run along the body of the mandibletowards the
mouth
Apex:
5th terminal branch of facial nerve: Cervical branch continues into
the neck (to platysma).
Anterior & posterior divisions of retromandibular vein

Posterior border:
Posterior auricular nerve
Posterior auricular artery
Posterior auricular vein
Along base:
superficial temporal artery
temporal branch of facial nerve
Auriculotemporal nerve
STRUCTURES WITHIN GLAND:
Arteries:
External carotid artery enters through posteromedial surface
Maxillary artery
Superficial temporal vessel
Posterior auricular artery
Veins:
The retromandibular veins
Facial Nerve
Parotid Duct (Stenson's duct)
The duct turns opens into the vestibule of the mouth (gingivo- buccal
vestibule) opposite the crown of the upper 2nd molar tooth
Nerve supply:
PARASYMPATHETIC:auriculo temporal nerve
SYMPTHETIC SUPPLY- plexus around the external carotid artery.
SENSORY NERVES: auriculotemporal nerve, except for parotid
fascia & overlying skin which are innervated by Great auricular
nerve (C2, C3).


11. External anal sphincter is innervated by?
a) S2,S3,S4
b) S2, S3
c) S1,S2
d) L5,Sl
e) L2,L3
Correct Answer - A
Ans. is'a' i.e.,S2, S3, S4
Anal Sphincters :
Two sphincters, internal and external, surround the anal canal:
A. Internal sphincter (sphincter ani internus):
Involuntary
Sympathetic fibers through superior hypogastric plexus a
Parasympathetic fibers from pelvic splanchnic nerves (S2 S3 S4).
B. External sphincter (sphincter ani externus)
Voluntary
Surrounds the entire length of anal canal
Inferior rectal nerve (S2 S3 S4) and perineal branch


12. A patient had a lesion in the wrist. On
examination the thumb was laterally
rotated & adducted, with ape thumb
deformity. Which is the nerve involved?

a) Median
b) Ulnar
c) Radial
d) Post interosseous nerve
e) None
Correct Answer - A
Ans. is 'a' i.e., Median
Ape thumb (Simian thumb) deformity : -
The Ape Hand Deformity is caused by damage to the distal median
nerve(also called a Median Claw lesion), and subsequent loss
of opponenspollicis muscle function.
The thumb is adducted and laterally rotated so that the thumb lies in
the same plane as the other fingers. It is due to over action of
adductor pollicis (supplied by ulnar nerve).


13. Spring ligament refers to?
a) Plantar calcaneonavicular ligament
b) Short planter ligament
c) Long planter ligament
d) Both'b' &'c'
e) None
Correct Answer - A
Ans. is 'a' i.e., Plantar calcaneonavicular ligament
The spring ligament (Plantar calcaneonavicular ligament) is a
group of ligaments which connect calcaneum to navicular.
It consists of :?
. Superomedial ligament.
. Medioplantar oblique ligament (medial or
intermedialcalcaneonavicular ligament).
. Inferoplantar longitudinal ligament (Lateral
calcaneonavicular ligament).


14. Which of the following is NOT a content of
medial wall of middle ear?
a) Oval window
b) Round window
c) Processuscochlearformis
d) Aditus & antrum
e) Notch of Rivinus
Correct Answer - D:E
Ans. is 'd' i.e., Aditus & antrum & 'e' i.e., Notch of
Rivinus Medial or inner labyrinthic wall of middle ear
(Parieslabyrinthica):

It is formed by the lateral wall of labyrinth.
It presents following structures:
. Promontory: It is a bony bulge which is due to the basal coil
of cochlea. Tympanic plexus present over it.
. Oval window (fenestra vestibuli/ovalis):
. The footplate of stapes
. Round window(fenestra cochleae/rotunda): covered by
the secondary tympanic membrane.
. Horizontal tympanic part of fallopian canal for facial nerve
. The tympanic segment of facial nerve canal
. Lateral semicircular canal
. Processuscochleariformis
The tendon of tensor tympani takes a turn on this process and then
is inserted on the neck of malleus.
Processuscochleariformis is an important surgical landmark for the


level of the genu of the facial nerve.


15. Pronator quadrates has same
innervations as following muscles?
a) Flexor pollicis longus
b) Flexor digitorum superficialis
c) Palmaris longus
d) Flexor digitorumprofundus of middle finger
e) Flexor carpi ulnaris
Correct Answer - A:B:C:D
Ans. is'a' i.e., Flexor pollicis longus, 'b' i.e., Flexor
digitorum superficialis, 'c' i.e., Palmaris longus, 'd' i.e.,
Flexor digitorumprofundus of middle finger

All the flexor muscles of the forearm are supplied by median
nerve, except the flexoraulnaris and the medial half of flexor
digitorumprofundus to the ulnar two fingers (4th& 5th finger).
MEDIAN NERVE INNERVATONS:
The median nerve innervates the majority of the muscles in
the anterior forearm, and some intrinsic hand muscles.
Anterior Forearm
Innervates muscles in the superficial and intermediate layers:
Superficial layer: Pronator teres, flexor carpi radialis and palmaris
longus.
Intermediate layer: Flexor digitorum superficialis.
The median nerve also gives rise to the anterior
interosseous nerve, which supplies the deep flexors:

Deep layer: Flexor pollicis longus, pronator quadratus, and the
lateral half of the flexor digitorumprofundus (the medial half of
the


muscle is innervated by the ulnar nerve).
Hand
The median nerve innervates some of the muscles in the
hand via two branches.
The recurrent branch :Thenar muscles
The palmar digital branch : Innervates the lateral two lumbricals.


16. Supination & pronation doesn't take place
at?
a) Superior radioulnar joint
b) Middle radioulnar joint
c) Inferior radioulnar joint
d) Radiocarpal joint
e) Midcarpal joint
Correct Answer - D:E
Ans. is 'd' i.e., Radiocarpal joint &`e' i.e., Midcarpal joint
Forearm rotation (supination pronation) occurs at radio-ulnar
joint complex i.e. Superior (proximal) radioulnar joint, Inferior
(distal) radioulnar joint & Middle radioulnar joint .
Movement: Muscles responsible for
movements Supination

Supinator (when elbow is extended)
Biceps brachii (when elbow is flexed)
Brachioradialis (supinates the pronated forearm to midprone
position)
Pronation
Pronaterteres (rapid pronator)
Pronator quadrates (strong pronator)
Brachioradialis (pronates the supinated forearm to midprone
position)


17.


Occlusion occurs at 2nd part of axillary
artery, blood flow is maintained by which of
the following anastomosis?

a) Suprascapular artery & post circumflex humeral artery
b) Anterior and posterior circumflex humeral artery
c) Circumflex scapular and posterior circumflex humeral artery
d) Deep branch of the transverse cervical artery and
subscapular artery
e) Anterior circumflex artery and subscapular artery
Correct Answer - A:D
Ans. is 'a' i.e., Suprascapular artery & post circumflex
humeral artery & 'd' i.e., Deep branch of the transverse
cervical artery and subscapular artery

Anastomosis around scapula provides blood supply to distal part if
first or second part of axillary artery is blocked.
Anastomosis around scapula connects the first part of subclavian
artery with third part of axillary artery.
Anastomosis Around Scapula
Connects 1st part of subclavian with 3rd part of axillary artery.
Around body of scapula
. Suprascapular (branch of 1st part of subclavian).
. Deep branch of transverse cervical (branch of thyrocervical trunk
from Ist part of subclavian).
. Circumflex scapular (branch of subscapular, branch of 3rd part
of axillary).
On the acromion process

. Acromial branch of thoracoacromial (branch of 2nd part of axillary).
. Ascending branch of posterior circumflex humeral (branch of
3rd part of axillary).
. Acromial branch of suprascapular (branch of 1st part of subclavian).


18. True regarding red pulp of spleen is/are?
a) Periarteriolar lymphoid sheaths
b) B-cell containing lymphoid follicles
c) Perisinusoidal macrophages
d) Composed of sinusoids and splenic cords
e) Removal of old RBCs and modification of new cells
Correct Answer - C:D:E
Ans. is 'c' i.e., Perisinusoidal macrophages, 'd' i.e., Composed
of sinusoids and splenic cords & `e' i.e., Removal of old RBCs
and modification of new cells
STRUCTURE OF SPLENIC PARENCHYMA
The spleen comprises many units of red pulp & white pulp, which
are centred around central arterioles (smaller branches of
splenic artery).
Red pulp
Red pulp contains large number of venous sinusoids draining into
tributaries of splenic vein.
Venous sinusoids are lined by endothelial stave cells which form
incomplete layer & present intracellular slits b/w them through
which blood can percolate.
Surrounding the sinuses is the parenchyma, which contains
lymphocytes, macrophages, plasma cells, etc.
It metabolizes senescent red blood cells (erythrocytes).
Adjacent blood spaces contain blood cells and arranged in cords
called splenic cords of billroth(splenic cords of reticulin fibers).


19. True regarding HbA2 is/are?
a) It has more capacity to carry oxygen
b) Concentration is more than HbA
c) Level is increased in Thalasemia
d) Consists of 2 alpha and 2 beta chains
e) None of the above
Correct Answer - B
Ans. is'c'i.e., Level is increased in Thalassemia
Ref: Ganong 23d/e p. 523-525 https: / /www. aafp.org/ alp / 2009
/08 1 5 /p3 39.html
"The hemoglobin electrophoresis with beta thalassemia trait usually
has reduced or absent HbA, elevated levels of HbA2. and
increased HbF.
However, a normal concentration of HbA2 does not rule out beta
thalassemia trait. especially if there was coexistent iron
deficiency, which can lower HbA2 levels into the normal range."
"Hemoglobin A2 may be increased in beta thalassemia or in people
who are heterozygous for the beta thalassemia gene.


20. TRUE statement(s) regarding "loop of
Henley" in kidney is/are?
a) Ascending limb actively absorbs Na
b) Ascending limb actively absorbs Cl
c) Ascending limb secretes water in lumen
d) Descending limb secretes water
e) Descending limb receives hyperosmolar fluid from PCT
Correct Answer - A:B
Ans. is 'a' i.e., Ascending limb actively absorbs Na+ & 'b'
i.e. Ascending limb actively absorbs Cl-

Ref: Ganong 25th/e P. 680-691; Essentials of medical physiology
p. 271
LOOP OF HENLE
The loop of Henle has a thin descending, a thin ascending and a
thick ascending segments.
REABSORPTION IN THIN ASCENDING LIMB:
NaCl- reabsorption occurs -
Due to high NaCl- permeability.
Less permeable to water.
Tubular fluid is iso-osmotic.
REABSORPTION IN THICK ASCENDING LIMB:
Sodium, Potassium & Chloride reabsorption:
By "Secondary active transport" -
Through Na2+-K+-2Cl-- carrier transporter.
Transports one Na2+, one K+, & two Cl-.
Active sodium absorption occurs.

30% filtered Na2+ reabsorbed.
Water reabsorption:
Totally impermeable to water.
Ascending segment also referred "Diluting" segment:
Due to sodium & solute absorption without water.
Resulting in tubular fluid dilution.
Tubular fluid is hypotonic.


21. Antioxidant effects are shown by?
a) Vitamin C
b) Vitamin E
c) Selenium
d) Zinc
e) Vitamin B
Correct Answer - A:B:C:D
Ans. is'a' i.e., Vitamin C, 'b' i.e. Vitamin E, 'c' i.e., Selenium
&'d', Zinc

Ref Harper's 30th/e p. 565; Robbins's th/e p. 66-67, http:/
/ www.rroij. coml
Naturally occurring antioxidants:
Alkaloids and related compounds
Amino acids and peptide derivatives:- cysteine, tryptophan,
melatonin, and tryptamine
Vitamins: -Beta carotene, Vitamin A, Vitamin C, Vitamin E
(tocopherol)
Minerals:- selenium, zinc
Enzymes :- catalase, superoxide dismutase (SOD), and glutathione
peroxidase
Flavonoids & Isoflavonoids: Chalcones and catechins
Carnosine
Clorogenic & melanic acids
Curcumin and derivatives
Ergothioneine
Free phenolic acids
Hydroquinones and quinones

Lignans
Lipoic acid
Lycopene
Tetrapyrroles
Uric acid and other ourines


22. Right combination of sensory receptor
and sensation carried by them includes?
a) Krouse's bulb - pressure
b) Pacinian corpuscels - vibration
c) Meissner's corpuscles - pressure
d) Ruffini's end organs - pressure
e) Merkel's disc - cold temperature
Correct Answer - B:D
Ans. is 'b' i.e., Pacinian corpuscles - vibration &'d' i.e.
Ruffini's end organs pressure

[Ref: Guyton 12th/e p. 560 table G6.1); Principles of
medical physiology p. 647, 648
Tactile (Touch) receptor:
Whether a tactile receptor senses pressure or vibration depends on
whether receptor is fastly adapting or slowly adapting.
Touch, pressure, & vibration are different forms of same sensation.
Pressure is felt when force applied on skin is sufficient to reach deep
receptors.
Touch is felt when force is insufficient to reach deep receptors.
Hence, detected by superficial receptors (Merkel's disc & Meissner's
corpuscle).
Vibrations are rhythmic variations in pressure.
I.e. Rhymic variations of force that reaches deep receptors.
Divisions:
2a. Slowly adapting:
Examples include, "One each from superficial & deep cutaneous


receptors"
Ruffini's end organ -
Meant to detect sustained pressure.
Useless for vibrations.
Merkel's disk -
Detect two-point discrimination.
2b. Rapidly adapting:
Examples,
"One each from superficial & deep cutaneous receptors"
Pacinian corpuscle -
Stops discharge in response to sustained pressure.
Useful to detect vibrations - I.e., when pressure fluctuates rapidly.
Meissner's corpuscle -
Detect surface texture.
Hence, Higher the rate of receptor adaptation Greater is
detectable vibration frequency.
3. Based on type of tactile sensations detected:

3a. Superficial sensations:
Generally touch
By Meissner's corpuscle (detect surface texture i.e. rough or
smooth)
By Merkel's disc (detect two-point discrimination).
3b. Deep sensations:
Pressure (Deep touch) - Detected by Rufini organ.
Vibrations - Detected by Pacinian corpuscle.
SUMMARY:
1. Superficial cutaneous receptors:
Detect touch (Superficial sensation)

Merkel's disk - Slowly adapting & detect two-point
discrimination.
Meissner's corpuscle - Rapidly adapting & detect surface
texture.
2. Deep cutaneous receptors:

Detect deep touch, pressure, & Vibration.
Ruffini's end organ - Slowly adapting & detect sustained
pressure/deep touch.
Pacinian corpuscle - Rapidly adapting & detect

vibrations (useful only when pressure fluctuates rapidly. i.e.
during vibrations).
Higher the rate of adaptation of receptor, the greater vibration
frequency it can detect.


23. According to WHO, normal semen
findings are?
a) Volume - 1.5 ml
b) Concentration - 15 million/ml
c) Progressive motility - > 40%
d) Normal morphology - > 10%
e) Ph - < 6.3
Correct Answer - A:B
Ans. is'a'i.e., Volume - 1.5 ml &'b'i.e. Concentration - 15
million/ml https://www.institutobernabeu.com/foro/en/2014/02/17/seme
quality-parameters-according-to-the-world-health-organisation-who/
The World Health Organisation (WHO) has published several
editions of the "Manual for the Examination of Human Semen
and Sperm-Cervical Mucus Interaction", the last one in 2010.
The concept of "Lower Reference Limit (LRL) was established in the
last manual of the WHO.
There are many parameters obtained through a spermiogram, the
most frequently studied are:
Volume:The normal volume of an ejaculate sample after 3 /5 days of
sexual abstinence is 1.5ml approximately. Lower volumes might
suggest hypospermia.
Color: Sperm is usually opalescent white, slightly yellow. When the
color is altered, it is recommended to study possible causes.
pH: Value should be greater than T.l. Lower values might be a sign
of azoospermia (lack of spermatozoa) or chronic inflammatory
processes.


Sperm concentration: Normal values are around 15 million per ml
eiaculated or 39 million per complete semen sample. When
these values are lower it could indicate Oligozoospermia.
Motility: The percentage of motile spermatozoa and progressively
motile is analyzed. The progressive motility value should be over
32%, on the contrary it might indicate Astenozoospermia.
Vitality: The percentage of vital spermatozoa must be over 58%.
Lower values could indicate Astenozoospermia.
Morphology: There might be 4% or more normal spermatozoa in a
usual spermiogram. Lower percentages could indicate
teratozoospermia.


24. Normal value of hydrogen ions in human
fluids is?
a) 35 meq/L
b) 40 meq/L
c) 45 meq/L
d) 50 meq/L
e) None of the above
Correct Answer - E
Ans is 'e' i.e., None of the above
The normal hydrogen ion concentration of blood and other body
fluids is quite low <0.0001 mEq/L) compared with the body fluid
concentrations of other electrolytes.
Because it is so low, hydrogen ion concentration is measured in pH
units, calculated as the negative logarithm of the concentration in
milliequivalents per liter.
Normal pH ranges from 7 .35 to 7 .45 for arterial blood and from 7
.31 to 7.41 for venous blood.


25. TRUE regarding hypoxemia is/are?
a) Decrease in ventilation
b) Decrease in delivery of oxygen to tissues
c) Inadequate utilization of oxygen by tissues despite
normal delivery of oxygen
d) Decreased oxygen pressure in blood
e) All of the above
Correct Answer - A:D
Ans is'a' i.e., Decrease in ventilation & 'd' i.e. Decreased
oxygen pressure in blood.

[Ref: Ganong 25th/e p. 647 6 24/e p. 649; Principles of medical
physiology p. 354, 355;
https://www.ncbi.nlm.nih.gov/pmc/articles/PMc52341994]
Option b & c are related with hypoxia, not hypoxemia.
Hypoxemia and hypoxia:
The term hypoxia and hypoxemia are not synonymous.
Hypoxemia is defined as a decrease in the partial pressure of
oxygen in the blood whereas hypoxia is defined b! reduced level
of tissue oxygenation.
Hypoxia can be due to either defective delivery or defective
utilization of oxygen by the tissues.
Causes of hypoxemia
The inspired air has reduced oxygen content (e.g., at high altitude or
due to other causes).
Insufficient gas exchange is caused by alveolar hypoventilation.


26. Iodine is transported to the thyroid gland
by ?
a) Active transport
b) Diffusion
c) Passive transport
d) Pinocytosis
e) None
Correct Answer - A
Ans. is 'a' i.e., Active transport [Ref: Ganongjs 24th/e p. 3411]
Iodine uptake for thyroid hormone synthesis:
Iodine uptake mediated by thyroid follicular cells from the blood
plasma is the first step for the synthesis of thyroid hormones.
This ingested iodine is bound to serum proteins, especially to
albumins.
The rest of the iodine which remains unlinked and free in
bloodstream, is removed from the body through urine.
Basolateral membrane of thyroid cells (follicular cells) have active
transport system for iodine uptake - Na+ : I- symporter (NlS)
(Secondary active transport).
This trapping stimulated by TSH,

27. Which of the following are Pyrimidine bases?
a) Adenine and Guanine
b) Guanine and Cytosine
c) Cytosine and Adenine
d) Thymine and Cytosine
e) None
Correct Answer - D
Ans: (D)Cytosine and Adenine
In DNA and RNA, Pyrimidine bases form hydrogen bonds with
their complementary purines.
Thus, in DNA, the purines adenine (A) and guanine (G) pair up
with the pyrimidines thymine (T) and cytosine (C),
respectively. In RNA, the complement of adenine (A) is uracil (U)
instead of thymine (T), so the pairs that form are adenine: uracil
and
guanine: cytosine.
Two types of bases are found in nucleotides : (i) purines and (ii)
pyrimidines.
. Purines : Two major purine bases found both in DNAs as well
as RNAs are (i) adenine (A) and (ii) guanine (G).
. Pyrimidines : Three major pyrimidine bases are (i) cytosine (C),
(ii) Uracil (U) and (iii) Thymine (T). Cytosine and uracil are found
in RNAs and cytosine and thymine are found in DNAs. Uracil is
not found in DNAs Q and thymine is not found in RNAs.
Ref: Rodwell V.W. (2011). Chapter 32. Nucleotides. In D.A.
Bender, K.M. Botham, P.A. Weil, P.J. Kennelly, R.K. Murray, V.W.
Rodwell (Eds), Harper's Illustrated Biochemistry, 29e


28. Major contribution towards
gluconeogenesis is by?
a) Lactate
b) Glyerol
c) Ketones
d) Alanine
e) None
Correct Answer - D
D i.e. Alanine


29. Which of the following will be more
towards the negative pole in gel
electrophoresis?

a) 5 bp
b) 50 kbp
c) 150 bp
d) 550 bp
e) 50000 bp
Correct Answer - B:E
Ans. is 'b' i.e., 50 kbp & 'e' i.e. 50000 bp [Ref Various
internet
sites]

In gel electrophoresis, DNA molecules move towards positive end as
they are negatively charged themselves.
larger molecules (> 500 bp) move slowly as compared to smaller
molecules. So, larger the molecule, it is more towards the
negative end and vise verse .
Polyacrylamide gels are usually used for proteins, and have very
high resolving power for small fragments of DNA (5-500 bp). Aga
rose gels on the other hand have lower resolving power for DNA but
have greater range of separation, and are therefore used for DNA
fragments of usually 50-20,000 bp in size, but resolution of over 6 Mb
is possible with pulsed field gel electrophoresis (PFGE)" - wikipedia


30. Beta galactosidase deficiency causes?
a) Goucher disease
b) Krabbe's disease
c) Fabry's disease
d) Neimann Pick disease
e) Metachromatic leukodystrophy
Correct Answer - B
Ans. is 'b' i.e., Krabbe's disease [Ref Harper 30th/e p. 251
& 29thie p. 235]

Krabbe disease, also known as globoid cell leukodystrophy or
galactosylceramide lipidosis, is an autosomal-recessive
sphingolipidosis caused by deficient activity of the lysosomal
hydrolase galactosylceramide beta-galactosidase (GALC).
GALC degrades galactosylceramide, a major component of myelin,
and other terminal beta-galactose-containing sphingolipids,
including psychosine (galactosylsphingosine).
Beta-galactosidase is a lysosomal enzyme responsible for catalyzing
the hydrolysis of gangliosides. The deficiency of this enzyme can
lead to 1 of the following conditions: GM1 gangliosidosis, Morquio
syndrome B, and galactosialidosis.


31. Methods that can be used to see protein -
protein interaction include?
a) Fluorescence life imaging
b) Fluorescence resonance energy transfer
c) Fluorescence polarization
d) Fluorescence complementation
e) All of the above
Correct Answer - E
Ans. is 'e' i.e., All of the above [Ref wiki]
Biochemical methods
. Co-immunoprecipitation
. Bimolecular fluorescence complementation (BiFC)
Biophysical & theoretical methods
. Bio-layer interferometry
. Dual polarisation interferometry (DPI)
. Fluorescence polarization/anisotropy
. Fluorescence resonance energy transfer (FRET)
. Fluorescence lifetime imaging microscopy (FLIM) .


32. Uncouplers of oxidative phosphorelation
include?
a) 2, 4 - DNP
b) H2S
c) Cyanide
d) Thermogenin
e) Carboxin
Correct Answer - A:D
Ans. is 'a' i.e., 2, 4 - DNP & 'd' i.e., Thermogenin [Ref Harper
30thie p. 132 er 29th le p. 339; Vasudevan 6thIe p. 234, 235;
Chatterjea Shinde 7th/e p. 132; Lippincott Ole p. 79]

Uncouplers block the coupling of oxidation with phosphorylation.
These compounds allow the transfer of reducing equivalents in
respiratory chain but prevent the phosphorylation of ADP to ATP
by uncoupling the linkage between ETC and phosphorylation.
Uncouplers may be :?
. Natural :- Thermogenin, thyroxine, long chain FAs
. Synthetic :- 2, 4-dinitrophenol (2, 4-DNP), 2, 4-dinitrocresol (2,
4-DNC), and CCCP.
Thermogenin is an uncoupler protein present in mitochondria of
brown adipose tissue (brown fat).
It uncouples oxidation and phosphorylation by acting as a channel
for H+ ions so that hydrogen ion gradient cannot build up


33. Pyridoxine is required for?
a) Decarboxylation
b) Carboxylation
c) Transamination
d) Transsulfuration
e) Oxidative deamination
Correct Answer - A:C:D
Ans. is 'a' i.e., Decarboxylation, 'c' i.e. Transamination & 'd' i.e.
Transsulfuration [Ref Harper's 30m/e p. 557 & 29th/e p.
536,
537]

. Transamination: PLP acts as coenzyme for transaminases.
. Decarboxylation: All decarboxylation reactions (by
decarboxylases) require PLP. Thus PLP is involved in generation
of important biogenic amines : GABA, Serotonin, Melatonin,
Histamine and catecholamines (epinephrine, norepinephrine).
In pyridoxin deficiency, 3-hydroxykinurenine accumulates and is
converted to alternate metabolite xanthurenic acid (xanthurenate).
Thus, xanthurenic acid (xanthurenate) excretion in urine is
increased in pyridoxine deficiency. Thus pyridoxin may be used in
xanturenic aciduria.


34. Amino acids derived from tissues are
directed towards?
a) Ammonia formation
b) Ammonium salts
c) Urea cycle
d) Urea formation
e) Amino acid pool of cells
Correct Answer - E
Ans. is 'e' i.e., Amino acid pool of cells [Ref: Harper 30"/e p.
298; Nutrition by Paul Insel, Don Ross, Kimberley
Mc?Mahon 4th/e p. 242]

When cells break protein, the protein's amino acids return to
circulation. These available amino acids, found throughout the body
tissues and fluids, are collectively referred to as amino acid pool.
The available amino acids will be utilized for protein synthesis.
Others may have their amino group removed and be used to produce
energy or non protein substances such as glucose.


35. Co-factors required for fatty acid
synthesis in human are?
a) ATP
b) NADPH
c) Biotin
d) Pyridoxine
e) Pentothenic acid
Correct Answer - A:B:C
Ans. is 'a' i.e., ATP, 'b' i.e. NADPH & 'c' i.e., Biotin [Ref Harper
30th le p. 236 er 29th/e p. 219; Lippincott 4'5/e p. 187]
pyridoxal phosphate is needed for elongation of already synthesized
fatty acids, not for synthesis itself.
Extramitochondrial (cytoplasmic) System is concerned with de novo
synthesis of fatty acid from acetyl CoA, and is present in cytosaol.
Palmitic acid is synthesized.
Cofactor requirements for fatty acid synthesis are NADPH, ATP,
Mn+2, biotin and HCO3- (as a source of CO2).
The major product of fatty acids synthesis is palmitate. Longer fatty
acids are formed by elongation reactions either in microsomes
(endoplasmic reticulum or in mitochondria.


36. All are True statements regarding Okazaki
fragment EXCEPT?
a) Requires DNA polymerase
b) Forms on leading strand
c) Forms on lagging strand
d) Requires helicase for opening
e) Requires RNA primer
Correct Answer - B
Ans. is 'b' i.e., Forms on leading strand [Ref Lippincott's 5thie p.
399, 401, 406; Harper's 30 le p. 383 & 250/e p. 367]
DNA polymerases responsible for copying the DNA templates are
only able to "read" the parental nucleotide sequence in 3' - 5'
direction, and they synthesize the new DNA strands only in 5' - 3'
direction. Therefore, 2 newly synthesized chains must grow in
opposite directions.
The DNA chain which runs in the 3' to 5' direction towards replication
fork as continued strand is called leading strand. This requires only
one RNA primer
The DNA chain which runs in the 5' to 3' direction away from the
replication fork is called lagging strand. It is synthesized
discontinuously and requires numerous RNA primers.
As the replication fork moves, RNA primers are synthesized at
specific intervals. These RNA primers are extended by
DNA
polymerase III into short pieces of DNA called
Okazaki fragments.



37. All are true regarding satellite DNA
EXCEPT?
a) Repeated DNA sequences in tandem
b) Clustered around centromere
c) Clustered around telomeres
d) Transcriptionally active
e) None of the above
Correct Answer - D
Ans. is 'd' i.e., Transcriptionally active [Ref Harper's 30"'/e p.
377-78; Lippincot 4t5 le p. 461]
Repetitive sequences in DNAis also called (satellite DNA)
These consist of 5-500 base pair lengths repeated many times.
These are often clustered in centromeres (central protein of
chromosomes where sister chromatids join each other) and
telomeres (repeated sequence at the end of chromosomes).
The majority of these sequences are transcriptionally
inactiveand play a structural role
microsatellite sequences most commonly are found as dinucleotide
repeats of AC on one strand and TG on the opposite strand.
Microsatellite repeat sequences consist of 2-6 bp repeated upto 50
times. The AC repeat sequences occur at 50000-100000 locations in
human genome.


38. True statement regarding t-RNA is?
a) Contains codon
b) Contains anti-codon
c) Contains blunt ends
d) Acts as a acceptor for amino acids in protein synthesis
e) Gets attached to ribosomes
Correct Answer - B:D:E
Ans. is 'b' i.e., Contains anti-codon, 'd' i.e. Acts as a acceptor
for amino acids in protein synthesis & 'e' i.e. Gets attached
to ribosomes

[Rep Lippincotese le p. 418; Harper's 30'Ve p. 394]
tRNA is the Smallest of the three major RNAS having 73 to 93
nucleotide residues. It comprises about 15% of total RNA in the
cell.
Acceptor arm consists of a base paired stem that terminates in
the sequence CCA at the 3' end. This is the attachment site for
amino acids. CCA tail is added during post-transcriptional
modification.
It contains anticodon that base pairs with the codon of coming
mRNA.Anticodon has nucleotide sequence complementary to the
codon of mRNA and is responsible for the specificity of the t RNA.
Through TC arm tRNA gets attached to ribosome.


39. Vitamin E deficiency in adult causes?
a) Hemolysis
b) Posterior column Involvement
c) Peripheral neuropathy
d) Hair loss
e) Impaired immunity
Correct Answer - A:B:C:D:E
Ans. is 'All' i.e., a, b, c, d & e [Ref : Harper's 30th/e p. 553 & 29th/e
p. 532, 541, 543]

vitamin E deficiency are characterized by axonal degeneration in the
posterior columns and a selective loss of large calibre myelinated
sensory axons in the spinal cord and peripheral nerves.
Subacute combined degeneration of spinal cord, also known as
Lichtheim's disease, refers to degeneration of the posterior
and lateral columns of the spinal cord as a result of vitamin
B12 deficiency (most common), vitamin E deficiency, and
copper deficiency.
Vitamin E activiy is present in several tocopherols, the most
important being a-, y- and 8- tocopherol. a-Tocopherol is the most
abundant and is taken as the standard.
Selenium and vitamin E supplement each other by their anti-oxidant
property.
Hemolytic anemia -due to oxidative damage to red blood
cells, Impairment of the immune response, Digestive problems &
Malabsorption leading to liver and pancreatic problems, Dry skin
and hair loss


40. Vitamin C deficiency is associated with?
a) Decreased immunity
b) Improper wound healing
c) Epistaxis
d) Seizures
e) Anemia
Correct Answer - A:B:C:E
Ans. is 'a' i.e., Decreased immunity, 'b' i.e., Improper wound
healing, 'c' i.e., Epistaxis & `e' i.e., Anemia [Ref Harper's 30th
le p. 561-65; Internet]

General symptoms include Low grade fever, irritability, tachypnea,
digestive disturbances, loss of appetite, weakness, weight loss,
vague myalgias and arthritis & arthralgias.
Anemia Due to defect in utilization of iron & folic acid.
Dermatological :- Dry skin, Follicular hyperkeratosis, coiled hair,
splitting of hair, Poor wound healing
Impaired immunity leading recurrent infections, Irritability and other
psychologic symptoms.


41. True about telomerase is?
a) DNA dependent RNA polymerase
b) RNA dependent DNA polymerase
c) Reverse transcriptase enzyme
d) Increased telomerase activity is seen in somatic cells
e) Telomerase increases the longevity of cells
Correct Answer - B:C:E
Ans. is 'b' i.e., RNA dependent DNA polymerase, 'c' i.e.
Reverse transcriptase enzyme & `e' i.e. Telomerase increases

the longevity of cells [Ref Harper 29th/e p. 358 & 28`"/e p. 315,
316; Robbin's 8th/e p. 40, 296]
Telomerase is a reverse transcriptase (RNA dependent DNA
polymerase) and is responsible for telomere synthesis and
maintaining the length of telomers (replication of end of
chromosome). Thus, telomerase provide longevity to the cells
which contain this enzyme.
Telomerase is absent from most of the somatic cells and hence they
suffer progressive loss of telomeres and they exit the cell cycle.
Senscent cells lack telomerase so their telomeres get shortened by
critical length and these cells remains in Go phase


42. Which is/are not transport protein?
a) Transferrin
b) Collagen
c) Ceruloplasmin
d) Hemoglobin
e) Albumin
Correct Answer - B
Ans.is'b'i.e., Collagen [Ref
http://www.gastrohep.com/ebooks/rodes/Rodes_2_4_1.pdf]
Collagen is a structural protein
Albumin has circulating transport proteins such as steroids,
thyroxine, triiodothyronine, fat soluble hormones, fatty acids to liver,
unconjugated bilirubin, many drugs, Calcium, magnesium, cations
& anions.
Ceruloplasmin has Copper
Hemoglobin Oxygen from lung to tisuues
Transferrin iron ions in the ferric form (Fe3+).


43. Full form of LCAT is ?
a) Lecithin cholesterol acyl transferase
b) Lecithin cholesterol alkyltransferase
c) Lecithin choline acetyltransferase
d) Lecithin choline alcohol transferase
e) Lecithin co A transferase
Correct Answer - A
Ans. is 'a' i.e., Lecithin cholesterol acyl transferase [Ref
Harper's 30`Ve p. 272; Lippincott 6th le p. 234]
Lecithin-cholesterol acyl transferase (LCAT) is present in HDL and
esterifies the cholesterol in HDL. Major activator of LCAT is Apo-
Al. Apo-C1 can also activate LCAT.
HDL-Cholesterol appears to be the best independent predictor of
coronary artery disease (inverse relationship) than any other known
risk factor. That means low HDL is a much stronger predictor of
coronary artery disease than increased LDL cholesterol or
increased total cholesterol
The intracellular cholesterol activates the intracellular enzyme acyl-
CoA cholesterol acyl transferase (ACAT). This enzyme catalyzes
transfer of an acyl group from a fatty acid derivative to cholesterol,
resulting in the formation of esterified choleserol, and this
cholesterol ester is stored for subsequent use.


44. Which organ cannot use ketone bodies ?
a) Brain
b) RBC
c) Muscle
d) Heart
e) Liver
Correct Answer - B:E
Ans. is 'b' i.e., RBC & `e' i.e., Liver [Ref Harper's 30th/e p.
227;
Vasudevan ele p. 145]

Liver itself cannot utilize ketone bodies as it lacks the enzyme CoA-
transferase which is required for activation of ketone body .
Beside liver, RBCs also do not utilize ketone bodies (only glucose is
the sole fuel for RBCs).
Acetoacetate and p-hydroxybutyrate are used in preference to
glucose as energy source by certain tissues, e.g. heart,
muscle, intestinal mucosa and renal cortex. Brain also
switches to using predominantly acetoacetate in starvation.
In extrahepatic tissues, acetoacetate is activated to acetoacetyl CoA
by succinyl-CoA-acetoacetate CoA transferase (thiophorase).


45. True regarding phenylketoneurea is?
a) Musty order is due to phenylalanine in sweat
b) Deficient enzyme is phenylalanine hydroxylase
c) Autosomal dominant
d) May be associated with impaired mental development
e) Infants are normal at birth
Correct Answer - B:D:E
Ans. is 'b' i.e., Deficient enzyme is phenylalanine
hydroxylase,'d' i.e. May be associated with impaired
mental development & 'e' i.e. Infants are normal at birth

[Ref Harper's 30th/e p. 304 & 29m/e p. 288; Chatterjee 5th/e p. 426]
In Phenylketonuria there is inability of oxidation of phenylalanine into
tyrosine. There is defective function of phenylalanine hydroxylase.
Toxic levels of phenylalanine (and insufficient levels of tyrosine) can
interfere with infant development in ways which have permanent
effects.
The disease may present clinically with seizures, hypopigmentation
and a "musty odor" to the baby's sweat and urine (due to
phenylacetate, a carboxylic acid produced by the oxidation of
phenylketone).
Untreated children develop microcephaly, and demonstrate
progressive impairment of cerebral function which can lead to
intellectual disability, behavioral problems, and mental
disorders.


46. Examples of chaperon include all except?
a) Calreticulin
b) Calnexin
c) Calbindin
d) BiP
e) Ubiquitin
Correct Answer - C:E
Ans. is 'c' i.e., Calbindin & `e' i.e. Ubiquitin [Ref Harper's 30th /e
p. 609; Vasudevan 5thle p. 17]
Chaperones are present in a wide range of species from bacteria to
humans. Many so called 'Heat shock proteins' (HSP) are
chaperones. They are also known/as stress proteins.
Some Chaperones and Enzymes Involved in Folding that are
Located in the Rough Endoplasmic Reticulum are BiP
(immunoglobulin heavy chain binding protein), GRP94 (glucose-
regulated protein), GRP-170, GRP-78, Calnexin, Calreticulin
PDI (protein disulfide isomerase), PPI (peptidyl prolyl cis-trans
isomerase), HSP47, ERp29


47.


Following cells are part of innate immunity?
a) B-cells
b) T-cells
c) NK-cells
d) Macrophages
e) Dendritic cells
Correct Answer - C:D:E
Answer- C, D, E, NK-cells, Macrophages, Dendritic
cells Important components of innate immunity are :-

. Cells : Phagocytic cells (macrophages, neutrophils), dendritic
cells, NK cells, eosinophils, mast cells, basophils, epithelial cells
(forming epithelial barrier).
. Complement component antimicrobial peptides
. Pattern recognition receptors (PRn)
There are two types of PRR :
. Soluble PRR (Mannose recePtors, C-reactive protein):
. Surface PRR (Scavenger receptors on macrophages, Toll-
like receptors).


48. Which of the following are type 3
hypersensitivity reactions?
a) Good Pasteur syndrome
b) Serum sickness
c) Arthus reaction
d) Asthma
e) Rheumatoid arthritis
Correct Answer - B
Answer- B. Serum sickness
Local-Arthus reaction
Systemic-serum sickness
Schick test
Polyarteritis nodosa (PAN)
Rheumatoid arthritis
SLE
Acute viral hepatitis
Penicillamine toxicity
Hyperacute graft rejection
Type 2 lepra reaction (ENL)
Hypersensitivity pneumonitis
Infective endocarditis
Henoch schonlein purpura
Glomerulonephritis


49. True regarding Down syndrome is?
a) Increased paternal age is a risk factor
b) Karyotyping is not needed in all patients
c) > 85% of affected patients have 1 more chromosome 21
d) Increased nuchal translucency
e) Associated with early onset of Alzheimer's disease
Correct Answer - B
Answer- B. Karyotyping is not needed in all patients
Down's syndrome is the most common chromosomal disorder and
most common congenital cause of mental retardation (2nd
most common genetic cause of mental retardation is Fragile -X
sydrome).
Trisomy 21- There is an extra chromosome 21 which is due to
meiotic nondisjunction in ovum.
The most important risk factor is advanced maternal age (> 35
Years).
Antenatal Screening for Down syndrome
Following methods are used :-
. Triple test It includes (i) Unconjugated estrogen (estriol):
decreased; (ii) Maternal serum alphafeto protein (MSAFP)
:decreased; and (iii) hCG: increased
. New markers: These are (i) Increased inhibin A in maternal blood;
and (ii) Decreased PAPA (pregnancy associated plasma protein).
. USG: It shows : (i) Increased nuchal translucency (increased nuchal
fold thickness); (ii) Ductus venous flow reversed; and (iii) Nasal
bone hypoplasia.


50. A 14 year old boy presented with
hereditary spherocytosis. Which of the
following indices is/are increased?

a) LDH
b) MCHC
c) MCV
d) Urine urobilinogen
e) Haptoglobin
Correct Answer - A:B:D
Answer- A, B, D LDH, MCHC, Urine urobilinogen
MCV decreased
MCHC increased
LDH increased


51. As compared to iron deficiency anemia,
which of the following is decreased in
anemia of chronic disease?

a) Endogenous bone marrow iron stores
b) Serum ferritin
c) Transferrin saturation
d) TIBC
e) MCV
Correct Answer - D
Answer- D. TIBC
MCV/MCH- Decreased or normal
Serum iron- Decreased
TIBC- Decreased, normal
Transferrin saturation- Decreased
Serum ferritin- Normal or increased


52. Antibody which is/are specific for SLE?
a) ANA
b) Anti-ds DNA
c) Anti-Sm
d) Anti-histone
e) Anti-RNP
Correct Answer - B:C
Answer- B, C Anti-ds DNA, and Anti-Sm
These are the most specific antibodies for SLE.


53. Microcytosis can be seen in deficiency
of?
a) Iron
b) Folic acid
c) Vitamin B12
d) Vitamin C
e) Vitamin B6
Correct Answer - A:D:E Answer- A, D, E,
Iron, Vitamin C, Vitamin B6 Seen in-

. Iron deficiency anemia (most common cause of anemia in
general and of microcytic anemia in particular)
. Thalassemia trait
. Other hemoglobinopathies such as hemoglobin C syndrome
& hemoglobin S syndrome
. Chronic inflammation
. Anemia of chronic disease
. Siderobastic anemia
. Deficiencies - Pyridoxin (Vit 86), vitamin C and copper


54. Blood tests done to see liver functions
include?
a) Bilirubin
b) Uric acid
c) Alanine transaminase
d) Urea
e) Albumin
Correct Answer - A
Answer- A. Bilirubin
. Aspartate aminotransferase/ SGOT
. Alanine transaminase/ SGPT
. Alkaline phosphatase
. Gamma- Glutamyltransferase
. Conjugated bilirubin
. Unconjated bilirubin
. Albumin


55. Anaplasia is malignant tumor may lead
to?
a) Change in nuclear size
b) Loss of cell polarity
c) Metaplasia
d) Increased mitosis
e) Malignant transformation
Correct Answer - A:B:D:E
Answer- A, B, D, E, Change in nuclear size, Loss of cell
polarity, Increased mitosis, Malignant transformation
Anaplastic cells show following features :-

. Loss of polarity
. Increased nuclear cytoplasmic size ratio
. Increased number of mitosis which is atypical
. Hyperchromatosia
. Pleomorphbm


56. During inflammation, mediators involved
in adhesion and movement include?
a) Compliment 5a
b) Leukotriene B4
c) Integrins
d) IL-8
e) L-selectin
Correct Answer - C:E
Answer- C & E, Integrins, L-selectin
Adhesion
Ligand on Extravasation
molecules
Cells
endothelial stage
cells
Naive T
GlyCAM-1,
L-selection
lymphocytes,other
CD34,
Tethering/Rolling
(CD62L)
leukocytes
MadCAM-1
E-selection
(CD26E),
PSGL-1
Neutrophils
Tethering/Rolling
P-Selectin
(CD62P),
Activated T
ICAM-1
LFA-1
lymphocytes,other
(CD54),
(2 Integrin
Tight adhesion
leukocytes, other
ICAM-2
CD11A/CD18) leukocytes
(CD102)
Activated
VLA-4
T leukocytes
VCAM-
(1 Inegrin,
monocytes,
1(CD106),
Tight adhesion

CD49d/CD28)
neutrophils,
Fibronection
eosiophils, basophils
Neutrophils,
ICAM-
Mac-1
Monocytes,
1,iC3b, Tight adhesion
(CD11b/CD18)
Macrophages fibronection
VCAM-1
LPAM-1 (7 Effector T
MAdCAM-1 adhesion
integrin)
lymphocytes
fibronection


57. Causes of unconjugated
hyperbilirubinemia include?
a) Sepsis
b) Criggler-Najar syndrome
c) Rotor syndrome
d) Gilbert syndrome
e) Intravascular hemolysis
Correct Answer - A:B:D:E
Answer- A, B, D, E, Sepsis, Criggler-Najar syndrome,
Gilbert syndrome, Intravascular hemolysis Unconjugated
hyperbilirubinemia:-

Increased production of bilirubin from hemoglobin, So that the
capacity of liver to conjugate bilirubin is overwhelmed by
increased production, e.g.
. Hemolytic anemia (both intravascular and extamascular)s Hereditary
sphnocytosis, G6PD defciency.
. Inefrective erythropoiesis- Thalassemia, Pernicious anemia.
. Reduced hepatic uptake of bilirubin from bilirubin - albumin
complex > Drugs,
. Infections:- Sepsis, UTI
. Impaired hepatic conjugation.


58. Major histocompatibility complexes are
found on which cells?
a) Dendritic cells
b) Basophils
c) Eosinophils
d) T cells
e) RBCs
Correct Answer - A:B:C:D
Answer- A, B, C, D, Dendritic cells, Basophils, Eosinophils,
T cells

HLA complex consists of three separate clusters of genes
:-
1) Class I (MHC-I)
Important cells with MHC-I (HLA-I) on surface are B-cells, T-cell,
macrophages /monocytes, neutrophils, langerhans cells, dendritic
cells, platelets (thrombocytes), epithelial cells of thymus and
hepatocytes. MHC class I present antigen to cytotoxic CD-8 T
cells. 2) Class II (MHC-II)
It comprises 'D' region (HLA-DR,HLA-DQ, HLADP).It is found only
on the cells of immune system, i.e.T-cells,B-cells,langerhans
cells, dendritic cells, and macrophages.
3) Class III (MHC-III)
Tumor necrosis factor- alpha and beta (TNF-alpha and beta).


59. Which of the following is NOT TRUE about
mutation of p53?
a) Cell will continue to multiply
b) Cancer formation
c) Cell cycle will be arrested
d) DNA repair
e) All of the above
Correct Answer - C:D
Answer- C & D, Cell cycle will be arrested, DNA
repair
p53 is a tumor suppressor gene.
The major functional activities of the p53 protein are cell cycle
arrest and initiation of apoptosis in response to DNA damage.
p53 causes-
. Cell cycle arrest- there is arrest of cell cycle late in G1 phase.
This allows time for DNA repair.
. DNA repair- GADD 45 encodes a protein that is involved in
DNA repair.
p53 induces apoptosis.
Mutation in p53 leads to loss of above protective mechanisms i,e.
cell cycle arrest & DNA repair. It will lead to unarrested cell
multiplication and finally carcinogenesis.
Non-mutated (wild type) p53 reduces the chances of cancer.


60. Correct dyad of disease and their
respective inheritance pattern include?
a) Wilson disease - autosomal recessive
b) Cystic fibrosis - autosomal dominant
c) Marfan syndrome - autosomal recessive
d) Gardner syndrome - autosomal dominant
e) Duchene muscular dystrophy - X-linked recessive
Correct Answer - A:D:E
Answer- (A) Wilson disease - autosomal recessive (D) Gardner
syndrome - autosomal dominant (E) Duchene muscular
dystrophy - X-linked recessive
Autosomal recessive disorders
1) Metabolic - Cystic fibrosis, Phenyl ketonuria,
Galactosemia, Homocystinuria, Lysosomal storage dis, alpha
l-antitrypsin deficiency,
Wilson disease, Hemochromatosis, Glycogen storage disorders.
Autosomal dominant disorders
1. GIT- Familial polyposis coli, Gardner's syndrome
2. Skeletal - Marfan syndrome
1) Musculoskeletal - Duchene muscular dystrophy,
Becker's dystrophy


61. Psammona bodies is/are seen in -
a) Medullary ca of thyroid
b) Ependymoma
c) Papillary ca of thyroid
d) Follicular ca of thyroid
e) Meningioma
Correct Answer - C:E
Answer- C, E, Papillary ca of thyroid, Meningioma
A psammoma body is a round collection of calcium, seen
microscopically. The term is derived from the Greek word
psammos meaning "sand." Psammoma bodies are commonly
seen in certain tumors such as:
Papillary thyroid carcinoma
Papillary renal cell carcinoma
Serous papillary ovarian adenocarcinoma (cystadenocarcinoma)
Endometrial adenocarcinomas (Papillary serous carcinoma ?
3%-4%)
Meningioma
Mesothelioma
Psammoma bodies usually have a laminar appearance.


62. Wound healing is affected by -
a) Age
b) Nutrition
c) Dryness of wound
d) Drugs
e) Temperature
Correct Answer - A:B:C:D:E
Answer- A, B, C, D, E, Age, Nutrition, Dryness
of wound, Drugs, Temperature

Intrinsic factor
Health status eg: diabetes
Age tactors
Body buiJd
Nutritional status (Drotein deficiencv.
Vitamin C deficiency)
Inadequate blood supply
Extrinsic factor-
. Temperature
. Desiccation and maceration
. Infection (single most important factor)
. Chemical stress
. Medications eg; corticosteroids


63. All are major criteria for rheumatic fever
except:
a) Pancarditis
b) Chorea
c) Arthritis
d) Subcutaneous nodules
e) Fever
Correct Answer - E
Answer- E. Fever
Chorea, Arthritis and Carditis are major criteria for diagnosis of
Rheumatic fever Fever is a minor criteria
Erythema Marginatum is a major criteria and not Erythema
nodosum.


64. Increase PT is seen with -
a) Warfarin administration
b) Factor V deficiency
c) Factor VIII deficiency
d) Factor IX deficiency
e) Vit K deficiency
Correct Answer - A:B:E
Answer- A, B, E, Warfarin administration, Factor
V deficiency, Vit K deficiency

. Bleeding time- Prolongation generally indicates the defect in
platelet number or function.
. Partial thromboplastin time (PTT)- A prolonged PTT V, VIII (factor
VIIIc, Von wille brand factor, IX. X, XI, XII, prothrombin or fibrinogen.
. Prothrombin time(PT)- PT can results from deficiency of factor V, VII,
X, prothrombin or fibrinogen V, VII, X, prothrombin or fibrinogen.
. Thrombin time- elevated in fibrinogen deficiency.
. Vitamin K deficiency also cause prolongation of both PT and
aPTT as it inhibits factor II, VII, IX and X.


65. True in sickle cell anemia -
a) Splenomegaly
b) Microcytosis
c) Microcardia
d) Autosplenectomy
e) Gamma gandy bodies
Correct Answer - A:D:E
Answer- A, D, E, Splenomegaly, Autosplenectomy, Gamma
gandy bodies
Chronic hemolysis
Vasoocclusive symptoms
. Painful bone crisis
. Hand-foot syndrome Dactylitis of bones of hands/feet.
. Autosplenectomy
. Acute painful enlargement of spleen
. There maybe cardiomegaly and leukocytosis.
. Gamma Gandy bodies
There is no microcytosis in sickle cell disease.


66. Microscopy which can be performed with
minimum optical illumination -
a) Dark field
b) Bright field
c) Phase contrast
d) Confocal
e) None
Correct Answer - A:D Answer- A &
D, Dark field, Confocal

The light has to be reduced while using dark field & phase contrast
microscope, other microscopes use full illumination.
Dark field microscopy-
Uses a carefirlly aligned light source to minimize the quantity of
directly transmitted light entering the image plane, collecting only
the light scattered by the sample.
Confocal microscopy/confocal laser scanning microscopy/
laser confocal scanning microscopy-
Uses a scanning point of light and a pinhole to prevent out of focus
light from reaching the detector.


67. Acanthocytes are seen in?
a) Abetalipoproteinemia
b) Severe liver disease
c) Patients with Macleod blood group
d) SLE
e) Hyperprolactinemia
Correct Answer - A:B:C
Answer- A, B, C, Abetalipoproteinemia, Severe
liver disease, Patients with Macleod blood group

Acanthocytes or spur cells, are abnormal erythrocftes which are
spiculated with a few spiny or thorny projections of cytoplasm
of varying size and surface distribution.
The most frequent and most significant conditions with
acanthocytosis include abetalipoproteinemia.
McLeod red cell Phenotype.


68. Which of the following drug is not used in
the treatment of mucormycosis?
a) Fluconazole
b) Voriconazole
c) Posaconazole
d) 5-flucytosine
e) Amphotericin B
Correct Answer - A:B:D
Ans. is'a' i.e., Fluconazole,'b' i.e., Voriconazole &'d' i.e.,
5-flucytosine

[Ref: Goodman 6 Gilman 1l,h/e p. 1254; KDT Vh/e p.
795; www.ncbi.nlm.nih.gov)
Amongst azole, only posaconazole is active against mucormycosis
Antifungal treatment of mucormycosis:
First-Line Monotherapy:
Mucormycosis is a serious infection and needs to be treated
with.prescription antifungal medication, usually amphotericin B
(given through an IV), posaconazole (given through an IV or
orally) or isavuconazole (given through an IV or orally).
Fluconazole, voriconazole, and itraconazole do not have reliable
activity against mucormycosis.
5-flucytosine:
Flucytosine is not employed as the sole therapy except occasionally
in chromoblastomycosis.
It is used in limited to the treatment of cryptococcal meningitis, in
conjunction with AMP-B.




69. Drugs contraindicated in myasthenia
gravis include?
a) Neostigmine
b) Neomycin
c) Edrophonium
d) Atropine
e) Paracetamol
Correct Answer - B:D
Ans. is 'b' i.e., Neomycin &'d' i.e. Atropine
Ref: KDT Vh/e p. 110 &/e p. 104; Katzung 1Ltu/e p. 107; internet
Aspirin, non-steroidal anti-inflammatory drugs (NSAIDs) such as
ibuprofen and naproxen, and acetaminophen (paracetamol) are
considered safe for MG, that is, they have not been shown
to worsen MG or cause muscle weakness.
Neostigmine is used in myasthenia gravis.
Edrophonium can be used for diagnosis of myasthenia gravis as
tensilon test (for diagnosis of cholinergic crisis).


70. Drugs which have both a & receptor
activity include?
a) Epinephrine
b) Nor-epinephrine
c) Phenylephrine
d) Dopamine
e) Isprenaline
Correct Answer - A:B:E
Ans. is 'a' i.e., Epinephrine, 'b' i.e. Nor-epinephrine & 'd'
i.e. Dopamine

Ref: Goodman and Gilman's 12th/e p. 812
Nor-adrenaline has mainly alpha action with slight effect on cardiac
beta 1 receptors (alpha & beta 1 action).
Adrenaline has nonselective action on both alpha (alpha 1 + alpha
2) & beta (beta 1 + beta 2) receptors.
Phenylephrine is a selective alpha 1 agonist and has negligible beta
action
Dopamine is a dopamine (D1 & D2) as well as adrenergic alpha and
beta 1 agonist (Not beta 2).
Isoprenaline has beta (beta 1 + beta 2 action), but no alpha action.


71. Side effects of tricyclic antidepressants
include?
a) Diarrhea
b) Weight loss
c) Hypertension
d) Tremers
e) Urinary retention
Correct Answer - D:E
Ans. is'd'i.e., Tremors &'e'i.e., Urinary retention [Rel
KDT Vh/e p. 459; Goodman & Gilman llth/e p. 448]
Adverse effects of TCAs:
Anticholinergic- Dry mouth, bad taste, urinary retention, blurred
vision, palpitations, constipation.
Sedation, mental confusion, weakness.
Increased appetite and weight gain.
Sweating and fine tremor.


72. Dopamine at the dose of 8?g/kg/min
produces?
a) Increased systemic vascular resistance
b) Decreased systemic vascular resistance
c) Renal vasodilatation
d) Increased stroke volume
e) Increased heart rate
Correct Answer - A:C:D:E
Ans. is 'a' i.e., Increased systemic vascular resistance, 'c'
i.e. Renal vasodilation, 'd' i.e. Increased stroke volume &
'e'i'e..Increased heart rate

[Ref: Modern pharmacology with clinical application 2d/e p.
208; KDT Vh/e p. 426; Katzung Llth/e p. 139)
Dopamine:
It is a dopamine (D1 & D2) as well as adrenergic alpha & beta 1
agonist (not beta-2) .
The D1 receptors in renal and mesenteric blood vessels are the
most sensitive.
I.v. infusion of low dose (1-5 microgram/kg/min) of dopamine dilates
these vessels by raising intracellular cAMp.
Moderately high doses produce a positive ionotropic effect (direct
beta 1 & D 1 action + due to NA release), but little chronotropic
effect on heart ).
The advantage of this greater inotropic effect (increased force of
contraction) than chronotropic effect (increased heart rate) of
dopamine is that it produces smaller increase in oxygen demand
by


the heart.
Large doses (>10 micro gram/kg/min produce vasoconstriction
(alpha 1 action).
At high doses, it is called inoconstrictor because it has inotropic and
vasoconstrictor effect.


73. Diuretic drugs which are carbonic
anhydrase inhibitors include?
a) Acetazolamide
b) Spironolactone
c) Furesemide
d) Chlorthiazide
e) Topiramate
Correct Answer - A:E
Ans. is'a' i.e., Acetazolamide &'e'i.e. Topiramate
Ref: KDT 7/e p. 5g7, 420, 155; Katzung 13h/e p. 255]
Carbonic anhydrase inhibitors (acetazolamide):
Carbonic anhydrase (CAse) is an enzyme which catalyzes the
reversible reaction of bicarbonate formation.
The enzyme is present in renal tubular cells (specialty PT), gastric
mucosa, exocrine pancreas, ciliary body of the eye, brain and
RBC,
Acetazolamide, dichlorphenamide, methazolamide and topiramate
are carbonic anhydrase inhibitors.
Dorzolamide & Brinzolamide is also a carbonic anhydrase inhibitors
specific for CA-II.
Carbonic anhydrase inhibitors act by a non-competitive, reversible
inhibition of the enzyme carbonic anhydrase.
As inhibition is reversible, action of carbonic anhydrase inhibitors is
self limiting.


74. Antiplatelet drugs with adenosine
receptor inhibition property include?
a) Ticlopidine
b) Clopidogrel
c) Prasugrel
d) Abciximab
e) Cilastazole
Correct Answer - A:B:C
Ans. is'a' i.e., Ticlopidine,'b' i.e. Clopidogrel &'c' i.e. Prasugrel
Ref: KDT Vh/e p. 629 & 6h/e p. 609; Katzung lLth/e p. 598
ADP mediated platelet activation through cAMP is inhibited by
(antagonism of P2 Y12) receptors on ADP :-
Irreversible : Ticlopidine. clopidogrel. prasugrel
Reversible; Cangrelor, ticagrelor


75. Drugs with first order kinetics show?
a) Increased clearance with increase in concentration
b) Decreased elimination with concentration
c) No relation rate of elimination and concentration
d) Constant fraction of drug is eliminated per unit time
e) Half life remains constant
Correct Answer - A:D:E
Ans. is.a, i.e., Increased clearance with increase in
concentration, 'd' i.e. constant fraction of drug is eliminated
per unit time &'e' i.e. Half life remains constant

[Ref: KDT p. 31; Katzung 13th/e p 48]
Order of kinetics:
The rate at which elimination take place is subjected to important
influences that are referred to as order of kinetics
There are two orders of such elimination:
First order kinetic (Linear Kinetics)
Second order kinetic (Non-Linear Kinetics)
In first order kinetic,
Rate of elimination directly proportional plasma concentration.
Clearance remains constant because, as the plasma concentration
Increases the rate of elimination increases proportionately (CL =
rate of elimination / Plasma Conc).
HaIf life remains constant because time required to reduce the
plasma concentration to half is same (rate of elimination).


76. Drugs which are used in acute asthma
include?
a) Budesonide
b) Terbutaline
c) Salbutamole
d) Theophylline
e) Sodium cromoglycate
Correct Answer - B:C:D
Ans. is'b'i.e., Terbutaline,'c'i.e. Salbutamole &'d'i.e.
Theophylline
[Ref: KDT Vh/e p. 223]
Treatment of acute asthma:
The only drugs effective for the treatment of acute attack of asthma
are bronchodilators (beta 2-receptor agonists, anticholinergics,
and methylxanthines).
Mild attacks:
For patients with mild attack inhalation of a short acting beta-2
receptor agonist, e.g. salbutamol (albuterol), terbutaline is
used.
An inhaled anticholinergic, e.g. ipratropium may be added if there
is no satisfactory response to beta 2- agonists alone.
In patients who are refractory to inhaled therapies, i.v. aminophylline
(theophylline) may be effective.
Severe attacks:
Oxygen phts continuous administration of aerosolized salbutamol
(albuterol) plus systemic steroids, e.g. methylprednisolone,
hydrocortisone.

Recently, MgSO4 has been tried in acute severe asthma by IV or
inhalation route.


77. Antiviral drug (s), which is/are a pro-drug?
a) Acyclovir
b) Ganciclovir
c) Ziduvudin
d) Tenofovir
e) Foscarnet
Correct Answer - A:B:C
Ans. is 'a' i.e., Acyclovir, 'b' i.e. Ganciclovir &'c' i.e.
Zidovudine
[Ref. KDT 7/e p. 22 & p. 24; Goodman Gillman p.
534: Bennet Brown clinical pharma p. 404; Katzung p. 361, 385;
ww.ncbi.nlm.nih.gov]
Anti-herpes virus: Acyclovir, Valacyclovir. penciclovir, Famciclovir,
Garciclovir, valganciclovir.
Anti-Retrovirus: Nucleoside reverse transcriptase inhibitors
(NRTIs) - Zidovudine, Didanosine. Zalcitabine, Stavudine,
amirudin, Abacavir.
Foscamet is unrelated to any nucleic acid precursor, so does not
require phosphorylation for activation.
Tenofovir is a nucleotide and does not require bioactivation by
kinases.
Oral bioavailability of tenofovir increases with meals (decreased for
other NRTIs).


78. Antiandrogenic drugs include?
a) Danazole
b) Finesteride
c) Ketoconazole
d) Latrezole
e) Spironolactone
Correct Answer - B:C:E
Ans. is'b'i.e., Finasteride,'c'i.e. Ketoconazole &'e'i.e.
Spironolactone
[Ref: KDT Vh/e p. 858, 302; Harrison 18th/e p.802,803]
Antiandrogens drugs:
5-alpha-reductase inhibitors:
These drugs inhibit the enzyme 5-alpha-reductase which converts
testosterone into more active dihydrotestosterone.
These drugs are finasteride and dutasteride.
These are used in benign prostatic hypertrophy, hirsutism and rnale
pattern baldness.
Side effects are decreased libido, impotence, skin rash and swelling
of lips.
Note:-
Finasteride selectively inhibits type-2 5 alpha-reductase, whereas
dutasteride inhibits both type- 1 and type-2 5-alpha reductases.
Other drugs with antiandrogenic action are:
Superactive GnRH agonists, spironolactone, cimetidine,
progesterone and ketoconazole.


79. Antibiotics acting by inhibition of protein
synthesis include?
a) Penicillin
b) Vancomycin
c) Aminoglycoside
d) Fluroquinolones
e) Chloramphenicol
Correct Answer - C:E
Ans. is'c'i.e., Aminoglycoside, &'e'i.e. Chloramphenicol
[Ref: KDT 7/e p. 734; Katzung 13'h/e p. 789]
Inhibition of protein synthesis (translation):
Drugs acting by inhibiting protein synthesis are tetracyclines,
chloramphenicol. aminoglycosides. Erythromycin,
clindamycin, linezolid.
All protein synthesis inhibitors are bacteriostatic except
aminoglycosides and streptogramins which are
bactericidal.
These drugs can be divided into :
a) Based on the steps of translation (protein synthesis)
on which drug act :

Freeze initiation: Aminoglycosides.
Inhibit elongation : Tetracyclines, puromycin, chloramphenicol.
Inhibit translocation : Clindamycin, erythromycin.
Causing premature termination : Puromycin.
b) Based on ribosome on which they act :
30 S ribosome z Tetracycline, streptomycin.
50 S ribosome: Chloramphenicol, erythromycin, clindamycin,


Linezolid, pleuromutilins (retapamulin).
Both 305 and 505 ribosomes: Aminoglycoside (except
streptomycin).


80. First dose syncope is seen in ?
a) Alpha blocker
b) Beta blocker
c) CCB
d) ACE inhibitors
e) All of the above
Correct Answer - A:D
Ans. is'a'i.e., Alpha blocker &'d'i.e., ACE
inhibitors [Ref: KDT Vh/e p. 565 & 6h/e p. 546]
First dose hypotension (First dose phenomenon):
The first-dose phenomenon is a sudden and severe fall in blood
pressure that can occur when changing from a lying to a
standing position the first time that an alpha blocker drug is used
or when resuming the drug after many months off.
This usually happens shortly after the first dose is absorbed into the
blood and can result in syncope (fainting).
The alpha blocker prazosin is the most notorious for producing a first
dose phenomenon.
Other drugs of the same family, doxazosin and terazosin can also
cause this phenomenon, though less frequently.
Other drugs associated with it are ACE inhibitors, Sargramostim &
Muromonab, CD3.


81. True about benzodiazepine is:
a) GABA mimetic
b) GABA facilitator
c) Not a safe drug
d) High absue potential
e) Powerful enzyme inducer
Correct Answer - B
Ans. is'b'i.e., GABA facilitator
[Ref.: KDT Vh/e p. 401, 402 & 6th/e p. 393; Katzung ILth/e p. 375;
Goodman & Gilman 1Lth/e p.405]
Mechanism of action of benzodiazepines (BZDs):
Acts on GABA-A receptors.
BZDs receptor increase the conductance of Cl- channel.
BZDs do not themselves increase Cl- conductance, i.e. they have
only GABAfacilitatory but no GABA mimetic action. (Barbiturates
have both GABA facilitatory and GABA mimetic actions).


82. Punishment for insulting the modesty of a
woman comes under?
a) IPC 354
b) IPC 375
c) IPC 376
d) IPC 506
e) IPC 509
Correct Answer - A:E
Ans. is 'a' i.e.,IPC 354 & `e' i.e.,IPC 509 [Ref Reddy 30/e p.
377; https://indiankanoon.org]

IPC 354. Assault or criminal force to woman with intent to outrage
her modesty.--Whoever assaults or uses criminal force to any
woman, intending to outrage or knowing it to be likely that he will
thereby outrage her modesty, shall be punished with
imprisonment of either description for a term which may extend to
two years, or with fine, or with both.
IPC 509: Word, gesture or act intended to insult the modesty of a
woman.--Whoever, intending to insult the modesty of any woman,
utters any word, makes any sound or gesture, or exhibits any
object, intending that such word or sound shall be heard, or that
such gesture or object shall be seen, by such woman, or intrudes
upon the privacy of such woman, shall be punished with simple
imprisonment for a term which may extend to three year, or with
fine, or with both.
IPC 504. Intentional insult with intent to provoke breach of the
peace.

IPC 506. Punishment for criminal intimidation/


83. BAL is used as an antidote in the
poisoning of?
a) Arsenic
b) Cadmium
c) Mercury
d) Lead
e) Iron
Correct Answer - A:C:D
Ans. is 'a' i.e.,Arsenic ; 'c' i.e., Mercury & 'd' i.e.,Lead [Ref:
KDT 7thie p. 395; Katzung 12th/e p. 398, 1033]

Chelating agents are used in heavy metal poisoning. They form
complex with metal which is more soluble in water than the
metal itself, resulting in higher renal excretion of complex.
Dimercaprol (BAL) is used in poisoning by gold (Au), copper
(Cu), bismoth (Bi), nickel (Ni), arsenic (As), antimony (Sb)
and
mercury (Hg).
It is used as adjuvant to calcium disodium edetate in lead poisoning
and adjuvant to penicillamine in copper poisoning (or wilson's
disease).
Penicillamine is the drug of choice for copper poisoning and
wilson's disease. It is also used as an adjuvant to CaNa2, EDTA
in lead poisoning and to BAL in mercury poisoning.


84. Miosis is caused by?
a) Carbamates
b) Organophosphorus
c) Aconite
d) Dhatura
e) Cyanide
Correct Answer - A:B
Ans. is 'a' i.e., Carbamates & 'b' i.e., Organophosphorus [Ref:
Essentials of forensic medicine-786; Reddy 33'/e p. 619]
Poisoning causing miosis (constriction of pupil) : Carbolic acid
(phenol), chloral hydrate, chloroform, barbiturates (slightly
contracted and reacting to light), organophosphates,
carbamates,
opioids (morphine), mushroom poisoning,
nitrobenzene.
Poisoning causing mydriasis (dilatation of pupil) : Dhatura (atropine
or belladona), alcohol (Ma Ewan's sign), aconite, nux vomica,
ether, viper venom, cyanide, cocaine, chloroform, calatropis,
pethidine and barium carbonate


85. A patient with a recent history of
convulsions, presented to emergency in
subconscious state with blood pressure
60/90 mm/Hg, bradycardia & slow gasping
respiration. There is increased
lacrimation, salivation & sweating. On
examination there is pin point pupil.
Which of the following poisoning should
be suspected?

a) Opioids
b) Phenobarbitone
c) Organophosphorus
d) Dhatura
e) Strychinine
Correct Answer - C
Ans. is 'c' i.e., Organophosphorus [Ref Parikh 6Th /e p. 10.43;
Reddy 33`d% p. 5231
Organophosphorus poisoning compounds are irreversible inhibitors
of enzyme cholinesterase, an enzyme which hydrolyzes
acetylcholine.
Thus organophosphates inhibit cholinesterase and protect
acetylcholine from hydrolysis. This results in increased
concentration of acetylcholine and excessive cholinergic activity.

They inhibit cholinesterase by phosphorylating the catalytic site of
enzyme. Less than 50% of cholinesterase activity is indicative of
poisoning.
Clinical features of poisoning are due to excessive cholinergic
activity. Local muscarinic manifestations at the site of exposure
(skin, eye, GIT) occur immediately and are followed by
complex, systemic effects due to muscarinic, nicotinic and
central actions


86. Which of the following is NOT correct

about postmortem changes?
2. Post-mortem lividity fixes at 6-8 hours
3. Rigor mortis occurs when ATPs decrease upto 85% of normal
4. Rigor mortis is delayed in cholera and strychnine poisoning
5. Cadaveric spasm is instantaneous at the time of death
6. Postmortem caloricity occurs after 5-6 hours of death
Correct Answer - C:E
Ans. is 'c' i.e., Rigor mortis is delayed in cholera and
strychnine poisoning & `e' i.e., Postmortem caloricity occurs

after 5-6 hours of death [Ref Reddy 30th/e p. 137 -148]
Postmortem caloricity:- Normally, temperature falls after death. But
in some situations, for initial 2-3 hours the dead body may gain
heat, i.e. postmortem caloricity
After 6-12 hours, lividity is fully developed and fixed (unchangeable),
i.e. primary lividity. It ends when putrification sets in. Fixation of
lividity is due to stagnation of blood in distended capillaries and
venules (not due to coagulation of blood)
Rigor mortis occurs when there is decrease in ATP upto 15% (or
85% of normal) .
Cadaveric spasm (instantaneous rigor or cataleptic rigidity)
Instantaneous rigor is defined as the condition wherein a group of
muscles, which were in contracted state at the time of death,
continue to be in spasm after death, without the stage of primary
relaxation. Therefore, this stage preserves the attitude of the person
at the time of death.




87. Cephalic index in Indian population is?
d) 60-70
e) 70-75
f) 75-80
g) 80-85
h) 85-90
Correct Answer - B
Ans. is 'b' i.e., 70-75 [Ref Reddy 30'"/e p. 52]
Cephalic index is defined as the index of breadth of skull
Cephalic index = (Maximum breadth of skull/Maximum length of
skull) x 100
Based on cephalic index races are clasified into following three
3. Dolichocephalic (long headed) : Cephalic index is 70-75 and
is a feature of Caucasions, Pure Aryan (Indians), Aborigines,
and Negroes.
. Mesaticephalic (medium headed) : Cephalic index is 75-80 and is
a feature of Europeans, and chinese.
. Brachycephalic (short headed) : Cephalic index is more than 80
and is a feature of Mongolian.


.
According to the "Treatment of terminally-
ill patients bill, 2016", incompetent patient
is below the age of?

. 12 years
. 16 years
. 18 years
. 20 years
. 24 years
Correct Answer - B
Ans is'b'i.e.,16years[Ref-
http://www.prsindia.org/uploads/media/draft/Draft%20Passive%20
pdf]
"Incompetent patient" means a patient who is a minor below the age
of sixteen years or a person of unsound mind or a patient who is
unable to
4.
Understand the information relevant to an informed decision
about his medical treatment;
. Retain that information;
5. Use or weigh that information as part of the process of
making his informed decision;
. Make an informed decision because of impairment of or
a disturbance in the functioning of his mind or brain; or
. Communicate his informed decision (whether by speech,
language or any other mode) as to medical treatment


.
Which of the following fracture is specific
for indicating child abuse?

6. Posterior rib fracture
7. Scapula fracture
8. Mandibular fracture
9. Variable fractures at multiple sites
10.
Pelvic fracture
Correct Answer - A:B:D
Ans is 'a' i.e., Posterior rib fracture, 'b' i.e., Scapula fracture
& 'd' i.e., Variable fracturesatmultiplesites[Ref:http://
pediatrics.aappublications.org/content/133/2/e477#T1]

High specificity
Medium specificity
Low specificity
Multiple fractures in
various stages of
Metaphyseal lesion
("bucket
healing, especially
bilateral
handle" or "corner")
Clavicular fracture
Fractures of different
Posterior rib fracture
Long-bone shaft
ages
Scapular fracture
fracture
Epiphyseal separation
Spinous process
Linear skull fracture
fracture
Vertebral body
Sternal fracture
fracture/subluxation
Digital fracture
Complex skull fracture


90. Bolam test is related to?
. Medical negligence
. Contributory negligence
. Priviledged communication
. Negligence by patient
. Professional miscunduct
Correct Answer - A
Ans. is 'a' i.e., Medical negligence [Ref
http://medind.nic.in/jal/t07/0/jalt07i1p7.pdf]

The judgment given by Mr. Justice Mc Nair in Bolam vs. Frien hos
,ital management committee (19511 IS a landmark decision in
deciding cases of medical negligence and is known as the
"Bolam test.
Actions of doctors are to be judged by actions of other doctors
skilled in that particular art under similar circumstances and at
a material time.


7. Sin needle used to kill animals is made
of?
. Dhatura seeds
. Ratti seeds
. Lead peroxide
. Arsenic
. Strychnine
Correct Answer - B
Ans. is 'b' i.e., Ratti seeds [Ref Internet]
In the folk medicine, seeds are ground in to paste and made into
needles which are inserted under the skin of the animal. Thus
the animal will be poisoned for obtaining the skin.
Similar needles have also been used to produce criminal abortion"
Abrus precatorius, known as Indian licorice, ratti or gunja is a
slender, twining, climbing plant, woody at base and is found all
over the India. All the parts of this plant are poisonous.
Seeds were used only after they have been boiled in cow's milk for 3
hours. The herb was used for sciatica, stiffness of shoulder joint, for
baldness, dandruff and other hair diseases, for erysipelas and
obstinate skin diseases.


92. True about pugilistic attitude?
. Indicate only antimortem burn
. Indicate only postmortem burn
. Cannot differentiate between antemortem & postmortem burn
. Occur due to intense heat
. Indicate defence by victim during antemortem death
Correct Answer - C:D
Ans. is 'c' i.e., Cannot differentiate between antemortem
& postmortem burn & 'd' i.e., Occur due to intense heat

[Ref Parikh 6thle p. 4.156-4.157; Reddy 32"dle p. 307]
In body exposed to extensive heat (> 650 C), there results stiffening
and flexion of all joints and clawing of fingers, known as `Pugilistic
or boxer's or defense posture', due to denaturation and coagulation
of proteins.
Contraction of paranasal sinuses causes marked ophisthotonus.
However, heat stiffening is permanent (unlike rigor mortis which
passes off after few hours)
It is indicative of exposure to intense heat.
It can be mistaken for a pre-death attempt to shield oneself
attacker.
This phenomenon occurs both in antemortem and postmortem
burns. So difficult to differentiate between the two.


93. Gram negative cell wall contains?
8. Peptidoglycan
9. Lipopolysaccharides
10.
Lipids
11.
Teichoic acid
12.
All of the above
Correct Answer - A:C
Ans. is 'a' i.e., Peptidoglycan & 'c' i.e., Lipids
[Ref: Ananthanarayan 9th/e p. 15 & 8th/e p. 17; Bailey &
Scott's diagnostic microbiology p. 19]
Lipoploysacharide are found in the outer membrane of cell wall, not
in cell wall itself
The peptidoglycan layer of gram positive bacteria is thick and
contain teichoic acid, while in gram negative bacteria it is thin and
does not contain teichoic acid.
cell wall is composed of mucopeptide (peptidoglycan or murein)
scaffolding formed by N acetyl glucosamine and N acetyl
muramic acid molecules alternating in chains, which are cross
linked by peptide chains.
The outer membrane is a bilayered structure composed of
lipopolysaccharide.
Scattered throughout the lipopolysaccharide macromolecules are
protein structures called porins. These control the passage of
nutrients and other solutes, including antibiotics, through outer
membrane.


9. Which of the following organism(s) is/are
not cultivable?
10.
Mycobacterium leprae
11.
Klebsiella rhinoscleromatis
12.
Rhinosporidium seeberi
13.
Pneumocystis jiroveci
14.
None
Correct Answer - C:D
Ans. is 'c' i.e., Rhinosporidium seeberi, 'd' i.e.,
Pneumocystis jiroveci

[RefAnanthnarayan8th/ep.609-610;Harrison18thlep.1671;
https://www.hindawi.com/journals/isrn/2013/703813/;
https://en.wik rable]

The life cycle of pneumocystis probably involves sexual and asexual
reproduction, although definitive proof awaits the development of a
reliable culture system
Rhinosporidium seeberi has not been cultivated in media.
M. leprae is obligate intracellular organism, thus cannot be grown in
cell free culture medium. Two animals are used for cultivation.


95. Halophilic vibrios include?
11.
Vibrio cholare
12.
Vibrio parahymolyticus
13.
Vibrio vulnificus
14.
Vibrio alginolyticus
15.
Vibrio mimicus
Correct Answer - B:C:D
Ans. is 'b' i.e., Vibrio parahymolyticus, 'c' i.e. Vibrio vulnificus
& 'd' i.e. Vibrio alginolyticus [Ref Ananthanarayan 9th le p. 311;

Harrison 19' le p. 1065; Greenwood 16th /e p. 296]
Vibrios that have a high requirement of sodium chloride are known
as halophilic vibrios. They are abundant in coastal water
throughout the world.
All vibrios are halophilic except V cholerae and V mimicus.
Examples are - V. Parahaemolyticus, V. fluvialis, V. alginolyticus, V.
Hallisae, V. Vulnificus, V. furnissii, V. damsel.
Vibrio alginolyticus is the most salt tolerant (most halophilic) species
of vibrio.


96. Lymphatic filariasis is caused by?
f) Loa loa
g) Wuchreria bacrofti
h) Brugia malayi
i) Brugia timori
j) Onchocerca volvulus
Correct Answer - B:C:D
Ans. is 'b' i.e., Wuchreria bacrofti, 'c' i.e., Brugia malayi & 'd' i.e.,
Brugia timori [Ref: Harrison 19th/e p. 1418 & 17th/e p.1324,
Panikar 6th le p.196]
Filariasis is a parasitic and infectious tropical disease, that is caused
by filarial nematode worms.
Lymphatic filariasis : Wuchereria bancrofti, Brugia malayi,Brugia
timori.
Lymphatic filariasis is caused by Wauchereria bancrofti: Most
commonly(Bancroftian filariasis); and B. malayi & B. timori
(Brugian filariasis).
Man is the definitive host and mosquito is the intermediate host.
Microfilariae resides in the blood and adult worm in the lymphatics.
The inflammatory phase is characterized by Lymphangitis,
lymphadenitis and adenolymphangitis. It lasts for a few days, then
subsides spontaneously & recurs at irregular intervals for a period of
weeks to months.


97. Fever with rash is caused by?
f) Streptococcus pyogens
g) Staphylococcus aureus
h) Meningococcus
i) Vibrio cholera
j) Salmonella typhi
Correct Answer - A:B:C:E
Ans. is 'a' i.e., Streptococcus pyogens, 'b' i.e. Staphylococcus
aureus, 'c' i.e. Meningococcus &`e'i.e.Salmonellatyphi
[Refttps://www.infectiousdiseaseadvisor.com/infectious-
diseases/fever-a nd-rash/article/659555/;
https://www.ncbi.nlm. nih.gov/pmc/articles/PMC4607768/1

Vesiculobullous eruptions are caused by Scarlet fever, Streptococcal
toxic shock syndrome, Staphylococcal toxic shock syndrome,
Staphylococcal scalded-skin syndrome
Central distributed maculopapular eruption are caused by Scarlet
fever, Typhus, Rickettsial spotted fever, Ehrlichiosis, Typhoid
fever (S. typhi), Leptospirosis,


98. Larva in stool can be seen in?
13.
Echinococcus granulosus
14.
Ankylostoma duodenale
15.
Ascaris lumbricoides
16.
Strongyloides stercoralis
17.
Trichinella spiralis
Correct Answer - B:D:E
Ans. is 'b' i.e., Ankylostoma duodenale, 'd' i.e. Strongyloides
stercoralis & `e' i.e. Trichinella spiralis [Ref Chatterjee 12thie

p. 172; Panikar 7th/e p. 162; Rajesh karyakarte p. 121
Infective forms of helminthes in stool (larva)
f) Strongvloides stercoralis (filariform larva)
g) Ancylostoma duodenale (filariform larva)
h) Nector americans (filariform larva)
i) Trichinella spiralis (encysted larva)
Infective forms of helminthes in stool (EGGS)
j) T-solium
k) Echinococcus granulosus
l) H. Nana
m) Ascaris lumbricoidus(enibryonated ergs)
n) Enterobius (embryonated egg)
o) Trichuris Trichiura (embryonated egg)


f) Which of the following is false regarding
electron microscopy?
. There is arisk of radiation leakage
. Can magnify upto 30,000 times
. Both fixed and living specimens can be studies
. Vaccume is not required for proper functioning
. Black and white image
Correct Answer - C:D
Ans. is 'c' i.e., Both fixed and living specimens can be studies
& 'd' i.e. Vaccume is not required for proper functioning [Ref
Essentials of medical microbiology]



.
Which of the following is not associated
with Borrelia burgdorferi?

15.
Relapsing fever
16.
Lyme disease
17.
Vincent's angina
18.
Transmitted by ixodid tick bite
19.
Culture in BSK medium
Correct Answer - A:C
Ans. is 'a' i.e., Relapsing fever & 'c' i.e. Vincent's angina [Ref:
Ananthanarayan Whie p. 380 & 8thle p. 381]
Lyme disease : Caused by B. burgdorferi transmitted by the
bite of Ixodid ticks
Relapsing fever : Caused by B. recurrentis, B duttoni, B.
hermsii, B. Parkeri, B. turicatae, B. persica, B. hispanica.
Vincent's angina : Caused by B. vincenti.
The culture of B. burgdorferi in Barbour - Stoenner - kelly (BSK)
medium permits definitive diagnosis, but this complex method
has been used only research studies.



f)
Indications of use of antibiotics in acute
diarrhea include?

. Traveller's diarrhea
. Bacillary dysentery
. Caused by shiga like toxin producing E.coli
. Cholera
. All of the above
Correct Answer - A:B:D
Ans. is 'a' i.e., Traveller's diarrhea, 'b' i.e. Bacillary dysentery
& 'd' i.e. Cholera [Ref https://www.aafp.org/afp/2014/0201/
p180.html]

Bacillary dysentery is associated with species of bacteria from the
Enterobacteriaceae family. The term is usually restricted to
Shigella infections.
when used appropriately, antibiotics are effective for shigellosis,
cholera,
C.
difficile
(pseudomembranous
enterocolitis),
traveler's diarrhea, and protozoal infections.
If the patient's clinical presentation suggests the possibility of Shiga
toxin-producing E. coli (e.g., bloody diarrhea, history of eating
seed sprouts or rare ground beef proximity to an outbreak),
antibiotic use should be avoided because it may increase the risk
of hemolytic uremic syndrome.
In traveller's diarrhoea, antimicrobial therapy is unequivocally
effective; Traveller's diarrhoea is mainly due to bacterial
enteropathogens (approximately 80%), the most frequently
isolated being enterotoxigenic E coli; Quinolone antibiotics are
now the


treatment of choice


102. Pre-formed toxin is?
. Staphylococcus aureus exotoxin
. Emetic toxin of bacillus cereus
. Clostridium botulinum toxin
. Vibrio cholera toxin
. Shigella toxin
Correct Answer - A:B:C
Ans. is 'a' i.e., Staphylococcus aureus exotoxin, 'b' i.e.
Emetic toxin of bacillus cereus & 'c' i.e. Clostridium
botulinum toxin [Ref Essentials of Microbialogy p. 820,

Harrison MI p. 1088 & 17h/e p. 816, 817]
Enterotoxins are produced by : Shigella, B. cereus, Staphylococcus
aureus, CI perfringens, V. cholera, ETEC, EHEC, Cl difficle,
Yersinia enterocolitica, Rota virus.
Preformed toxins are produced by : Staph aureus, B. cereus, Cl
botulinum and Cl perfringens.
Heat labile toxins are produced by : Cl perfringens, LT of ETEC, V.
cholerae, Diarrheal form of B. cereus.
Heat stable toxins are produced by : Staph aureus, Y. enterocolitica,
ST of ETEC, emetic form of B. cereus.


16. Antibody present in antigen binding site
of B-cells?
f) Ig G
g) Ig M
h) Ig A
i) Ig D
j) Ig E
Correct Answer - B:D
Ans. is 'b' i.e., IgM & 'd' i.e. IgD [Ref Ananthanarayan 9thle p. 136
& 8thle p. 100] B-cells
B cells are part of adaptive immunity. B cells constitute 10% to 20%
of the circulating peripheral lymphocytes. B cells originate as well
as mature in bone marrow.
Ig M and Ig D, present on the surface of all naive B cells, constitute
the antigen binding component of B-cell receptor complex.
Combination of cell membrane bound Ig M or Ig D with the
corresponding antigen leads to specific stimulation of the B cells -
either activation and cloning to produce antibody, or suppression.


104. Viral hemorrhagic fever includes?
17.
Yellow fever
18.
West nile fever
19.
Lassa fever
20.
Ross fever
21.
Crimian - Congo fever
Correct Answer - A:C:E
Ans. is 'a' i.e., Yellow fever, 'c' i.e., Lassa fever & `e' i.e., Crimian
f) Congo
fever
[Ref
http://nasi.org.in i) Febrile group
(fever and myalgia)

This is the most common group which comprises a large number of
undifferentiated fevers, generally begin with or without rashes and
joint pain.
Important viruses in this group are :- Dengue, Chikungunya,West
Nile, Colorado tick fever, Sandfly fever and Sindbis.
ii) Encephilitis group
This group affects CNS and causes encephalitis or
meningoencephalitis.
Important virus in this group are :- California serogroup viruses
(California encephalitis, Jamestown canyon, Kesstone, La
Cross,Trivittatus), St Louis, JE, central European, Russian
spring summer, West Nile, Powassan, Eastern Equine, Western
Equine and Venezuela.
iii) Hemorrhagic group
Hemorrhagic group is associated with hemorrhage.
Important viruses in this group are :- Dengue, KFD, Chikungunya,
Yellow fever, Lassa, Crimean HE Congo, Omek, Rift valley,


Hantavirus, Marbung or Ebola.


105. True regarding odds ratio is/are?
f) Indicator of increased risk of disease in pre-disposed population
g) It is cross productivity ratio
h) Used in cohort study
i) Used in case control study
j) It is similar to relative risk
Correct Answer - A:B:D:E
Ans. is 'a' i.e., Indicator of increased risk of disease in pre-
disposed population, 'b' i.e., It is cross productivity ratio,`d'
i.e., Used in case control study & 'e' i.e., It is similar to relative

risk [Ref Park 24th le p. 78]
From a case control study odds ratio can be derived which a
measure of the strength of association between risk factor and
outcome. Indicator of increased risk of disease in pre-
disposed
population
Odds ratio is a key parameter in analysis of case control
studies.
Interpretation of Odds ratios (OR): Is similar to Relative risk
(RR) in cohort study (as OR is an estimate of RR)


.
Maximum chances of HIV transmission
are associated with?

. Receptive anal sex
. Insertive anal sex
. Receptive oral sex
. Insertive oral sex
. Vaginal sex [female to male]
Correct Answer - A
Ans.is'a'i.e.,Receptive anal
sex[RefHttp://www.aidsmap.com/HIV-risk-levels-for-the-
insertive-and-receptive-partner-in-different-types-of-sexual-
intercourse/page/14434904
HIV risk levels for the insertive and receptive partner in different
types of sexual intercourse
Anal intercourse has highest risk of transmission (anal intercourse >
vaginal intercourse > oral sex).
Unprotected anal intercourse carries a higher risk of sexual HIV
transmission than unprotected vaginal intercourse. Although
either sexual partner can acquire HIV from the other during
unprotected anal intercourse, HIV is more likely to pass from an
HIV-positive insertive partner to his receptive partner than from
an HIV-positive receptive partner to his or her insertive partner.


107. Vector born diseases are?
. Dengue
. KFD
. Japanese encephalitis
. Plague
. Yellow fever
Correct Answer - A:B:C:D:E
Ans. is 'All' i.e., a, b, c, d & e [Ref: Park 24`"/e p. 194 & 23rd/e
p.
185]



108. Screening tests should be?
f) Costly
g) Easy to perform
h) Difficult to perform
i) More specific
j) Less sensitive
Correct Answer - B:D
Ans. is 'b' i.e.,Easy to perform, 'd' i.e., More specific [Ref: Park
24th/e p. 149 & 23'ale p. 387; Modern epidemiology 3ra /e p. 204]
The disease should be an important health problem with a
recognizable latent or asymptomatic stage.
There should be a test (screening test) which can detect the disease
prior to clinical stage, with availability of a diagnostic (confirmatory)
test.
A test should have high sensitivity and specificity.
Accuracy =(Sensitivity) (Prevalence)/(Specificity) (1-Prevalence)
Accuracy = True positive + True negative/True positive + False
positive + True negative + False negative
Other important criteria are : Simplicity, rapidity, low cost (cost
effectiveness), safety, and ease of administration


109. Cash benefits in ESI scheme include?
f) Sickness
g) Medical
h) Maternal
i) Liability
j) Funeral
Correct Answer - A:B:C:E
Ans. is 'a' i.e., Sickness, 'b' i.e., Medical, 'c' i.e., Maternal &
`e' i.e., Funeral [Ref Park 24th/e p. 853 & 23"1/e p. 816]
Medical benefit: -905 Rs per capita.
Disablement benefit
Dependents benefit
Funeral expenses: -Rs. 5000 is given
Maternity benefit:- For confinement, the duration of benefit is 12
weeks. For miscarriage it is 6 weeks. For sickness arising out
of confinement it is 30 days.
Sickness benefit: The sickness benefit is payable for a maximum
period of 9ldays in any continuous period of 365 days.
TOTAL 34 different diseases are given importance such as, Mental
diseases (Psychoses), Chronic congestive cardiac failure,, Aplastic
anaemia, Monoplegia etc


110. True regarding frost bite is/are?
21.
Occurs at temperature below freezing point
22.
When on foot, known as trench foot
23.
On face it is superficial and severe from occur on extrimities
24.
Both 1st & 2nd degree show inflammation, edema & swelling
25.
3rd degree shows damage to skin and blood filled
blister formation
Correct Answer - A:D:E
Ans. is 'a' i.e., Occurs at temperature below freezing point,
'd' i.e., Both 1st & 2nd degree show inflammation, edema &

swelling & 'e' i.e., 3rd degree shows damage to skin and
blood filled blister formation [Ref Park 23rd/e p. 748; wiki]
Frostbite is an integer that is caused by exposure of our body to
below freezing point. The underlying mechanism involves injury
from ice crystals and blood clots in small blood vessels following
thawing.
Areas that are usually affected include cheeks, ears, nose and
fingers and toes. There is no difference in the severity among
these areas.
In first degree The skin is numb, usually becomes white and possibly
swollen, with a reddened border. Sometimes the skin is red, may
feel hard or stiff., If it is treated quickly, the skin usually recovers
fully.
In second degree There is also usually quite a lot of swelling of the
affected area., Blisters filled with a clear or milky fluid appear on
the skin., At a late stage, lasting cold sensitivity and numbness can
develop.

Blood filled Blisters also develop. The skin feels hard and cold. In
the weeks after injury, pain persists and a blackened crust
(eschar) develops.the can be long term ulceration and damage to
growth plates.


f)
Primary prevention of hypertension
includes?

f) Weight reduction
g) Dietary salt reduction
h) Exercise promotion
i) Early diagnosis
j) Antihypertensive drugs
Correct Answer - A:B:C
Ans. is 'a' i.e., Weight reduction, 'b' i.e., Dietary salt reduction
& 'c' i.e., Exercise promotion [Ref Park 24th/e p. 393 & 23'd /e
p. 374]

Dietary modification (Nutrition) : Reduction in salt intake to < 5 gm/
day, moderate fat intake, avoidance of alcohol intake, and
restriction of energy intake appropriate to body needs.
Weight reduction and exercise promotion.
Behavioral changes : Reduction of stress, avoidance of smoking,
doing yoga and meditation.
Health education and self care, e.g. measuring own BP.
Secondary prevention includes early case detection by diagnosis
(i.e. identification of hypertension) and treatment.


23. Organism(s) included in category 'A' bio-
terrorism is/ are?
f) Vibrio cholera
g) Clostridium Botulinum
h) Yersinia pestis
i) Bacillus anthracis
j) Burkholderia mallie
Correct Answer - B:C:D
Ans. is 'b' i.e., Clostridium Botulinum & 'c' i.e., Yersinia pestis &
'd' i.e., Bacillus anthracis [Ref Harrison 18thie p. 1 76 9]
Bioterrorism agents can be separated into three categories,
depending on how easily they can be spread and the severity of
illness or death they cause. Category A agents are considered
the highest risk and Category C agents are those that are
considered emerging threats for disease.
Small pox, Anthrax (B. anthracis), Botulism (Clostridium botulinum),
Plague (Yersinia pestis), tularemia (Francisella tularensis) are
categeroy A
V. cholerae, Q. fever (Coxiella burnetii), Typhus fever (Rickettsia
prowazekii), psittacosis (Chlamydia psittaci), glanders (Burkholderia
mallie), Malioidosis (Burkholderia pseudomallie) are category B
1Emerging infections like Nipah, Hantavirus, SARS coronavirus
category C


113. Breast milk contains?
4. Fat
5. Protein
6. Vitamine A
7. Vitamine C
8. Vitamine K
Correct Answer - A:B:C:D
Ans. is 'a' i.e., Fat, 'b' i.e., Protein, 'c' i.e.,Vitamine A
3. 'd' i.e.,Vitamine C [Ref Park 24thie p. 574 & 23"1/e p. 630]
Breast milk is rich in polyunsaturated fatty acids, necessary for the
myelination of the nervous system and brain growth.
Active lipase in the breast milk promotes digestion of fats and
provides FFA.
Iron of breast milk is very well absorbed, breastfeeding prevents
against iron deficiency anemia.
Breast milk also prevents deficiencies of vitamin A, C, D, E and zinc.
Most of the protein is whey proteins (lactalbumin and
lactoglobulin), which can be digested easily (In contrast cow milk
contains more casein).
Breast milk contains the ideal ratio of the amino acids cystine,
taurine and methionine to support development of central
and peripheral nervous system.

Exclusive breast feeding may cause deficiency of vitamin B12
(if mother is pure vegetarian), vitamin K, Vitamin D and fluoride.
Vitamin K deficiency can cause hemorrhagic disease of new
born


There may be neonatal jaundice and golden color stool.


114. True regarding Mondini's disease is/are?
24.
Inner ear malformation
25.
Predisposes to recurrent meningitis in children
26.
Mostly occurs as an isolated entity
27.
May be associated with Penderd and DiGeorge syndromes
28.
Cochlear implants are used in treatment
Correct Answer - A:B:C:D:E
Answer- A, B, C, D, E
It is an abnormality of the inner ear that is associated with
sensorineural hearing loss.
Mondini dysplasia can also predispose to recurrent meningitis.
Mondini dysplasia usually occurs sporadically as an isolated
abnormality but it can be associated with a variety of syndromes
including Klippel Feil syndrome, Pendred syndrome, DiGeorge
syndrome, Wildervanck syndrome, Fountain syndrome,
Johanson-Blizzard syndrome.
Treatment includes surgery to repair the defect to prevent recurrent
meningitis, prophylactic antimicrobial therapy and conjugate -
pneumococcal vaccination, hearing amplification aids & cochlear
implants.


f)
True regarding rhinitis medica mentosa
is/are?

25.
Caused by 132 - agonist sprays
26.
Caused by corticosteroids spray
27.
Spray itself can cause terbinate hypertrophy
28.
There is inferior terbinate hypertrophy
29.
Red mucosa is characteristic
Correct Answer - D
Answer- D. There is inferior terbinate hypertrophy
It is a condition characterized by nasal congestion that is triggered
by the extended use of topical decongestants and certain oral
medications that constrict blood vessels in the lining of the nose:
recreational use of intranasal cocaine malt also cause a similar
condition.
The classic Presentation is that the nasal mucous membranes
appear "beefy-red," inflamed, and may show areas of
punctate bleeding and scant mucus.
The swelling of the nasal Passages caused by rebound congestion
may eventually result in permanent turbinate hypertrophy.
ToPical intranasal corticosteroids aPPear to have beneft in rhinitis
medicamentosa to control the inflammation caused by chronic
vasoconstrictor use.


f)
True regarding Cald-Well-Luc surgery
is/are?

f) Approach for maxillary antrum
g) Antrostomy through inferior meatus
h) Sublabial Approach leading to opening of mandibular antrum
i) Opening of maxillary antrum through gingivolabial approach
j) Opening the maxillary antrum through canine fossa
Correct Answer - A:B:D:E
Answer- A, B, D, E
Caldwell-Luc operation is a process of opening the maxillary antrum
(maxillary sinus through canine fossa by sublabial approach and
dealing with the pathology inside the antrum.
Antrum is reached through an incision in gingivolabial sulcus.
During the surgery a noso-antral window is made (antrostomy)
through the inferior meatus.


117. True regarding otosclerosis is/are?
27.
More common in men
28.
Involves stapes/oval window
29.
Flamingo pink hue behind ear drum
30.
Carhart's notch is at 4000 Hz
31.
Hears better in noisy environment
Correct Answer - B:C:E
Answer- B, C, E
In otosclerosis-50% cases have positive family history.
Females are affected more than males.
Bilateral conductive deafness seen in otosclerosis is not irreversible
as it can be successfully treated by stapedectomy / Stapedotomy.
Sensorineural hearing loss occurs when later in the course of time
osteosclerotic focus reaches the cochlear endosteum but actually
most common hearing loss seen is conductive type.
Carharts notch is seen in bone conduction curve at 2000 Hz.


f)
Clinical features of zygomatic fracture
include?

f) Hypoesthesia along the distribution of infra-orbital nerve
g) Flat face
h) Malocclusion of teeth
i) Flattening of malar prominences
j) Step deformity
Correct Answer - A:D:E
Answer- A, D, E
Clinical features of zygoma fracture
Considerable swelling over zygomatic arch is common and makes
clinical diagnosis more difficult.
Flattening of malar prominence.
Step-deformity of infraorbital margin.
Anaesthesia in the distribution of infraorbital nerve.
Trismus.
The cheek may appear flattened; compared symmetry with the
opposite side.


.
True regarding the use of head mirror
is/are?

. Applied on right eye
. Focal point should be within 6 inches
. One eye should be closed while examining
. Both eye should be open while focusing
. All of the above
Correct Answer - A
Answer- A. Applied on right eye
Two types of illumination is used in
otolaryngologcial examination:

. Semi mobile illumination like the Bull's lamp
29. Mobile illumination like the Clair's head light, or cold light
based head bands.
Bull's lamp: is a semi mobile source of illumination.
The approximate focal length of the mirror is about l0 inches.
The mirror is fured over the right eye in such a way part of the mirror
touches the nose.
The minor is adiusted while keeping the left eye closed and the right
eye is kept open to focus. Then both eyes are opened while
examining.


120. Tone decay test is used for
f) Meinneirs disease
g) Otosclerosis
h) Cochlear deafness
i) Sensory neural deafness
j) Middle ear perforation
Correct Answer - D
Answer- D. Sensory neural deafness
Threshold tone decay test is used for retrocochlear type of SNHL.


121. Causes of hypopyon include?
30.
Retinitis pigmentosa
31.
Fungal keratitis
32.
Episcleritis
33.
Bacterial keartitis
34.
Multiple sclerosis
Correct Answer - B:D:E
Answer- B, D, E
Hypopyon refers to accumulation of polymorphonuclear leucocytes
in the lower angle of anterior chamber. It is usually accompanied
by redness of the conjunctiva and the underlying episclera.


122. Superior rectus palsy causes?
f) Hypotropia to same side
g) Head tilt to opposite side
h) Blapheroptosis
i) Diplopia while looking on same side
j) Hypertropia on opposite side
Correct Answer - A:B:D
Answer- A, B, D
Muscle paralysed- Superior rectus palsy
Deviation of eye- Infero- medially (left) (Hypotropia)
Maximum diplopia while looking- left (temporal) and superior to left
eye to the left
Head position- Tilted to the right and turned


123. True regarding tarsal plate is/are?
f) Acts a skeleton for eyelids
g) Wider in upper eyelids
h) Have Mebomian glands
i) Attached to lateral palpebral superiosis ligaments
j) Inferior plate is semilunar in shape
Correct Answer - A:B:C:D
Answer- A, B, C, D
The tarsi (tarsal plates) are located directly above the lid margins.
The medial and lateral ends of the tarsi are attached to the orbital
rims by the medial and lateral palpebral ligaments.
The superior tarsus is larger and wider.
The lower border of the superior tarsus forms the posterior lid
margin
The inferior tarsus is elliptical in form.
They may contain Meibormian glands and eyelash follicles.


32. In Field defects seen in pituitary
adenoma is
f) Bitemporal hemianopia
g) Binasal hemianopia
h) Quadronopian
i) Pie in the sky
j) Amaurosis in one eye & temporal heminopia in other eye
Correct Answer - A:B:C:D
Answer- A, B, C, D
Central (sagittal) chiasmatic iesions Bitemporal hemianopia
Lateral chiasmatic lesions Binasal hemianopia
Lesion to chiasma produces upper temporal quandarantic visual
field defect (upper temporal quandarantic hemianopia)
Initially pituiatry lesions causes visual field in one of upper quadrant,
it is also called 'pie in the sky' .


.
TRUE statement regarding infective
endocarditis is/ are?

. Jahnway lesions are blanchable hemorrhages on plams & soles
. Jahnway lesions are tender
. Jahnway lesions are non- tender
. Osler's nodes are palpable nodules on the pulp of fingers
& toes
. Osler's nodes are non-tender
Correct Answer - A:C:D
Answer- A, C, D, Jahnway lesions are blanchable
hemorrhages on plams & soles, Jahnway lesions are non-
tender, Osler's nodes are palpable nodules on the pulp of
fingers & toes
Patients with IE presents with features occuring
due to microembolization of the cardiac vegetation to the various
small vessels-
Osler's nodes
Painful tender erythematous nodules.
Seen in the skin of extremities usually in the pulp of the finger &
sometimes toes.
Janeway lesions-
Small flat, red spots, irregular in outline
Non tender
Seen in palms and soles.
They are hemorrhagic and blanch on pressure


126. Causes of pulsus pardoxusus include?
33.
Constrictive pericarditis
34.
Cardiac tamponade
35.
Pulmonary embolism
36.
Restrictive pericarditis
37.
Emphysema
Correct Answer - A:B:C
Answer- A, B, C, Constrictive pericarditis,
Cardiac tamponade, Pulmonary embolism

In normal individual the systolic B.P. decreases by l0mm Hg during
inspiration.
Causes of Pulsus paradoxus:
. Cardiac tamponade
f) Chronic constrictive Pericarditis
g) Emphysema
h) Pulmonary embolism


.
Hypercalcemia with normal or increased
with paratharmone is associated with?

f) Primary hyperparathyroidism
g) Vitamin D intoxication
h) Thiazide diuretics
i) Milk alkali syndrome
j) Familial hypercalciuric hypercalcemia
Correct Answer - A:E
Answer- A, E, Primary hyperparathyroidism,
Familial hypercalciuric hypercalcemia

90% cases of hypercalcemia are caused by malignancy or
hyperparathyroidism.
Medications and familial hypocalciuric hypercalcemia.
Familial cases of high PTH levels
Neonatal severe hyperparathyroidism
Primary hyperparathyroidism
Secondary & tertiary hyperparathyroidism


.
Which of the following is TRUE regarding
second heart sound?

35.
Wide split in complete RBB
36.
Splitting increased in inspiration & decreased in expiration
37.
Spilitting decreased in inspiration & increased in expiration
38.
Wide spilt in complete LBB
39.
Best heard at Erb's point
Correct Answer - A:B:E
Answer- A, B, E, Wide split in complete RBB, Splitting
increased in inspiration & decreased in expiration, Best
heard at Erb's point

The changes in the intrathoracic pressures during breathing are
transmitted to the heart and great vessels.
The increased amount of blood flow through the pulmonary valve
produces delay in the closure of pulmonary valve.
Prolonged P2 and Early A2 resulting in splitting of 2nd heart sound.
During inspiration (A2 and P2 are separated by more than 30 s)
During expiration the splitting disappears.
Erb's Point refers to the third intercostal space on the left sternal
border where both components of S2 (A2 and P2) can be well
appreciated.
f) Delayed electrical activation of the right
ventricle- Complete RBBB (proximal type)
g) Prolonged left ventricular mechanical systole
Complete LBBB (peripheral type)


36. True statement regarding Emery-
Dreifuss muscular dystrophy is/are?
f) X-linked
g) Sudden death
h) Conduction defects
i) Cardiac involvement is rare
j) Contractures
Correct Answer - A:B:C:E
Answer- A, B, C, E, X-linked, Sudden death,
Conduction defects, Contractures

Emery-Dreifuss Muscular Dystrophy (EDMD) is a rare genetic
degenerative disease affecting skeletal muscle and the heart.
EDMD can be subdivided into 3
categories-. X-linked EDMD
37. Autosomal dominant
38. Autosomal recessive
Clinical features-
Triad of symptoms strongly suggests EDMD-
39. Slowly progressive muscle weakness and wasting
in a scapulohumero- peroneal distribution
40. Early contractures of the elbow, ankle, and posterior neck
41. Cardiac conduction defects, cardiomyopathy
Onset is usually in the teenage years
Cardiac disease-
Cardiac disease may present with sudden cardiac death.


f)
Which of the following are correct about
cardiac arrest management according
2015 American Heart Association
guidelines for Cardiopulmonary
resuscitation (CPR) & Emergency
Cardiovascular Care [ECG]?

38.
Ventricular fibrillation requires synchronized cardioversion
39.
Monophasic defibrillators are preferred over biphasic devices
40.
Epinephrine (1 mg) is the DOC for cardiac arrest
41.
Lidocaine may be considered as an alternative to
amiodarone for unresponsive VF/pVT
42.
Vasopressin provides added advantage when
combined with epinephrine
Correct Answer - C:D
Answer- C, D, Epinephrine (1 mg) is the DOC for
cardiac arrest, Lidocaine may be considered as an
alternative to amiodarone for unresponsive VF/pVT

Defibrillation is used to treat certain types of arrhythmias (ventricular
fibrillation and pulseless ventricular tachycardia) while
synchronized cardioversion is used to treat others i.e. unstable
narrow and wide complex tachyarrhythmias such as atrial
fibrillation. atrial flutter and ventricular tachycardia.)


f)
Endocrinal causes of weight gain
include?

f) Thyrotoxicosis
g) Hypothyroidism
h) Addison disease
i) Cushing syndrome
j) Pheochromocytoma
Correct Answer - B:D
Answer- B, D, Hypothyroidism, Cushing syndrome
Cushing's syndrome
Hypothyroidism
Insulinoma
Craniopharyngioma


132. Hyperprolactinemia is associated with?
f) Pituitary adenoma
g) Hyperinsulinemia
h) Pregnancy
i) Lactation
j) All of the above
Correct Answer - E
Answer- E. All of the above
f) Physiologic
Pregnancy
Lactation
g) Hypothalamic-pituitary stalk
damage Craniopharyngioma
Suprasellar pituitary mass
Empty sella
Granulomas
Rathke's cyst
h) Pituitary hypersection
f) Systemic disorder
Chronic renal failure
Hypothyroidism
Cirrhosis
g) Drug induced
Dopamine receptor blockers
Opiates
H2 antagonists - Cimetidine, ranitidine




133. Causes of chronic empyma include?
f) Drainage of pleural effusion
g) Inadequate antibiotic treatment for acute empyma
h) Inadequate needle aspiration of acute empyma
i) Vigorous chest physiotherapy
j) Ruptured subphrenic abscess
Correct Answer - A:B:C:E
Answer- A, B, C, E, Drainage of pleural effusion, Inadequate
antibiotic treatment for acute empyma, Inadequate needle
aspiration of acute empyma, Ruptured subphrenic abscess

f) Parapneumonic empyema- Streptococcus pneumonia is the
major pathogen.
f) Antecedent conditions such as malnutrition, measles or infection
with antibiotic-resistant organisms may increase the risk of
severe pneumonia accompanied by empyema.
. Trauma is another important cause of empyema thoracis
f) CET is therefore an important sequela of untreated or poorly
treated empyema thoracis (treatment includes antibiotics).
f) Empyema may include spread from a ruptured subphrenic abscess,
cardiothoracic surgery sharp force trauma to the chest, esophageal
rupture and following drainage of a pleaural effusion (thoracentesis).
f) In the treatment of acute empyema, the chronic condition is often
due to an impertecily drained sinus with continuance of the
original empyema cavity.


.
A patient presented with recurrent
hemoptysis. Vessel which should be
evaluated for angiography include?

. Pulmonary artery
. Pulmonary vein
. Branchial artery
. Branchial vein
. Superior vena cava
Correct Answer - A:C
Answer- A, C, Pulmonary artery, Branchial artery
Hemoptysis is defined as massive (massive hemoptysis) when blood
loss is more than 400-600 ml/day.
The lung has two blood circulations-
A. System vessels (Bronchial vessels)
B. Pulmonary vessels
Most often hemoptysis originates from a bronchial artery source,
with only 70% of cases arising from the pulmonary artery.


49. Community acquired native valve
infective endocarditis is caused by?
f) Streptococcus viridians
g) Staphylocoocus aureus
h) Enterococcus
i) Candida
j) Pseudomonas aerogenosa
Correct Answer - A:B:C
Answer- A, B, C, Streptococcus viridians,
Staphylocoocus aureus, Enterococcus

Staphylococcus aureus followed by Streptococci of the viridans
group and coagulase negative Staphylococci are the three
most common organisms responsible for infective endocarditis.
Staphylococcus aureus is the most common overall cause of
infective endocarditis and is also the most common cause in
native valve and intravenous drus users.
Coagulase-negative staphylococci (staphylococcus aureus) is the
most common cause in prosthetic valve endocarditis.
Streptococcus viridians are the most frequently isolated
microorganisms when the infection is acquired in a
community setting.
Streptococcus mutans is the most common cause of endocarditis
after dental procedure.
Bacterias causing infective endocarditis are Pseudomonas species (
i/v drug abuser), S. bovis, Clostridium septicum and HACEK
organisms.

Candida albicans is associated with endocarditis in IV drug users,
patients with prosthetic valves and immunocompromised patients.


f)
Diagnostic criteria for Gullian Barre
syndrome includes?

. Areflexia
. Progressive weakness of at least 2 limbs
. Exclusion of vasculitis
. Albumin-cytological dissociation
. Presence of fever
Correct Answer - A:B:C:D
Answer- A, B, C, D, Areflexia, Progressive weakness of at
least 2 limbs, Exclusion of vasculitis, Albumin-cytological
dissociation

Diagnostic criteria for Guillain Barre
syndrome-Required-

. Progressive weakness of 2 or more limbs due to neuropathy.
. Areflexia
. Exclusion of other causes [e.g. vasculitis]
Supportive-
Mild sensory involvement
Facial nerve or other cranial nerve involvement
Typical C.S.F profile (albumino- cytological dissocation)
Absence of fever


.
True statement about diabetic
ketoacidosis is/are?

. Ph < 7.3
. Ketonemia
. Absent urinary ketone bodies
. Glucose level > 300mg/dl
. Bicarbonate < 15 meq/1
Correct Answer - A:B:D:E
Answer- A, B, D, E, Ph < 7.3, Ketonemia, Glucose level
> 300mg/dl, Bicarbonate < 15 meq/1

Ketoacidosis is rare in type II diabetes where insulin levels although
functionally inadequate are still sufficient to prevent ketone body
formation.
Arterial pH is 7.25 - 7.35, 7.0 -7.24 & < 7.0 in mild, moderate &
severe DKA.
Diabetic ketoacidosis is characterized
by-. Hyperglycemia,
51. Ketosis (ketonemia) and ketonuria
52. Acidosis
Ketones are an early indicator of diabetic ketoacidosis and should
be measured in individlual with type I diabetes mellitus.
When the plasma glucose is consistently >16.7 mmol/L (300 mg/dl).
Hyperketonemia and acidosis-
Hormone sensitive lipase is inhibited by insulin and activated by
counter regulatory hormones.
The serum bicarbonate level in D.K.A. is typically decreased to less


than 15meq/l.


f)
In a case of stroke which of the following
heart condition can be suspected?

52.
Mitral stenosis
53.
Aortic regurgitation
54.
Patent foramen ovale
55.
Recurrent atrial arrhythmias
56.
Heart failure
Correct Answer - A:C:D:E
Answer- A, C, D, E, Mitral stenosis, Patent foramen
ovale, Recurrent atrial arrhythmias, Heart failure

The most common cause of embolic strokes are Intra-cardiac
Thrombi.
Male sex
Previous Stroke or Transient Ischemic Attack
High blood pressure
Heart disease - myocardial infarction (heart attack), mitral stenosis,
heart failure
Cardiac arrhythmias - especially atrial fibrillation, ventricular
tachycardia and ventricular fibrillation.
Smoking
Diabetes
High blood cholesterol levels
Sickle cell disease
Oral contraceptives
Excessive alcohol intake


139. Components of Cushing's triad include?
f) Bradycardia
g) Tachycardia
h) Wide pulse pressue
i) Hypotension
j) Irregular breathing
Correct Answer - A:C:E
Answer- A, C, E, Bradycardia, Wide pulse pressue,
Irregular breathing

f) Hypertension
g) Bradycardia
h) Irregular breathing


54. Features of variant Crutzfeldts-Jakob
disease are -
f) Viral disease
g) Prion disease
h) Occur due to consumption of beef from cattle with
bovine spongiform encephalopathy
i) Sporadic form is the most common
j) Prognosis is good
Correct Answer - B:C:D
Answer- B, C, D, Prion disease, Occur due to consumption
of beef from cattle with bovine spongiform encephalopathy,
Sporadic form is the most common

It is a rare fatal degenerative disease of central nervous system that
is caused by infectious protein called prion.
Prion are only known infectious pathogens.
There are four forms of CJD. They are sporadic (80-85%), familial
inherited (15%); iatrogenic/acquired (1%), new variant (can be
acquired by eating beef meat from cattle afected by a disease
similar to CID called bovine spongiform encephalopathy (BSE) or
commonly "mad cow" disease).
Clinical features-
Most of the Patient of CJD presents as dementia and myoclonus.


.
The marker used for determining efficacy
of hepatitis B vaccination is?

. HBsAg
. IgM anti HBc Ag
. IgG anti HBc Ag
. Anti HBs Ag
. Anti HBe Ag
Correct Answer - D
Answer- D. Anti HBs Ag
"Positive Anti HBsAg determines the efficacy of hepatitis B
vaccination.


.
Which of the following favours diagnosis
of chronic renal failure rather than acute
renal failure -

55.
Anemia
56.
Peripheral neuropathy
57.
Small kidney
58.
Renal osteodystrophy
59.
Daily increase in creatinine
Correct Answer - A:B:C:D
Answer- A, B, C, D, Anemia, Peripheral neuropathy,
Small kidney, Renal osteodystrophy

Renal sonogram showing small kidneys- Usually CKD
Oliguria, daily increases in serum creatinine and BUN- Probably
ARF or ARF superimposed on CKD
Severe anemina renal osteotdystrophy (hyperphosphatemia,
hypocalcemia)- Possibly CKD but may be ARF Peripheral
neuropathy


143. Feature of hypomagnesemia -
f) Seizures
g) Athetoid movements
h) Tremors
i) Bradycardia
j) Improvement with Ca supplements
Correct Answer - A:B:C
Answer- A, B, C, Seizures, Athetoid movements,
Tremors Clinical features-

. Hypocalcemia & hypokalemia like tiredness, generalized
weakness, muscle cramps.
56. Cardiovascular:- Arrhythmias, hypertension, tachycardia &
cardiac arrest including torsade de pointes.
f) Neuromuscular and CNS :-increased irritability of the nervous
system with tremors, parasthesias, system, spasticity.
. hypomagnesemia are athetosis, jerking, nystagmus, and an
extensor plantar reflex, confusion, disorientation, hallucination,
& depression.
57. Severe hypomagnesemia may cause generalized tonic-
clonic seizures.
Hypocalcemia can be worsened by isolated treatment of
hypomagnaemia with intravenous magnesium sulfate
because sulfate binds ionized calcium.


f)
Features of parkinsonism include all
except -

58.
Intention tremors
59.
Flaccidity
60.
Mask face
61.
Rigidity
62.
Resting tremors
Correct Answer - A:B
Answer- A, B, Intention tremors, Flaccidity
Four cardinal features of PD that can be grouped under the acronym
TRAP-
Tremor at rest, Rigidity, Akinesia (or bradykinesia) and Postural
instability.


f)
Which of the following can occur in
COPD -

. Hypoxemia
. Hypercarbia
. Decreased gas exchange in terminal bronchioles
. Acidosis
. Hypocarbia
Correct Answer - A:B:C:D
Answer- A, B, C, D, Hypoxemia, Hypercarbia, Decreased
gas exchange in terminal bronchioles, Acidosis

The most common symptoms of COPD are sputum production,
shortness of breath, and a productive cough.
Emphysema is characterired by destruction of gas-exchanging air
spaces i.e. the respiratory bronchioles, alveolar ducts and alveoli.
Low oxygen levels (hypoxia) then high carbon dioxide level in the
blood (hypercapnia /hypercarbia)
There is a development of respiratory acidosis alo called
hpyercapnic acidosis.


146. True regarding porphyria is/are?
59.
Hydroxymethylbilane synthase(HMBS) deficiency causes
acute intermittent porphyria
60.
Photosensitivity is common in acute intermittent pophyria
61.
Erythropoeitic porphyria is caused by
uroporphyrinogen decarboxylase (UROD) deficiency
62.
Porphyria cutanea tarda is characterized by
vesicubullous lesions
63.
Erytheropeitic porphyria shows strong photosensitivity
Correct Answer - A:D:E
Answer- A,Hydroxymethylbilane synthase(HMBS) deficiency
causes acute intermittent porphyria D,Porphyria cutanea
tarda is characterized by vesicubullous lesions
E,Erytheropeitic porphyria shows strong photosensitivity

The following table gives summary of the major findings
of porphyrias

Type and
Enzyme involved Major symptoms
class
Hepatic
porphyrias:
Acute
Uroporphyrinogen I Abdominal pain
intermittent
synthase
Neuropsychateic.
porphyria
Uroporphyrinogen Photosensitivity Abdominal pain
porphyria
decarboxylase
Neuropsychateic
cutanea tarda Coproporphyrin
Photosensitivity
Hereditary
oxidase
Abdominal pain
Protoporphyrinogen


copro-porphyria
Neuropsychateic Photosensitivity
oxidase Variegate
porphyria
Erythrohepatic
porphyrias:
Uroporphyrinogen
Congenital
Photosensitivity
f) synthase
erythrohepati
c porphyrias
Erythrohepatic
porphyrias:
Photosensitivity
Ferrochelatase
Protoporphyria


60. TRUE statement regarding scleroderma
is/are?
f) Localized disease more commonly involves face
and extremities
g) There is progressive pulmonary fibrosis in most cases
h) Pulmonary arterial disease without fibrosis can also
cause pulmonary arterial hypertension
i) ACE inhibitors can be used in renal hypertension
j) All of the above
Correct Answer - E
Answer- E. All of the above
Systemic sclerosis characterized by abnormal accumulation of
fibrous tissue in skin and multiple organs.
The skin is most commonly affected, but the GIT, kidney, heart,
muscles and lungs also are involved.
The disease is divided into two
categories-l. Diffuse scleroderma-

There is rapid progression with early visceral involvement.
2. Limited (localized) scleroderma (morphea)-
Skin involvement is confned to fingers, forearm and face.
Pulmonary fibrosis- Frequent, early and severe
Pulmonary arterial hypertension- often in association with pulmonary
fibrosis.
Treatment-
Among patients with SRC, "ACE inhibitors" are recommended rather
than other antihypertensive agents.




.
A tuberculosis patient was sputum
positive even 5 months after the
treatment with rifampicin, isoniazid,
moxifloxacin & amikacin. Diagnosis is?

. MDR TB
. XDR TB
. Polydrug resistance TB
. Monoresistance
. Rifampicin resistance
Correct Answer - B:C
Answer- B,XDR TB C,Polydrug resistance TB
Polydrug resistance: Resistance to more than one first-line anti-TB
drug (other than INH and rifampicin).
Multi drug resistance TB (MDR TB) is referred to resistance to
"isoniazid and rifampicin".
Rifampicin-resistance TB (RR-TB) : It includes any resistance to
rifampicin. whether non resistance, multidrug resistance,
polydrug resistance or extensive drug resistance.


61. WHO conditioned guidelines for
treatment of MDR TB 2016, includes 4
core drugs and add on drugs. Add on
drugs are?

f) Bedaquiline
g) Linzolide
h) Delamnaid
i) Capreomycin
j) Moxifloxacin
Correct Answer - A:C
Answer- A,Bedaquiline C,Delamnaid
Group A=levofloxacin, moxilloxacin. gatifloxacin
Group B=amikacin, capreomycin, kanamycin, (streptomycin);
Group C= ethionamide (or prothionamide), cycloserine (or
terizidone), lineznlid, clofazimine;
Group D2=bedaquiline, delamanid


2)
A patient presented with haematuria with
acute renal failure. On doing renal
biopsy, it showed cresentric
glomerulonephritis. Immunofluoresence
findings showed C3 & IgG deposition.
Most likely diagnosis among the
following is?

3. Membranous glomerulonephritis
4. Minimal change disease
5. Monoclonal deposition disease
6. Acute post-infectious glomerulonephritis
7. Focal segmental glomerulosclerosis
Correct Answer - D
Answer- D. Acute post-infectious glomerulonephritis
PSGN appears 1 to 4 weeks after infection of pharynx or skin by
specific nephritogenic strains (12, 4 and 1) of group A beta
hemolytic streptococci.
The lesions are caused by Type III hypersensitivity reaction with
immune complex deposition and complement activation, causing
decreased complement level.
Clinical features-
Hematuria
Oliguria, Non-selective proteinuria
PSGN causes acute renal failure.

By immunofluorescence microscopy, there are irregular granular
deposits of IgG. IgM and C3 in the mesangium and along the
basement membrane (starry sky appearance).


2)
In a patient with ileal resection, ileoc-
coecal vales are spared. Which of the
following can develop?

f) Bacterial overgrowth
g) Malabsorption
h) Steatorrhea
i) Cholelithiasis
j) Renal oxalate stones
Correct Answer - B:C:D:E
Answer- B, C, D, E
Patients with short-bowel syndrome invariably present with a history
of several intestinal resections.
Malabsorption, diarrhea (with or without steatorrhea) is an almost
constant clinical findings.
Terminal ileum resection- watery diarrhea/ steatorrhea,
malabsorbtion, megaloblastic anemia, oxalate kidney
stones, cholesterol gall stones.


f)
Alcohol is a risk factor for which of the
following carcinoma(s)?

. Esophagus
. Liver
. Pancreas
. Cervix
. Larynx
Correct Answer - A:B:C:D:E
Answer- A, B, C, D, E
Alcohol most strongly increased the risk for cancers of the oral
cayity, pharynx, esophagus and larynx.
Significant increases in risk also existed for cancers of the stomach,
colon, rectum, liver, female breast, and ovaries.
Alcoholic women are at high risk for in situ and invasive cervical
cancer and for cancer of the vagina.


64. True regarding esophageal squamous
cell carcinoma is/ are?
f) Barrett's esophagus is a risk factor
g) Common in middle third of esophagus
h) Stomach, jejunum or colon can be used for replacement
after surgical removal
i) Chemoradiation has little role in inoperable patients
j) Staging is done by CECT
Correct Answer - B:C:E
Answer- B, C, E
"The stomach remains the most Preferred esophageal substitute
following esophagectomy for cancer.
Squamous cell carcinoma-
It is the most common type of esophageal carcinoma worldwide and
in India. It usually occurs in middle 1/3rd (most
common) and upper 1/3rd of esophagus. Some may also arise in
lower 1/3rd.
Important risk factors-
Plummer -Vinson- Paterson Kelly syndrome
Tylosis plamaris et plantaris
Human papilloma virus (HPV) infection
Flexible endoscopy with biopsy is the primary method for diagnosis
of esophageal cancer.
CECT chest and abdomen and Positron emisison tomography (PET)
are one for staging of esophageal cancer.
Stomach, jejunum or colon can be used for replacement after


surgical removal


154. True regarding acute sialedinitis is/are?
f) Most common in submandibular glands
g) Most common type is viral
h) Can present with stasis of saliva
i) There may be tender pre-auricular nodes
j) Stone removal may be done by probing through oral route
Correct Answer - B:C:E
Answer- B, C, E, Most common type is viral, Can present
with stasis of saliva, Stone removal may be done by probing
through oral route

Sialadenitis is most common in the parotid gland.
The most common cause of acute inflammation of the salivary
glands is mumps virus.
Sialadenitis due to bacterial infections is most commonly caused by
Staphylococcus aureus.
Stone or a kink in the salivary duct can also diminish saliva
flow/stasis of saliva.
Surgical management-
Abscesses require drainage
Gland excision incases of recurrent acute sialadenitis.


.
True regarding chronic pancreatitis
is/are?

. Can present with steatorrhea and malabsorption
. Presents with mid epigastric pain radiating to back
. Markedly raised level of amylase & lipase
. Predisposes to carcinoma
. Complete pancreactomy relieves pain in majority of patients
Correct Answer - A:B:D:E
Answer- A,Can present with steatorrhea and malabsorption
B,Presents with mid epigastric pain radiating to back
D,Predisposes to carcinoma E,Complete pancreactomy relieves
pain in majority of patients
Etiology
Alcoholic pancreatitis - most common
Hyperparathyroidism
Hypertriglyceridemia
Autoimmune
Clinical features-
Abdominal pain is the most common presenting symptom.
The patient experiences intermittent attacks of severe pain, often in
the mid-abdomen or left upper abdomen and occasionally raditing
in a bandlike fashion or locatized to the midback.
Chronic pancreatitis include maldigestion, malabsorption, diarrhea,
bloating and weight loss. This may be due either to fear of eating or
due to pancreatic exocrine insufficieny and steatorrhea &
azotorrhea (protein malabsorption).

Serum enzymes-
Serum amylase and lipase levels are normal or slightly elevated in
chronic pancreatitis.
In later stage of chronic pancreatitis, atrophy of the pancreatitic
parenchyma results in normal serum enzyme levels because
of fibrosis of the pancrease.
Treatment-
i) Pancreaticduct drainage
In patients with a dilated pancreatic duct, a Roux-en-Y side-to-side
pancreatico jejunostomy is indicated.
ii) Pancreatic resection-
If the disease is limited to the head of the pancreas, a Whipple
operation (pancreaticoduodenectomy) can produce good
results.


156. q-SOFA score includes?
. Pulse rate
. Respiratory rate
. Systolic blood pressure
. Altered mentation
. Mean arterial pressure
Correct Answer - B:C:D
Answer- B,Respiratory rate C,Systolic
blood pressure D,Altered mentation

It is a validated ICU mortality prediction score, to help identify
patients with suspected infection that are at high risk for poor
outcome (defined as in-hospital mortality, or ICU length of stay
>3 days) outside of the ICU.
The qSOFA simplifies the SOFA score drastically by only including
its 3 clinical criteria.
It is used in patients > l8 years old in a non-ICU setting (pre-hospital,
ward, emergency department, step down unit) with
confirmed or suspected infection.
Low blood pressure - 1
HIGH RESPIRATORY RATE - 1
Altered mentation- 1


.
True regarding leg ulcers & their location
is/are?

f) Arterial insufficiency - tip of the toes
g) Arterial insufficiency - medial side of leg [above
medial malleolus]
h) Venous insufficiency - above lateral malleolus
i) Diabetic neuropathic ulcer - planter aspect of metatarsal head
j) Pressure ulcer - heel
Correct Answer - A:C:D:E
Answer- A, Arterial insufficiency - tip of the toes C,Venous
insufficiency - above lateral malleolus D, Diabetic
neuropathic ulcer - planter aspect of metatarsal head E,
Pressure ulcer - heel

Venous ulcer-
Located below the knee,most often on the inner part (medial) of the
ankles. Those associated with lasser saphenous vein insufficiency
may occur on outer side (lateral).
Associated with aching, swollen lower legs that feel more
comfortable when elevated.
Arterial ulcer-
Usually found on the feet, heels or toes. Toes are afected most
commonly especially tips.
The borders ofthe ulcer appear as though they have been 'punched
out'.
Frequently painful, particularly at night in bed or when the legs are at
rest and elevated.

Neurotrophic (diabetic) Ulcers-
Neuropathic ulcers usually occur on the plantar aspect of the foot
under the metatarsal heads or on the plantar aspects of the toes.
Pressure ulcer (pressure sore /decubitus ulcer)-
The hip and buttock regions account for up to 70% of all pressure
injuries, with ischial tuberosity, trochanteric, and sacral locations
being most common.
The lower extremities account for an additional l5-25% of all
pressure injuries, with malleolar, heel, patellar, and
pretibial locations being most common.


.
TRUE statement regarding hepatocellular
carcinoma is/are?

67.
Frequently associated with raised AFP
68.
Good prognosis even without resection
69.
Ultrasound guided biopsy is diagnostic
70.
There is extensive vascular invasion
71.
Most cases present with resectable tumor
Correct Answer - A:C:D
Answer- A,Frequently associated with raised AFP
C,Ultrasound guided biopsy is diagnostic D, There is
extensive vascular invasion

It is the most common primary malignant tumor of liver.
Most important risk factor for HCC is HBV infection.
Risk factors are chronic alcoholism, food contaminants
Clinical Features-
Hepatocellular carcinomas have a strong propensity for invasion of
vascular channels. Extensive intrahepatic metastasis occurs and
the tumour occasionally invades the portal yein or inferior
venacava.
Unresected hepatocellular carcinoma has a very poor prognosis.
The fibrolamellar variant of the HCC is associated with a more
favorable prognosis.
Diagnosis-
Liver Biopsy under US or CT guidance- The diagnosis can be
established by percutaneous core biopsy or aspiration biopsy
in most patients if the biogsy site is selected.
MRI is the investigation of choice.

Serological markers
Elevated level of serum alphafetoproteins are seen in 50 - 75%
cases.
Treatment-
OnIy 15-20% of HCC are resectable because of multicentricity,
bilobar involvement, portal vein invasion and lymphatic
metastasis.


f)
TRUE statement regarding pyogenic liver
abscess is/ are?

f) More common on left side of liver
g) Surgical drainage is the treatment of choice
h) Most common organism responsible is E. coli
i) X-rays are diagnostic
j) Diagnosis is confirmed by aspiration and culture
Correct Answer - C:E
Answer- C,Most common organism responsible is E. coli
E, Diagnosis is confirmed by aspiration and culture

It can be multiloculated or a single cavity.
It usually involves the right lobe (-75%)
Clinical features
the classical description of hepatic abscess is - fever, jaundice and
rt. upper quadrant pain; but this is rarely seen (-10% cases)
most common presentation includes fever with chills and
abdominal pain (Fever is MC symptom)
Nonspecific symptoms like malaise and anorexia are also seen
Laboratory investigations
Abnormalities of LFTs are generally seen but are not severe.
Alkaline phosphatase & transminases are mildly elevated
Bilirubin is elevated
Ultrasound and CT are the main diagnostic modalities.
Diagnosis is confirmed by aspiration and culture.
(Serology tests are helpful for diagnosing amoebic abscess not
pyogenic liver abscess.)

Treatment involves antibiotics and percutaneous catheter drainage


69. Poor prognosis of breast carcinoma is
associated with?
f) Over expression of Her-2/neu
g) Increased estrogen & progesterone receptor expression
h) Triple negative tumor
i) < 2% cells in 'S' phase of mitosis
j) > 65% cells expressing Ki-67 marker
Correct Answer - A:C:E
Answer- A,Over expression of Her-2/neu C,Triple
negative tumor E, > 65% cells expressing Ki-67 marker

A diagnosis of triple negative breast cancer means that the three
most common tlpes of receptors i.e. estrogen receptors (ER),
Progesterone receptors (PR), and the hormone epidermal
growth factor receptor 2 (HER-2/neu gene) are not present in
the tumor cells.
This type of cancer is more common in women with BRCA1 gene
mutations.


70. Luminal A breast cancer shows following
feature?
f) Low grade tumor
g) Her2 /neu amplification
h) Good prognosis
i) High grade tumor
j) ER negative
Correct Answer - A:C
Answer- A,Low grade tumor C, Good prognosis
Most common subtype.
Low grade, Slow growing.
Best prognosis of all subtypes
Hormone-receptor (ER and/or PR) positive with high expression.
HER2 negative/ low expression
Low expression of the protein Ki-67


71. True regarding epidural hematoma
is/are?
f) Arterial bleed
g) More dangerous than subdural variety
h) On CT scan it gives biconvex lenticular hyperdense appearance
i) Located on lateral side of hemisphere
j) Common after Injury at pterion
Correct Answer - A:B:C:D:E
Answer- A,Arterial bleed B,More dangerous than subdural
variety C,On CT scan it gives biconvex lenticular hyperdense
appearance D,Located on lateral side of hemisphere
E,Common after Injury at pterion

Epidural hematoma brain injuries (also referred to as extradural
hemorrhages) involve blood pooling between the outer
membrane (the dura) and the skull.
Cause-
Injury to middle meningeal artery.
Accumulation of blood-
b/w skull and dura
Location-
lateral cerebral convexities
Clinical features-
Classicalll lucid interval then coma, but more varialble.
Pupillary dilatation with contralateral then bilateral limb weakness.
Slowly evolving stupor then coma
Radiological features-

Biconvex (lens shaped or lenticular)
In Acute cases Hyperdense (2/3) or mixed density (1/3).
In chronic cases Hypodense
Surgical intervention-
Urgent evacuation


163. In acute abdomen examination signs
which are elicited in supine position
include?

a) Ilio-psoas sign
b) Obturator sign
c) Rovsing sign
d) Carnett sign
e) Balance sign
Correct Answer - A:B:C:D
Answer- A,Ilio-psoas sign B,Obturator sign
C,Rovsing sign D,Carnett sign

Psoas sign( Cope's psoas test or Obraztsova's sign)-
It indicates irritation to the iliopsoas group of hip flexors in the
abdomen.
It is elicited by performing the psoas test by passively extending the
thigh of a patient lying on his side or supine with knees extended. A
positive psoas sign on the right may suggest appendicitis.
Obturator Sign-
The obturator sign is based on the same principle as the psoas sign,
that stretching a pelvic muscle irritated by an inflamed appendix
causes pain.
To stretch the right obturator internus muscle and elicit the sign, the
patient's right hip and knee and then internally rotates the right hip
with a patient lying on his side or supine.
Rovsing's sign-
The patient is said to have a positive Rovsing's sign and may have


appendicitis.
It is done in supine position.
Carnett's sign-
Carnett's sign is a finding on clinical examination in which (acute)
abdominal pain remains unchanged or increases when the
muscles of the abdominal wall are tensed.
A supine patient can be asked to lift the head and shoulders from
the examination table to tense the abdominal muscles.
Ballance sign-
The presence of a dull percussion note in both flanks of a patient
lying in the left decubitus position.


164. FALSE statement regarding urotheilal
bladder tumor is/are?
a) Most common variety
b) Schistostomiasis is not a risk factors
c) Strongly related to smoking
d) Pain is the most common presenting feature
e) Most common site is trigone
Correct Answer - B:D
Answer- B,Schistostomiasis is not a risk factors D,Pain is
the most common presenting feature

There are 3 types of bladder
cancer-. Transitionl cell cancer- (90%)
. Squamous cell carcinoma (5-10%)
. Adenocarcinoma (2%)
Risk factors-
Cigarette smoking - it's the most common etiological factor
Schistostoma haematobium - risk factor for both transitional cell
carcinoma & SCC
Drugs such as Phenacetin & chlorphenazine
Clinical features-
Painless haematuria is the presenting feature in 85-90% of bladder
cancer patients.
Pain (secondary to clot retention, tumor extension into retro
peritoneum or ureteral obstruction or metastasis in bony Pelvis)
may rarely occur.
Site-

Trisone and adjacent postero-lateral wall


165. TRUE regarding hypertrophic gastric
polyp is/are?
a) Most common gastric polyp
b) Pre-malignant
c) Mostly non-Hodgkin's lymphoma
d) More common in young adults
e) Surgery is done if they are symptomatic
Correct Answer - A:E
Answer- A,Most common gastric polyp E,Surgery is done
if they are symptomatic

They are the most common benign tumor of stomach.
Gastric polyps are of following types
- 1) Hyperplastic polyp (75%)

It is the most common gastric polyp. It is non neoplastic. It has no
malignant potential.
Hyperplastic polyps are also referred to as inflammatory polyps.
2) Adenomatous polyp (Adenoma)
Adenoma contains proliferative dysplastic epithelium and thereby
has malignant potential.
Treatment-
Gastric polyps that are symptomatic > 2cm or adenomatous, should
be removed.


166. True statement(s) regarding lymphedema
is/are?
a) Can be complicated by cellulitis
b) Congenital lymphedema is also known as Millroy's disease
c) Commonly caused by Wouchreria bancrofti
d) Lymphoedema congenita more likely to be unilateral
e) Lymphedema precox is more common in males
Correct Answer - A:B:C
Answer- A,Can be complicated by cellulitis B,Congenital
lymphedema is also known as Millroy's disease
C,Commonly caused by Wouchreria bancrofti

Lymphedema is an interstitial edema of lymphatic origin.
Types-
A. Primary lymphedema-
l) Congenital lymphedema-
More likely to be bilateral and involve whole leg.
Familial version of congenital lymphedema is called Milroy's disease.
2) Lymphedema precox-
Most common form of primary lymphedema (90%).
Familial verison is known as Meige's disease.
B. Secondary lymphedema
Common organisms causing lymphatic filariasis including
Wuchereria bancrofti, Brugia malayi, Brugia timori.
The most common manifestation of lymphedema is edema.
Complications-
It can result in chronic eczema, dermatophytosis, onychomycosis,


cellulitis, lymphangitis, lymphadenitis and in severe cases
skin ulcers.


167. Steps taken to prevent post-operative
incised wound infection is/are?
a) Start antibiotics at least 1 day pre-operatively
b) Shaving of hair
c) One dose of antibiotic just before the incision
d) Shower preoperatively using an antiseptic
e) Prevent intraoperative hypothermia
Correct Answer - C:D:E
Answer- C,One dose of antibiotic just before the
incision D,Shower preoperatively using an antiseptic
E,Prevent intraoperative hypothermia

They are defined as infections that occur 30 days after surgery with
no implant, or within 1 year if an implant is placed and infection
appears to be related to surgery.
Most SSIs are caused by endogenous microorganisms present on
the patient's skin when the surgical incision is made.
Gram positive bacteria such as Staphylococcus aureus are the most
common causative skin-dwelling microorganisms.
Exogenous sources of microorganisms include surgical instruments,
operating room surfaces, the air, and personnel.
Hair removal was once theorized to reduce the risk of post-operative
infection is actually associated with increased incidence of SSI.
Prevention strategies-
Preoperative:
. Tobacco cessation at least 30 days prior to elective surgery
. Antimicrobial prophylaxis

Antimicrobial prophylaxis should be adrninistered only for Class I
(clean wound) and II wounds (clean/contaminated wound).
Patients with Class III (contaminated wound) or IV wounds (Dirty
wound) are presumed to be taking antimicrobial therapy already.
c) Preoperative showering with chlorhexidine gluconate 4%
solution the night before surgery.
Appropriate timing of administration:
30 to 6O minutes prior to incision
1 -2 hours for antibiotics with longer periods of infusion, such as
vancomycin.
Perioperative techniques
Prevent intraoperative hypothermia
Treatment-
For superficial SSI this involves opening the wound at skin and
subcutaneous level and clearing the wound.


168. True regarding opsite dressing is/are?
a) Wound can be seen
b) Vapor permeable
c) Impermeable to bacteria
d) Water permeable
e) Increased chances of maceration
Correct Answer - A:B:C
Answer- A,Wound can be seen
B,Vapor permeable C,Impermeable to
bacteria OPSITE post-op dressing

It is a transparent adhesive film. The film is moisture vapor
permeable.
OPSITE helps prevent skin maceration.
OPSITE is resistant to water and boily fluids (waterproof).
It also acts as barrier to bacteria including MRSA.
Allow constant monitoring on the wound and peri-wound area as
wound can be seen.


169. True statement (s) regarding spegalian
hernia is/are?
a) Protrudes through linea alba
b) Occurs at the termination of transverse abdominis muscle
c) Occurs at the lateral edge of rectus abdominis muscle
d) Contents of hernia mostly include small intestine
e) Surgery is the treatment of choice
Correct Answer - C:D:E
Answer- C,Occurs at the lateral edge of rectus
abdominis muscle D,Contents of hernia mostly include
small intestine E,Surgery is the treatment of choice

Spigelian hernias occur secondary to a defect in the transversus
abdominis muscle and rectus sheath aponeurosis.
Spigelian hernia contents most often includes small intestine but can
also include cecum appendix, sigmoid colon or omentum.
Spigelian hernia (or lateral ventral hernia)
A spigelian hernia is a hernia through the spigelian fascia close to
the level of the arcuate line.
Spigelian fascia is the aponeurotic layer between the rectus
abdominis muscle medially and the semilunar line laterally.
Treatment-
Once a hernia appears, surgical treatment is the only way to repair
it.


170. TRUE regarding 2nd degree superficial
burn is/are?
a) Very painful
b) Dry eschar formation
c) No spontaneous healing
d) Blister formation
e) Capillary refilling present
Correct Answer - A:D:E
Answer- A,Very painful D,Blister formation E,Capillary
refilling present

Superficial 2nd degress burn (Superficial partial
thickness burn)

Involves only upper dermal layer
Blister formation occurs
Erythematous
Blanch on touch
Quite painful.
Heal without scarring in 1- 2 weeks


171. Diagnostic criteria for blood stream
infection from central venous catheter
includes all except?

a) Colony count from catheter culture is 5 times more
than peripheral blood culture
b) Blood culture form catheter shows colony formation at least
2 hours before the peripheral blood culture
c) Quantitative catheter tip culture showing >103 CFU/ catheter
segment whereby the same organism [species and
antibiogram] is isolated from the catheter segment and a
peripheral blood sample
d) Apparent source of blood stream infection present
e) At least one positive peripheral blood culture
Correct Answer - D
Answer- D. Apparent source of blood stream infection present
It is defined as the presence of bacteremia originating frorn an
intravenous catheter.
The most comlnon cause of nosocomial bacteremia.
Laboratory diagnosis-
CRBSI means a patient with an intravascular catheter has at least
one positive blood culture obtained from a peripheral
vein, clinical manifestations of infections.
Long-term catheters-
Semi-quantitative growth of 15 cfu/catheter segment of the same
microbe from both the insertion site culture, and the catheter
hub culture strongly suggests that the catheter is the source ofa


bloodstream infection.


172. A patient with history of trauma,
presented with multiple fracture/bilateral
femoral fracture, respiratory distress &
red urine. For evaluation of patient all the
following are included in major criteria of
Gurd's criteria except?

a) Unexplained decrease in platelets
b) Tachycardia
c) Petechiae
d) CNS depression
e) Pulmonary edema
Correct Answer - A:B
Answer- A,Unexplained decrease in platelets
B,Tachycardia Gurd's criterion is for the diagnosis of fat
embolism syndrome. Gurd and Wilson's criteria for FES-
Axillary & subconjunctival petechiae
Hypxemia
Central nervous system depression
Pulmonary edema


173. In classification of contaminated wound,
which of the following are included?
a) Resection of unprepared bowel
b) Perforated appendix resection
c) Resection of intestinal fistula
d) Inguinal hernia repair
e) Hysterectomy
Correct Answer - A:B
Answer- A,Resection of unprepared bowel B,Perforated
appendix resection
Class III (contaminated)-
Open, fresh accidental wounds. ln addition, operations with major
breaks in sterile technique (e.g., open cardiac massage) or gross
spillage from the gastrointestinal tract, and incisions in which acute,
nonpurulent inflammation is encountered.
Examples:Appendectomy (with perforation/ peritonitis) Bowel
Resection (unprepared), bile spillage.


174. True about compression stocking are:
a) Worn even after ulcer heals to prevent recurrence
b) Worn in morning & taken off at night before bedtime
c) Compression occurs maximum at calf
d) Worn only at edema sites
e) Provide calf pump
Correct Answer - A:B:E
Answer- A,Worn even after ulcer heals to prevent
recurrence B,Worn in morning & taken off at night
before bedtime E,Provide calf pump

These are specialized elastic hosiery designed to help prevent the
occurrence of and guard against further progression of, venous
disorders such as edema, phlebitis and thrombosis.
They also aid in the treatment & prevention of ulcers of the lower
legs.
Compression stockings are tightest at the ankles and gradually
become less constrictive toward the knees and thighs. Therefore
the compression level is highest around the ankle and lessens
towards the top of the hose.
Stockings are best applied upon waking before the person has got
out of bed.


175. Ulcerative colitis true statement is/are?
a) No skip lesion
b) Almost always includes rectum
c) Can present with pain & bloody diarrhea
d) More common in smokers
e) Transmural involvement
Correct Answer - A:B:C
Answer- A,No skip lesion B,Almost always includes
rectum C,Can present with pain & bloody diarrhea



176. Indications of hemorrhoidectomy in
hemorrhoids include?
a) Large first & second degree hemorrhoids
b) Third & fourth degree hemorrhoids
c) If not able to differentiate prolapsed hemorrhoids & lower
rectal prolapse
d) Complicated by strangulations
e) Failure of conservative therapy
Correct Answer - B:D:E
Answer- B,Third & fourth degree hemorrhoids D,Complicated
by strangulations E,Failure of conservative therapy Operative
hemorrhoidectomy (excisional hemorrhoidectomy)-

3rd & 4th degree: hemorrhoids
Other degree not cures by conservative methods
Mixed (combined internal/external) hemorrhoids
Hemorrhoids complicated by strangulation


177. Sterile OT zone is?
a) Changing room
b) Scrub room
c) Set up room
d) Cleaner room and stores
e) Anaeasthesia inducing room
Correct Answer - B:C:E
Answer- B,Scrub room C,Set up room E,Anaeasthesia
inducing room

Zone- 3 (Sterile area)
Operating Theatre
Scrub Room
Anesthesia Room
Set up Room


178. True about Phylloides tumor is/are?
a) Associated with BRCA 1
b) FNAC can diagnose reliability
c) Treated with mastectomy
d) Axillary lymph nodes are commonly involved
e) Associated with BRCA 2
Correct Answer - C:E
Answer- C,Treated with mastectomy E,Associated with BRCA
2 It is a rare, predominantly benign tumor that occurs almost
exclusively in the female breast. Phyllodes tumor is the
most commonly occurring nonepithelial neoplasm of the
breast. Large tumor - simple mastectomy is done


179. Metabolic abnormality seen in congenital
hypertrophic pyloric stenosis is?
a) Hypochloremic hypokalemic metabolic alkalosis
b) Hyperchloremic hypokalemic metabolic alkalosis
c) Hypochloremic hypokalemic metabolic acidosis
d) Hyperchloremic hypokalemic metabolic acidosis
e) None
Correct Answer - A
Answer- A. Hypochloremic hypokalemic metabolic alkalosis
Proiectile nonbillotrs vomiting is the initial synptoms of pyloric
stenosis, The vomiting usually starts alter 3 weeks of age, but
symptoms may develop as early as the lst week of life and as late
as the 5 months of life.
Hpyochloremic hypokalemic metabolic alkalosis.


180. Congenital hypertrophic pyloric stenosis
causes
a) Bilious vomiting
b) Non bilious vomiting
c) Projectile vomiting
d) Non projectile vomiting
e) None
Correct Answer - B:C
Answer- B,Non bilious vomiting C. Projectile vomiting
The vomiting may or may not be projectile, however non-bilious
vomiting after feed is the characteristic feature.
In about 10-15% of infants vomitus may contain blood. This bleeding
is due to reflux esophagitis, a common association with
hypertrophic pyloric stenosis.
Also know Associated anomalies occur in 6-20% of pts. This
includes
esophageal
atresia,
Hirshprung's
disease,
anorectal anomalies, and intestinal malrotation.


181. Most common post-operative complaint
is/are?
a) Pain
b) Nausea
c) Vomiting
d) Sedation
e) Shivering
Correct Answer - A:B:C Answer-
A,Pain B,Nausea C,Vomiting

Nausea and vomiting episodes still persist as the most common
complaint.
Post-operative nausea and vomiting (PONV) is a common
complication of surgery and anaesthesia
Soreness in the throat if the patient needs artificial ventilation.
Soreness and swelling around the incision site.


182. False about abdominal artery aneurysm
(AAA)is:
a) Surgery is indicated when AAA > 6 cm
b) 90% of AAA is present below renal artery
c) Mortality rate after surgery is > 25%
d) Commonly causes colon ischemia
e) Most common cause is atherosclerosis
Correct Answer - A:C:D
Answer- A,Surgery is indicated when AAA > 6 cm C,Mortality
rate after surgery is > 25% D,Commonly causes colon ischemia

It is the most common true arterial aneurysm.
Most common site (85-90%) is infrarenal.
Etiology-
Atherosclerosis is the most common cause.
Non-specific aorto-arteritis:- the 2nd most common cause.
The 5 year risk of rupture of aneurysm of < 5 cm is 1 to 2%, where
as it is 20-40% for aneurysm > 5cm.
Investigation
IOC is CECT
Management-
Operative repair of the aneurysm with insertion of prosthetic graft or
aortic stent graft is indicated for-
For asymotomatic aneurysms AAA repair is indicated if size > 5.5
cms.
In elective surgery of AAA the operadve mortality rate approximates
about l-2% whereas after acute rupture the mortality rateof


emergency surgery generally approximate about 45-50%.
Complications-
Death (1.8-5% mortality for elective open repair <1% for
endovascular repair and 50% if the AAA has ruptured)
Colon ischemia is l% for elective repair 15-20% if the AAA has
ruptured


183. Hypoparathyroidism can occur in:
a) After thyroid surgery
b) Digeorge syndrome
c) Radical resection of head & neck cancer
d) MEN I
e) All
Correct Answer - A:B:C
Answer- A,After thyroid surgery B,Digeorge
syndrome C,Radical resection of head & neck
cancer Genetic disorders (eg: DeGeorge syndrome)

. Abnormal parathyroid gland development
. Abnormal PTH synthesis
o Activating mutations of calcium sensing receptor (autosomal
dominant hypocalcemia or sporadic isolated
hypoparathyroidism)
. Post-surgical (thyroidectomn parathyroidectomn radical neck
dissection)
o Autoimmune polyglandular syndrome


184. True regarding 10-0 sutures is/are?
a) Thicker than 1-0 sutures
b) Synthetic sutures
c) Diameter is 0.9 mm
d) Stronger than 1-0
e) All of the above
Correct Answer - B
Answer- B. Synthetic sutures
The larger the size ascribed to the suture, the smaller the diameter
be.
10-0 sutures- 0.2 (0.020-0.029)


185. Effects of hypoxemic ischemia in a
neonate include?
f) Neurological damage
g) Subcutaneous fat necrosis
h) Pulmonary hypertension
i) Hyperglycemia
j) Hlpercalcemia
Correct Answer - A:B:C
Ans. is 'a' i.e., Neurological damage, 'b' i.e. Subcutaneous
fat necrosis & 'c' i.e., Pulmonary hypertension
Clinical features of hypoxic ischemic encephalopathy'
o Encephalopathy progress over time ?
83. Birth to 12 hours Decreased level of consciousness, poor
tone, decreased spontaneous movement, periodic breathing or
apnoea, seizures.
f) 12-24 hours -4 More seizures, Apnoeic spells, jitteriness, weakness.
g) After 24 hours Hypotonia, consciousness, poor
feeding, brainstem signs (oculomotor) and pupillary
disturbances.
Hypotonia is generalized, involves both limbs and trunk and
all muscles simultaneously.


186. Crouzon syndrome consists of :
84.
Maxillary hypoplasia
85.
Syndactyly
86.
Hydrocephaly
87.
Macrocephaly
88.
Mandibular prognathism
Correct Answer - A:C:E
Ans. is 'a' i.e., Maxillary hypoplasia, 'c' i.e. Hydrocephaly & `e'
i.e. Mandibular prognathism
Crouzon syndrome is a genetic disorder characterized by the
premature fusion of certain skull bones (craniosynostosis).
This early fusion prevents the skull from growing normally
and affects the shape of the head and face.
Many features of Crouzon syndrome result from the
premature fusion of the skull bones:

Wide-set, bulging eyes and vision problems caused by shallow eye
sockets
Strabismus
Midfacial hypoplasia
Beaked nose:Upper airway obstruction develops secondary to septal
deviation, midnasal abnormalities, choanal abnormalities, and
nasopharyngeal narrowing
Underdeveloped upper jaw(Maxillary hypoplasia)
Dental problems and hearing loss, which is sometimes accompanied
by narrow ear canals.
Opening in the lip and the roof of the mouth
Mandibular prognathism




f)
Following are signs of good attachment
during breast feeding ?

f) Baby's mouth is wide open
g) Baby's lower lip is inverted
h) Upper areola is more visible than lower
i) Baby's chin touching the breast
j) Pain during sucking
Correct Answer - A:C:D
Ans. is 'a' i.e., Baby's mouth is wide open, 'c' i.e., Upper areola is
more visible than lower & 'd' i.e., Baby's chin touching the breast
Signs of good attachment are
. Baby's mouth is wide open,
86. Most of the nipple and areola in the mouth, only upper
areolar visible, not the lower one,
f) Baby's chin touches the breast and
g) Baby's lower lip is everted


87. All are metabolic causes of liver disease
except
7. Histiocytosis
8. Hemochromatosis
9. Gaucher's disease
10.
Wilson disease
11.
Galactosemia
Correct Answer - A
Ans. is 'a' i.e., Histiocytosis
Metabolic liver diseases can be classified into 3 categories:
Manifestations due to hepatocellular necrosis:
Galactosemia, hereditary fructose intolerance, tyrosinemia type
I, Wilson disease, Hemochromatosis and al -antitrypsin deficiency.
Cholestatic jaundice :Al -antitrypsin deficiency, Byler's disease,
cystic fibrosis, Niemann-Pick disease and some disorders
of peroxisome biogenesis.
Hepatomegaly (disorders with liver damage & storage diseases ):
Glycogen storage diseases, cholesteryl ester storage disease
and, when associated with splenomegaly, lysosomal storage
diseases (eg:- Gaucher disease).


i)
In a patient with L4-L5 disc prolapse,
which of the following nerve roots can
get compressed?

4. L5
5. S1
6. S2
7. S2-S4
8. L4
Correct Answer - A:E
Ans.is 'a' i.e.,L5 &'e' i.e.,L4
Like at every level, there are two nerve roots at L4-L5 - exiting nerve
root i.e. L4 nerve root & traversing nerve root i.e. L5 nerve root.
So, herniated disc at the L4-L5 level, can create an L5
radiculopathy or an L4 radiculopathy, depending on where the disc
herniation occurs (to the side or to the back of the disc) and which
nerve root is affected:?
Posterolateral (paracentral) - most common (90-95%), affects the
traversing /descending /lower nerve root i.e. L5 nerve root
Foraminal (far lateral, extraforaminal) - less common (5-
10%), affects exiting/upper nerve root i.e. L4 nerve root


.
Deformity associated with posterior
dislocation of hip joint?

. Flexion
. Extension
. Abduction
. Adduction
. Internal rotation
Correct Answer - A:D:E
Ans. is 'a' i.e., Flexion, `d' i.e., Adduction & 'e' i.e.,
Internal rotation

Usually, the head of the femur is dislocated posterior to the
acetabulum when the thigh is flexed, for example, as may occur in
a head-on automobile collision when the knee is driven violently
against the dashboard.
The significant clinical findings are shortening, adduction, and
internal rotation of the extremity.
Anteroposterior, lateral and, if fracture of the acetabulum is
demonstrated, oblique radiographic projections (Judet views)
are required.
Common associated injuries include fractures of the acetabulum or
the femoral head or shaft and sciatic nerve injury




191. True regarding achondroplasia is?
. Autosomal recessive
. Disproportionate dwarfism
. Subnormal intelligence
. Bullet shaped vertebral bodies on radiology
. Abnormal sexual development
Correct Answer - B:D
Ans. is 'b' i.e., Disproportionate dwarfism &'d' i.e.,
Bullet shaped vertebral bodies on radiology
ACHONDROPLASIA:

Achondroplasia is a disorder of bone growth that prevents the
changing of cartilage (particularly in the long bones of the arms
and legs) to bone.
SYMPTOMS:
80%-99% of people have these
symptoms Abnormality of the metaphysis
Abnormality of the ribs
Anteverted nares
Chronic otitis media
Depressed nasal bridge
Frontal bossing
Genu varum
Hyperlordosis
Macrocephaly
Neonatal short-limb short stature
Rhizomelia
Wormian bones

30%-79% of people have these
symptoms Abnormal form of the vertebral
bodies Conductive hearing impairment
Dental crowding
Dental malocclusion
Diaphyseal thickening
Hyperhidrosis
Intrauterine growth retardation
Joint hyperflexibility
Kyphosis
Long thorax
Midface retrusion
Muscular hypotonia
Narrow chest
Narrow sacroiliac notch
Obesity
Obstructive sleep apnea
Ventriculomegaly
5%-29% of people have these
symptoms Acanthosis nigricans Death in
infancy
Dysarthria
Elbow dislocation
Hydrocephalus
Joint stiffness
Spinal canal stenosis
Sudden cardiac death
Percent of people who have these symptoms is not
available through HPO

Autosomal dominant inheritance
Brachydactyly
Brain stem compression
Flared metaphysis
Generalized joint laxity
Infantile muscular hypotonia
Limited elbow extension
Limited hip extension


Lumbar hyperlordosis
Lumbar kyphosis in infancy
Malar flattening
Megalencephaly
Motor delay
Recurrent otitis media
Short femoral neck
Small foramen magnum
Spinal stenosis with reduced interpedicular distance
Trident hand
RADIOLOGICAL FINDING:
The inner contour of the pelvis has a typical, classic "champagne
glass" appearance
Large skull with relatively short base and a narrow and funnel -
shaped foramen magnum
Bullet shaped vertebral bodies
Long and short tubular bones are short and thick with apparent
increased diameter.
Metaphysis of long bones are widened and flared, physis are
notched or V -shaped.
The spinal canal is narrow with decreased interpedicular distance as
one proceeds from LI to L5.


.
Signs of compartment syndrome
include?

5. Pain on passive flexion
6. Pain on active flexion
7. Swelling of fingers
8. Pallor
9. Paraesthesia
Correct Answer - B:C:D:E
Ans. is 'b' i.e., Pain on active flexion, 'c' i.e. Swelling of
fingers, 'd' i.e. Pallor & `e' i.e. Paresthesia

Clinical features of compartment syndrome
Four signs are reliable in diagnosing a compartment syndrome :-
6. Paraesthesia or hypesthesia in nerves traversing the compartment
7. Pain with passive stretching of the involved muscles (stretch pain)
8. Pain with active flexion of the muscles
9. Tenderness over the compartment
Amongst these, stretch pain is the earliest sign of impending
compartment syndrome. The ischemic muscles, when
stretched, give rise to pain.
Passive extension of fingers (stretching the fingers) produce pain in
flexor compartment of forearm.
Other features are Pulselessness, paralysis, Pallor and pain out of
proportion to physical findings.
Peripheral pulses, are present initially and disappear later.
Therefore, pulse is not a reliable indicator for
compartment syndrome.




.
Which of the following statement is/are
correct about fracture management in
children?

. Supracondylar fracture of humerus can be managed by
closed reduction
. Lateral condylar fracture of humerus is known as fracture
of necessity
. Lateral condylar fracture of humerus is managed by
open reduction and screwing
. Forearm fracture in children can be managed by
closed reduction and casting
. Femoral neck fracture in adults is managed by surgery and
3 screws
Correct Answer - A:B:C:D:E
Ans. is 'a' i.e., Supracondylar fracture of humerus can be
managed by closed reduction; 'b' i.e., Lateral condylar fracture
of humerus is known as fracture of necessity ; 'c' i.e., Lateral
condylar fracture of humerus is managed by open reduction
and screwing, 'd' i.e., Forearm fracture in children can be
managed by closed reduction and casting & `e' i.e., Femoral
neck fracture in adults is managed by surgery and 3 screws
MANGEMENT OF SOME COMMON PEDIATRIC FRACTURES:

LOCATION OF
TYPE
MANAGEMENT
FRACTURE
Undisplaced(Gartland'sImmobilization in
type1)
plaster for 3 weeks.


Reduction (closed)
Angulated(Gartland's under anaesthesia
Supracondylar #
type 2)
followed by
of humerus
percutaneous pinning.
Same as type 2 / open
reduction if Open
Completely displaced (compound fracture)
(Gartland's types 3)
or associated vascular
injury (complicated
fracture).
If fracture is stable :
Immobilization in
above elbow cast in
90 degree flexion &
Lateral condyle
undisplaced
neutral rotation.
fracture of humerus
If fracture is unstable :
Closed reduction and
percutaneous pinning
(k wire fixation)
Closed reduction and
Only displaced
percutaneous pinning.
displaced and rotated ORIF with screws.
Shaft of the
displacement is
Closed reduction and
forearm bones
minimal
elbow cast
Conservative
treatment by
immobilization in
Undisplaced fracture or thomas knee splint
valgus impacted
In situ fixation with
three parallel
cannulated screws
Femur neck
(preferred)
< 60 years :CRIF with
3 parallel cannulated
screws> ORIF.
60 - 70 years :CRIF
Displaced


>70 YEARS: Bipolar
hemiarthroplasty;AMR
Head is viable: Mc-
Murry's osteotomy;
Meyer's procedure
Arthroplasty
Physiological age < 65
:Bipolar
years
Head hemiarthroplasty
OLD >3WEEKS
not or AMR. viableWith
arthritis:
Total hip
replacement.
Bipolar
hemiarthroplasty or
Physiological age > 65
AMR.
years
With arthritis: Total
hip replacement.
. of distal third
of radius with
Open reduction of the
dislocation or
Galeazzi fractures
radius and the distal
subluxation of inferior dislocation
radio-ulnar joint.
(distal) radio-ulnar
joint


11. All are features of inflammatory arthritis
except?
. Morning stiffness
. X-ray showing sclerosis
. Elevated ESR
. Weight gain
. Swelling of joints
Correct Answer - B:D
Ans. is `b' i.e., X-ray showing sclerosis & 'd' i.e., Weight
gain Features of inflammatory arthritis :?

Presence of some or all four cardinal signs of inflammation:
Erythema
Pain
Warmth
Swelling
Systemic symptoms
8. Prolonged morning stiffness,often lasting for several hours.
(Non inflammatory arthritis are associated with intermittent
stiffness, Stiffness usually lasts less than 1hr).
. Fatigue
. Fever
. Weight Loss
Laboratory evidence of inflammation
. Elevated ESR
. Thrombocytosis
. Elevated C reactive protein

. Anemia of chronic disease.
X-ray
X-ray feature of inflammatory arthritis shows rarefaction while x-ray
features in non-inflammatory arthritis reveals sclerosis.


.
Risk factor(s) for Leg-Calf-Perthe's
disease is/are?

13.
Accelerated skeletal growth
14.
Growth hormone abnormalities
15.
Positive family history
16.
Female sex
17.
Passive smoking
Correct Answer - C:E
Ans. is 'c' i.e., Positive family history & `e' i.e., Passive smoking
PERTHE'S DISEASE (LEGG-CALVE PERTHE'S DISEASE)
Perthe's disease is also known as osteochondritis deformans
juvenilis or Coxa piano or Pseudocoxalgia.
Perthe's disease is an osteochondritis of the epiphysis of the femoral
head (capital femoral epiphysis). In the disease, the femoral head
becomes partly or wholly avascular and deformed. The disease
occurs commonly in males in the age group of 5-10 years.
Perthe's disease is the most common form
ofosteochondrosis (osteochondrosis are characterized by avascular
necrosis (AVN) and defective endochondral ossification of primary
or secondary ossification centres).
Etiology of Perthe's disease
The etiology remains unknown, but is currently accepted that the
disorder is caused by an interruption of the blood supply to the
capital femoral epiphysis, causing avascular necrosis.


Risk factors
Risk factors for Legg-Calve-Perthes disease include:
Age. Although Legg-Calve-Perthes disease can affect children of
nearly any age, it most commonly begins between ages 4 and 8.
Your child's sex. Legg-Calve-Perthes is up to five times more
common in boys than in girls.
Race. White children are more likely to develop the disorder than
are black children.
Genetic mutations. In a small number of cases, Legg-Calve-
Perthes disease appears to be linked to mutations in certain
genes.


196. All are true about septic arthritis except ?
10.
Staph aureus is the most common cause
11.
Most common cause is E. coli
12.
Common in children
13.
Affects growth plate
14.
Aspiration of joint fluid used for diagnosis
Correct Answer - B
Ans. is 'b' i.e., Most common cause is E. coli
Septic arthritis (Acute suppurative arthritis)

Septic arthritis refers to pyogenic infection of a joint, i.e., infection of
a joint by pyogenic organism (bacteria).
The microbial spectrum is diverse in suppurative arthritis, but
staphylococcus aureus infection is most common.
The joint can become infected by : ?
. Hematogenous spread from a distant site (most common route).
15. Direct invasion through a penetrating wound, intra articular
injection, arthroscopy.
16. Direct spread from adjacent osteomyelitis especially in joints
where Metaphysis is intra articular e.g., hip and shoulder.
Clinical features
Disease is more common in children.
Knee joint is the most commonly affected joint.
Other joint which are affected are hip, shoulder and elbow.
The child is toxic with fever, tachycardia, tachypnea.
There is severe pain, swelling, and redness over the joint.


Movements are severely restricted and the joint is held in
the position of ease.
Weight bearing on limb is not possible.
Diagnosis:
Aspirated synovial fluid in septic arthritis will have markedly
increased polymorphonuclear leukocytes


197. Hanging cast is used for?
k) Femur #
l) Radius #
m)
Humerus #
n) Tibia #
o) Fibula #
Correct Answer - C
Ans. is 'c' i.e., Humerus #
Cast/ Brace
Used In
Hanging cast & "Sugar tong" Fracture shaft
or Coaptation splints
humerus
Cylinder cast
Fracture patella
Minnerva cast, Halo device Cervical spine
Risser's cast, Milwaukee
brace
Scoliosis
Boston brace
Palvic harness, Von Rosen Congenital
splint,
(Developmental)
Ilfeld or Craig splint
Dysplasia of Hip
Broom stick (Petrie)
cast, Snyder sling, Pattern -
bottom brace, Toronto brace
with universal joint,
Legg Calve-
Birmingham brace,
Perthes
Tachdjian brace (trilateral
Disease


hip abduction orthosis),
Newington brace, Atlanta
scottish Rite brace


198. True about acute osteomyelitis ?
k) Cannot be detected on X-ray before 2 weeks
l) Bone scan detect after 2 weeks
m)
Severe pain
n) Secondary osteomyelitis associated with compound fracture
is more common than primary variety
o) Limitation of movements
Correct Answer - C:E
Ans. is 'c' i.e., Severe pain &`e' i.e., Limitation of movements
ACUTE OSTEOMYELITIS:
18. It Primary (hematogenous): - Organisms reach the bone
through blood stream.
p) Secondary: - Organism gain entry directly through wound such
as in compound fractures or surgical operation.
Hematogenous osteomyelitis is the commonest form of osteomyelitis
and most common source of bone and joint infection is
hematogenous.
Clinical features :
Metaphysis of long bone is the earliest and most common site
involved.
Most common in children.
The bones most commonly :
Proximal tibial
Distal femur
Proximal humerus.
Features:
Severe pain, fever, malaise, chills & rigors, sweating, and even


shock.
Local tenderness (finger tip tenderness), raised local temperature,
Local erythema and limitation of movements (typically the limb is
held still).
Diagnosis:
X-rays
Earliest sign : periosteal reaction (periosteal new bone formation) at
the metaphysis, which takes about 7-10 days.
Bone scan
Increased uptake by bone in metaphysis within 24 hours of onset of
symptoms (earliest sign.


199.


Investigations which are a part of pre-
conceptional diabetes testing are?

g) Hb Al c level
h) LH & FSH level
i) Frequent blood sugar estimation
j) TSH estimation
k) All of the above
Correct Answer - A:C:D
Ans. is 'a' i.e., HbAlc level, 'c' i.e., Frequent blood
sugar estimation & 'd' i.e.,TSH estimation Pre-
conceptional tests in diabetic woman

Testing that should be done as part of preconception care for
women with diabetes include the following
Maternal
Routine rubella, rapid plasma reagin
Hepatitis B virus and HIV testing
Blood typing
Frequent blood sugar estimation
Hemoglobin A1C
Thyroid-stimulating hormone
Serum creatinine
Urine albumin-to-creatinine ratio, and
Cervical cultures
Fetal
Biophysical profile
NST

Doppler umbilical arterial velicometry


.
True statement regarding magnesium
sulphate use in eclampsia & pre-
eclampsia is/are?

. Used for the treatment of hypertension
. Can be used continuous intravenous as well as
intermittent intramuscular
. In > 95% of cases successfully controls seizures of eclampsia
. Toxicity can be treated by stopping further administration
and giving calcium gluconate
. Administration is continued for 24 hours after delivery
Correct Answer - B:C:D:E
Ans. is 'b' i.e., Can be used continuous intravenous as well as
intermittent intramuscular, 'c' i.e., In > 95% of cases
successfully controls seizures of eclampsia, 'd' i.e., Toxicity
can be treated by stopping further administration and giving
calcium gluconate & `e' i.e., Administration is continued for
24 hours after delivery

MAGNESIUM SUPHATE IN ECLAMPSIA & PRE-
ECLAMPSIA:
Indicated to prevent seizures associated with pre-eclampsia, and for
control of seizures with eclampsia
In > 95% of cases successfully controls seizures of eclampsia.
Dose: 4-5 g (diluted in 250 mL NS/D5W) IV in combination
with either :

Up to 10 g (10 mL of undiluted 50% solution) divided and
administered IM into each buttock or

After initial IV dose, 1-3 g/hr IV.
MgSO4 is continued 24 hours after delivery to prevent post-partum
eclampsia
Monitoring:
Throughout the administration of magnesium, the patient needs
continuous clinical monitoring for magnesium toxicity
Toxicity is manifested by loss of deep tendon reflexes (patellar),
decrease in respiratory rate, oliguria and altered mental status.
Comparatively, loss of deep tendon reflexes appears to be the
earliest of all the signs and it occurs when the magnesium level
exceeds 5 mmol/L.
Toxicity of MgSO4 is monitored by:
Urinary output,
Respiratory rate,
Knee jerk


20. Enzymes elevated in dysgerminoma
include?
g) AFP
h) HCG
i) Insulin
j) PLAP
k) LDH
Correct Answer - B:D:E
Ans. is 'b' i.e.,HCG, 'd' i.e., PLAP & 'e' i.e., LDH
Dysgerminoma is the most malignant germ cell tumour (not
a virilising tumour).

Seen in young females like other GCT (not in post menopausal
women).
It is unilateral.
Its cut section is soft due to degeneration (gritty cut section is seen
in Brenner's tumour).
Tumour markers for Dysgerminoma are :
LDH
Alpha fetoprotein is normal in dysgerminoma.
Placental alkaline phosphatase.
Beta HCG


.
Which of the following is NOT effective in
controlling the hot flushes of menopause
in a woman?

. Hormone replacement therapy
. Tibolone
. Raloxifene
. Isoflavones
. Vitamin E
Correct Answer - C
Ans is 'c' i.e., Raloxifene
Hot Flushes : They are the 'hallmark' of menopause. Hot flushes
are described as recurrent transient period of flushing, sweating
and a sensation of heat often accompanied by palpitations. feelings
of anxiety. and sometimes followed by chills.
The entire episode lasts no more than 1-3 minutes and recurs 5-10
times / day (can occur upto 30 times a day). Short term estrogen
therapy results in resolution of hot flushes.
Hormone therapy
Estrogen therapy (most effective)
Combined estrogen and progestin therapy
Progestin therapy (to be given in those women in whom estrogen is
contraindicated)
Tibolone
Non hormonal prescription medicines :Not FDA
approved Clonidine
Selective serotonin reuptake inhibitor : paroxetine, fluoxetine

Serotonin and nor epinephrine reuptake inhibitor : venlafaxine
Dopamine antagonist : Veralipride Gabapentin
Bellergal (combination of ergotamines, phenobarbital and belladona,
approved for the treatment of migrain).)
Mertazapine
Trazodone
Non prescription medicines :
Isoflavones (100 mg/day)
Soy products (60 g/d)
Vitamin E (800 IU/day)
Life style changes :
Reducing body temperature
Maintaining a healthy weight
Smoking cessation
Paced respiration


17. Which of the following are included in
causes of precocious puberty?
k) Hypothyroidism
l) Adrenal insufficiency
m)
McCune Albright syndrome
n) Craniopharyngioma
o) All of the above
Correct Answer - E
Ans. is 'e' i.e., All of the above
Precocious puberty
Puberty before the age of 8 years in girls or 9 years in boys is
considered precocious puberty.
Menarche before the age of 10 years in girls is also considered as
precocious.
Etiology:
Central precocious puberty
Idiopathic : Sporadic or familial.
Central nervous system abnormalities
Congenital anomalies of CNS: Hypothalamic hamartoma,
hydrocephalus, porencephaly, arachnoid cysts.
Acquired lesions of CNS : Inflammation, granuloma, trauma,
surgery, radiation, chemotherapy.
Tumors of CNS : Pineal tumors, optic glioma, ependymoma,
craniopharyngioma.
Hypothyroidism
Peripheral precocious puberty : Isosexual


Girls
Ovarian causes : McCune-Albright syndrome, benign follicular cysts,
granulosa-theca cell tumors; Gonadoblastoma
Adrenal causes : Feminizing adrenal neoplasia
Exogenous estrogen administration
Boys
Testis : Leydig cell tumor, adrenal rest tumor, testotoxicosis.
Adrenal: CAH (21 or 11-(3 hydroxylase deficiency), virializing
tumors.
hCG secreting tumors : Hepatoma, hepatoblastoma,
choriocarcinoma, chorionepithelioma, teratoma,
dysgerminoma.


204. Primary amenorrhea is associated with?
22.
Polycystic ovary disease
23.
Turner syndrome
24.
Laurance-Moon-Biedl syndrome
25.
Kallmann syndrome
26.
Kustner-Hauser- syndrome
Correct Answer - A:B:C:D:E
Ans. is 'All' i.e., a, b, c, d & e
CAUSE OF PRIMARY AMENORRHEA:
Stress , emotional disturbances , infection,
Cerebral cortex trauma
Delayed puberty, Kallmann syndrome, vigorous
Hypothalamus
exercise, weight loss, anorexia nervosa, chronic
disease (such as TB)
Empty sella, Frohlich syndrome, Laurance-Moon-
Pituitary
Biedl syndrome, cushing disease, pineal tumor,
prolactinoma, galactosemia
Primary ovarian failure (Savage syndrome),
Ovary
resistant ovarian syndrome, PCOD, 17-
hydroxylase deficiency
Absent uterus in Mayer-Rokitansky-Kiister-Hauser
(MRKH) syndrome, refractory endometrium,
Genital tract
imperforated hymen, transverse vaginal septum,
atresia of upper third of vagina and cervix,
complete absence of vagina
Intersex, Turner syndrome (45X0), testicular


Chromosomal
feminization syndrome (46XY), pure Gonadal
dysgenesis (46XX OR XY)
Other endocrine Juvenile diabetes, cretinism, adrenogenital
glands
syndrome
Tranquilizers, antidepressants, antihypertensives,
Drugs
estrogen, metaclopramide
Nutrition/systemic Overweight, Malnutrition, weight loss, anemia,
illness
TB


g) Methods used for delivery of shoulder in
shoulder dystocia include?
k) Lovcetmaneuver
l) Macrobert'smaneuver
m)
Panderdmaneuver
n) Wood's maneuver
o) Jacquemiermaneuver
Correct Answer - B:D:E
Ans. is 'b' i.e., Macrobert'smaneuver, 'd' i.e., Wood's maneuver&
'e' i.e., Jacquemiermaneuver
Shoulder Dystocia Maneuvers
Maternal Maneuvers
McRoberts maneuver
Suprapubic pressure
Gaskin maneuver (all-fours)
Sims maneuver (lateral decubitus)
Ramp maneuver
Symphysiotomy
Fetal Maneuvers
Rubin maneuver
Jacquemier maneuver (posterior arm delivery)
Wood screw maneuver
Zavanelli maneuver (cephalic replacement)
4 Cleidotomy
Shute forceps maneuver


206.


Which of the following important points
needed to be informed to a pregnant woman
when taking consent vaginal birth after
caesarean section (VBAC)?

. Chances of uterine rupture are 4-5%
. Chances of uterine rupture are double with previous history
of more than one caesarean section
. Uterine delivery is contraindicated in females with
previous history of lower segment caesarean section
. Chances of fetal compromise are l5-25%
. None
Correct Answer - E
Ans. is 'None'
Vaginal Birth After Cesarean (VBAC) Trial of Labour
(TOL Selection Criteria

One previous lower segment transverse scar
Pelvis adequate for the fetus
Continued labour monitoring possible
Availability of resources (Anesthesia,blood transfusion and OT), for
emergency cesarean section within 30 minutes of decision
Informed consent of the women
Contraindications
Previous classical or inverted-T shaped uterine incision
Previous 2 or more LSCS
Pelvis contracted or suspected CPD
Presence of other complications in pregnancy- Obstetric(Pre-


eclampsia, malpresentation, placenta praevia) or medical
Resources limited or emergency cesarean delivery or
patient refusal
. Note- according to new recommendations (2017), 2 or more
LCSC are no more considered absolute indication for VABC.
. Women who have had two or more lower segment caesarean
deliveries may be offered VBAC after counselling by a senior
obstetrician. This should include the risk of uterine rupture and
maternal morbidity, and the individual likelihood of successful VBAC
(e.g. given a history of prior vaginal delivery).
k) Labour should be conducted in a centre with suitable expertise
and recourse toimmediate surgical delivery."


207. True statement regarding CIN-l is/are?
k) Associated with HPV-16
l) Occurs at squamo-columnar junction of cervix
m)
Dysplasia involves 2/3rd of the thickness of the epithelium
n) It is kept under observation with PAP smear or HPV DNA tests
o) Disappears spontaneously in 60% percentage of females in l-
2 years
Correct Answer - A:B:D:E
Ans. is 'a' i.e.,Associated with HPV-16, 'b' i.e., Occurs at
squamo-columnar junction of cervix, 'd' i.e., It is kept under
observation with PAP smear or HPV DNA tests & 'e' i.e.,
Disappears spontaneously in 60% percentage of females in 1-
2 years

Cervical intraepithelial neoplasia (CIN) (cervical dysplasia)
CIN refers to the potentially premalignant transformation of cells
of the cervix.
Site:
Squamo-columnar junction (SCI) of the cervix(most common),
vaginal walls and vulvar epithelium.
Etiology:
Chronic Human papilloma virus (HPV):high risk type of HPV are 16,
18, 31, or 33
Risk factors:
Immunodeficiency
Women who give birth before age 17/ are sexually active before age
18
Poor diet, multiple sexual partners, lack of condom use, and


cigarette smoking
Diagnosis:
Pap smear (abnormal cells)
Next step :colposcopy
Biopsy
DNA test (To detect high risk HPV)
Treatment:
CIN 1:
No treatment required
Left untreated, about 60-70% of CIN-1 will regress within one year,
and 90% will regress within two years
Higher-grade CIN:
Cryocautery, electrocautery, laser cautery, loop electrical excision
procedure (LEEP), or cervical conization.
Simple hysterectomy (abdominal/vaginal):
CIN persists or does not improve after other procedures
If family is complete
Associated prolapse or fibroid or when the patient is not ready for
regular follow-up.


208. Indications of Karyotyping include?
26.
Recurrent abortions
27.
Multiple malformations in child
28.
Delayed puberty
29.
Advanced maternal age
30.
All of the above
Correct Answer - E
Ans. is 'e' i.e., All of the above
INDICATIONS FOR KARYOTYPING:
Prenatal period:
Advanced maternal ageBalanced chromosomal rearrangement (
translocation, inversion) or not ( marker, mosaicism, sexual
aneuploidy) in one of the parents
Previous pregnancy with an abnormal chromosomal complement
Previous stillbirth with an abnormal chromosomal complement
Abnormal maternal serological screening test
Abnormal foetal ultrasound
Risk of an unstable chromosomal syndrome
Perinatal Period:
In a still birth
Postnatal Period:
Confirmation of clinical Analysis
Ambiguous genitalia
Congenital dysmorphic syndromes and /or Malformations and/or
neurological deficit
Pseudohermaphrodism (male /female)
Gonadal dysgenesis

True hermaphrodyte
Childhood & adolescence:
Developmental delay (array testing preferred)
Delayed puberty or inappropriate secondary sexual development
Psycho motor retardation (fragile X, Klinefelter)
Dysmorphism with visceral anomalies undetected during infancy
Unexplained short stature (Turner)
Hypogonadism (Klinefelter, Turner).
Chromosome breakage syndromes: ataxia telangiectasia, Bloom
syndrome, Fanconi anaemia and Nijmegan breakage syndrome.
Adult:
History of pregnancy loss
Parents of a child with a chromosomal disorder
Infertility/Sterility workup once gynecological or endocrine causes
have been ruled out (Klinefelter, Turner).
Testicular feminization syndrome


g) TRUE statement regarding pre-natal
aneuploidy screening is?
k) AFP is tested in Down syndrome
l) Nuchal translucency is seen at 10-12 weeks
m)
Anomaly scan is done at 12-16 weeks
n) Triple marker test is done at 15 - 22 weeks
o) Quadruple marker test is done at 10- 12 weeks
Correct Answer - A:B:D
Ans. is 'a' i.e., AFP is tested in Down syndrome, 'b' i.e., Nuchal
translucency is seen at 10-12 weeks & 'd i.e., Triple marker
test is done at 15 -- 22 weeks

Screening methods to detect aneuploidy:
Combinations of first-trimester and second-trimester screening yield
Down syndrome detection rates as high as 90 to 95%. First
trimester screening (11-14 weeks)
Combined first trimester screening:
PAPP(Pregnancy associated plasma protein -A) at 9 weeks
Nuchal translucency (NT):
11-13 weeks
Marker for Down syndrome
Serum hCG at 12 weeks
In cases of fetal Down syndrome: Serum free Beta-hCG level is
higher approximately 2.0 MoM and the PAPP-A level is lower,
approximately 0.5 MoM.
With trisomy 18 and trisomy 13, levels of both analytes are lower.
Second trimester screening (15-22 weeks)

15 weeks & 22 weeks
MSAFP: low level are found in down syndrome
Triple test:
Includes MSAFP, hCG& uE3 (unconjugated estriol).
Detection of down syndrome.
In an affected pregnancy, levels of MSAFP & uE3 tend to low while
that of hCG is high
Quadruple (Quad) screening:
MSAFP, hCG, uE3 & dimeric inhibin -A.
Detect trisomy 21 in 85% cases.
Level of serum analytes in cases with trisomy 21: MSAFP -reduced,
hCG-increased, uE3-reduced& dimeric inhibinA-elevated.
Anomaly scan
Anatomy scan, 18-22 weeks ultrasound, or level 2 ultrasound)
Includes:
Fetal number, including number of amniotic sacs and chorionic sacs
for multiple gestations
Fetal cardiac activity
Fetal position relative to the uterus and cervix
Location and appearance of the placenta, including site of umbilical
cord insertion when possible
Amniotic fluid volume
Gestational age assessment
Fetal weight estimation
Fetal anatomical survey
Evaluation of the maternal uterus, tubes, ovaries, and surrounding
structures when appropriate


24. Best predictors of estimation of
gestational age is/are?
k) Biparietal diameter at age of 18 weeks
l) Crown rump length at age of 7-10 weeks
m)
Crown rump length at age of I11-14 weeks
n) Gestational sac diameter at 5-6 weeks
o) Crown rump length at age of 14-18 weeks
Correct Answer - A:B:D
Ans. is 'a' i.e., Biparietal diameter at age of 18 weeks, `b' i.e.,
Crown rump length at age of 7-10 weeks & 'd' i.e., Gestational
sac diameter at 5-6 weeks
FIRST TRIMESTER
Fetal anatomy and viability:
Mean Sac Diameter (MSD)
Findings
5?8 mm
Yolk sac
12 mm
Embryo
15?18 mm
Cardiac activity
Embryo CRL > 4 mm
Cardiac activity
Gestational age and Embryonic Structures:
Menstrual Age
Fetal Structures
Choriodecidual thickness,?
4 weeks
chorionic sac
9. weeks
Gestation sac, embryo yolk
6 weeks
sac Fetal pole, cardiac activity
Lower limb buds, midgut?
7 weeks
herniation (physiological)


8 weeks
Upper limb buds, stomach
9 weeks
Spine, choroid plexus
SECOND TRIMESTER
Fetal growth:
Normal:10th-90th percentiles
Biparietal diameter (BPD):(12 week )Outer skull edge of
proximal skull to inner edge on distal skull
Head circumference (HC)
Abdominal circumference (AC):Measured at junction of left and right
portal veins & umbilical vein
Femur length (FL):Measured when beam from the transducer is
perpendicular to the shaft
Brithweight(BW)
Hadlock formula :
log(10)BW = 1.335 - 0.0034(AC)(FL) + 0.0316(BPD) + 0.0457(AC)
+ 0.1623(FL)
Gestational age assessment:
Optimum time for most accurate assessment :14 -20 weeks
by Crown rump length
Transcerebellar diameter (TCD):14 weeks- 28 weeks
Dating ultrasound done before 22 weeks should be used in
preference to menstrual dates
THIRD TRIMESTER:
Estimated fetal weight (EFW) is determined from the average
of three readings for:

FL
AC is most important.
BPD.


4.
Pregnancy is contraindicated in which
heart disease?

29.
Eisenmenger syndrome
30.
Multi valvular disease
31.
Congenital heart disease
32.
Coarctation of aorta
33.
Ejection fraction < 40%
Correct Answer - A:D:E
Ans. is 'a' i.e., Eisenmenger syndrome, 'd' i.e., Coarctation
of aorta & 'e' i.e., Ejection fraction < 40%

Heart diseases in which pregnancy is contraindicated :
Marfan syndrome
Aortic disease with dilatation of > 50mm with bicuspid aortic valve.
Coarctation of aorta
Eisenmenger syndrome
Severe fixed obstructive lesions ( aortic stenosis, mitral stenosis,
pulmonic stenosis, hypertrophic obstructive cardiomyopathy)
NYHA class 3 & 4 heart diseases (severe systemic ventricular
dysfunction/CHF)
Ejection fraction < 30-40%
Previous peripartum cardiomyopathy with any residual impairment of
LV function
Pulmonary arterial hypertension of any cause
Severe cyanosis


g) A female with IUCD develops pelvic
inflammatory disease. which of the
following should be done:

30.
Keep the IUCD, give antibiotic, follow up for antibiotic
response & then take decision regarding IUCD removal
31.
Start antibiotic & remove IUCD
32.
Remove IUCD & start antibiotic
33.
Wait for next menstrual cycle for any intervention
34.
Do nothing
Correct Answer - A
Ans. is 'a' i.e., Keep the IUCD, give antibiotic, follow up
for antibiotic response & then take decision regarding
IUCDremoval

If an IUD user receives a diagnosis of PID, the IUD does not need to
be removed. However, the woman should receive treatment
according to these recommendations and should have close clinical
follow-up no clinical improvement occurs within 48-72 hours of
initiating treatment, providers should consider removing the IUD.
PID treatment regimens must provide empiric, broad spectrum
antibiotic coverage of likely pathogens


g) Treatment of cervical incompetence
includes?
k) Shirodkar's stitch
l) Mc donald's stitch
m)
B lynch stitch
n) Fothergills operation
o) Sling operation
Correct Answer - A:B
Ans. is 'a' i.e., Shirodkar's stitch & 'b' i.e., Mcdonald's stitch
TREATMENT OF CERVICAL INCOMPETENCY:
Surgical:
Transvaginal techniques
McDonald cerclage: purse-string suture that passes through the
cervical stroma
Shirodkar cerclage :requires dissection of the bladder anteriorly and
the rectum posteriorly in order to place the stitch at the level of the
internal os.
Transabdominal cerclage (Benson & Durfee cerclage): Indicated in
Anatomic limitations (eg, after trachelectomy) Congenital short
cervix
History of failure of a transvaginal cerclage.
Incompetent cervix.
About other options
. B lynch stitch ? compression sutures to control PPH
. Fothergills operation ? surgical treatment of uterine prolapse
. Sling operation ? synthetic tapes used to support uterus in 2"dand


3rd degree uterine prolapse


214. Langerhan's cells are reduced in?
32.
Oral lichen planus
33.
Basal cell carcinoma
34.
Contact dermatitis
35.
Langerhan's cell histiocytosis
36.
Sarcoidosis
Correct Answer - B:E
Ans. is'b'i.e. Basal cell carcinoma &'e'i.e. Sarcoidosis Ref:
hltlts://www.ncbi.nlm,nih.govlpmc/articles/PMC3424941/:
htt,s://www.ncbi.nlm.nih.gov/pubmed/24691282: https://www.ncbi.nlm.n
"Langerhans cells play a role in cell-mediated immune reactions
which are often depressed in sarcoidosis".
"We have shown that the number of Langerhans cells is
considerably lower in cutaneous basal and squamous cell
carcinomas, compared with their number in the normal
skin.
Reduced in,
Ageing
Basal cell carcinoma skin
Squamous cell carcinoma skin
Sarcoidosis
HIV/AIDS
HPV infection
Lichenoid drug eruptions
Increased in,
Gingivitis and periodontitis
Oral LP
Contact hvpersensitivily

Recurrent aphthous stomatitis
Behcet's disease
Oral cancers
Langerhans cell histiocytosis ( LCH )


g) Which of the following not a sub-type of
lichen planus?
k) Lichen planus hypertrophicus
l) Lichen planus pigmentosus
m)
Ulcerative lichen planus
n) Lichen nidatus
o) Lichen scrofulosorum
Correct Answer - D:E
Ans. is 'd'i.e., Lichen nitidus &'e' i.e. Lichen scrofulosorum
Ref: Neena Khanna 3'd/e p. 52-53; Behl l$h/e p.265; Rook Vhelp.
5.13; IADVL textbook of dermatology 3'd /e p.1070; en.wikipedia.org
Forms:
Annular, Linear, Hvoerlroohic, Atrophic, Bullous, Pigmented.
Overlap syndromes:
Lupus erythematosus overlap syndrome.
Lichen sclerosus overlap syndrome,
Other variants of cutaneous LP mav include:
Lichen planus pemphigoides
Keratosis lichenoides chronica ("Nekam's disease")
Lichenoiil keratosis (Benign lichenoid keratosis/Solitary lichen
planus)
Lichenoid dermatitis.
Lichen nitidus:
Uncommon inflammatory skin condition that usually presents with
tiny skin-coloured bumps in children.
Although it has been considered a variant of lichen planus.

Lichen nitidus is now believed to be a separate and distinct entity!.
Lichen scrofulosorum (tuberculosis cutis lichenoides):
Rare tuberculid that presents as a lichenoid eruPtion of minute
papules in children and adolescents with tuberculosis.


216. Nail changes in psoriasis include?
. Pitting
. Pterygium formation
. Subungual hyperkeartosis
. Thinning of nails
. Oil spots
Correct Answer - A:C:E
Ans. is 'a' i.e., Pitting, 'c' i.e. Subungual hyperkeratosis &'e'
i.e. Oil spots

Ref: Neena Khanna 4th/e p. 40-4$ Roxburgh p. 128-42;
Venkataram little p. 49)
Associated clinical features in psoriasis
Nail changes
Occurs in 10-50% of patients.
The characteristic changes are : -
Pitting (thimble pitting) > Most common nail change.
Other changes > Oil spots, nail plate thickening,
Subungualhyperkeratosis, Onycholysis, Yellow-brown discoloration


217. "Facies leprosa" is characterized by?
. Madarosis
. Resorption of anterior nasal spine
. Collapse of nasal bridge
. Loss of upper incisors
. Diffuce infiltration of face
Correct Answer - B:D
Ans. is 'b' i.e., Resorption of anterior nasal spine &'d' i.e. Loss
of upper incisors Ref, https://www.ncbi.nlm.nih.gov/ p ubmed/ 3268
5 20? adopt= Abstract: http : / / ila. ilsl.b r / p dfs/ v 56n I a0 3.p df l
Note- other mentioned facial features are also seen in leprosy but they
are not included in term 'Facies leprosa'.
Facies leprosa:
Facies leprosa is a term used to describe resorption of bone in the
facial region of patients with leprosy, was first introduced by
Moller-Christensen and colleagues.
It is characterized by a combination of nasal change and resorption
of nasal bone, anterior nasal spine, supra-incisive alveolar region
and anterior alveolar process of the maxillae, associated with the
loss of upper incisors teeth, according to the criteria of
radiographic interpretation.
Other facial features of leprosy
Nodules with predilection for external ears.
Madarosis (loss of lashes and eyebrows).
Saddle Nose (Collapse of the nasal bridge) and perforation of
palate.
Testicular involvement results in loss of testicular sensation, loss of


libido, impotence & gynaecomastia.
Leonine Face (Diffuse dermal infiltration of face.


30. All the following are causes of scarring
alopecia except?
k) Lichen planus
l) Tinea capitis
m)
DLE
n) Sarcoidosis
o) Trichotilomania
Correct Answer - B:E
Ans. is'b'i.e., Tinea capitis &'e'i.e., Trichotillomania
[Ref Neena Kenna p. 141; Harrison lgh/e p. 355 ; Roxburgh 17/e
p. 270
Causes of Scarring alopecia (Cicatricial alopecia):
Local cuases:
Cutaneous Lupus(DLE)
Lichen Planus
Folliculitis decalvans
Linear scleroderma (Morphea)
radiation
Central centrifugal cicatricial alopecia
Alopecia cutis
Congenital atrichia
Systemic causes:
SLE
Sarcoidosis
Cutaneous Metastasis
Cicatricial alopecia is also known as pseudopelade.

Note - Most common congenital cicatricial alopecia is cutis
congenita i.e., f<tcal absence of epidermis with or without
other layers.


219. Painful genital ulcers are seen in?
35.
Syphilis
36.
Chancroid
37.
LGV
38.
Herpes genitalis
39.
Granuloma inguinale
Correct Answer - B:D
Ans' is'b'i.e., Chancroid &'d'i.e., Herpes genitalis Ref,
Neena Khanna 3d/e p. 259-60; Khopkar # /e p. 232
Primary syphilis (hard chancre)
Donovanosis (Granuloma inguinale)
Chancroid (soft chancre)
LGV
Herpes genitalis


220. Skin hazards of swimming are ?
k) Pyoderma gangrenosum
l) Verrucae
m)
Mmarinum infection
n) Mulcerans infection
o) Shigella
Correct Answer - B:C:E
Ans. is'b'i.e. Verrucae,'c' i.e., M marinum infection
"Large outbreaks of infection due to M. marinum have been
described in association with swimming pools (swimming
pool granuloma) and fish tanks (fish tank granuloma)"
"Verrucas, like most warts, are due to a viral infection of the growing
layers of the skin.
They get in when the skin is injured in some way.
This is especially the case for barefoot activities surrounding
swimming pools.
The plantar skin on the base of the feet gets soggy) and is more
easily damaged.
Shigella is transmitted directly or indirectly via the fecal-oral route
and may occur due to the ingestion of contaminated food or
water.
Of great significance is the low infective dose of between 10-100
organisms.
This disease may be acquired by swimming in contaminated surface
waters or pools and spa.




k)
A patient, planned for cesarean section,
develops hypotension 8 minutes after
the spinal anesthesia. Drugs which can
be used to treat this are?

33.
Ephedrine
34.
Mephenteramine
35.
Adrenaline
36.
Dopamine
37.
Steroids
Correct Answer - A:B:C:D
Ans. is'a'i.e., Ephedrine, 'b' i.e., Mephenteramine, 'c'
i.e., Phenylephrine &'d' i.e. Dopamine

[Ref: Miller Vh/e p. 1617]
Managing hypotension induced by spinal anesthesia for
caesarean section:
Treatment
In spite of using all the prophylactic measures,40o/o to 60% of
patients will still need treatment for hypotension:-
k) Fluid loading is superior to no-fluid regimen; however, the
incidence of PSH is still high with all fluid loading protocols
l) Vasopressors:-
Phenylephrine(PE) is preferred vasopressor.
Prevention and treatment of PSH because of faster onset.
Ephedrine may be more beneficial in patients with bradycardia.
Norepinephrine infusion was recently investigated as an alternative
for prophylaxis of PSH.

Ondansetron was reported as a prophylactic drug from PSH
Other sympathomimetic drugs used are mephentermine.
metaraminol, methoxamine, dopamine and, angiotensin II
Atropine should be given for bradycardia


.
In comparison to IJV cannulation, true
about subclavian vein cannulation
is/are?

. More chances of pneumothorax
. More incidence of catheter malposition
. More infectious complications
. More safety in ultrasound guided technique
. All of the above
Correct Answer - A:B
Ans. is'a'i.e., More chances of pneumothorax &'b'i.e.,
More incidence of catheter malposition

[Ref: https://www.ncb i. nlm.nih. gov/ pmc/ articles/ PMC i 27 09
25 /17]
Subclavian vein cannulation
Good external landmarks
Large radius
Practical method of central line in cardio-respiratory arrest
Blind procedure
Ultrasound not much useful
Should not be attempted in children < 2 years
Unable to compress bleeding vessels
More common & frequent: Catheter malposition, Pneumothorax,
hemothorax, Pinch-off sy:rdrome.
Less common & frequent: Arterial puncture, Thrombosis,
infectious complications.


38. Which of the following anesthetic should
not be used in a patient of chronic renal
failure?

i) Methoxyflurane
j) Ketamine
k) Pancuronium
l) Succinylcholine
m) Desflurane
Correct Answer - A:B:C
Ans. is 'a' i.e., Methoxyflurane, 'b' i.e., Ketamine &'c'
i.e., Pancuronium

[Ref: Morgan 4th/e p. 219]
Muscle relaxants
Atracurium/cisatracurium are the muscle relaxant of choice as there
elimination is not dependent on kidney.
Mivacurium is an alternative as its elimination is also independent of
kidney.
Gallamine and metocurine are entirely dependent on renal excretion
for elimination) Contraindicated in renal disease .
Pancuronium. pipecuranium, Alcuronium and doxacurium are
Primarily dependent on r contraindicated, however
neuromuscular function should be closely monitored r.f these
agents are used in Patients with abnormal renal function.
Vecuronium and Rocuronium are primarily excreted in Bile (hepatic
elimination) but some amount is eliminated in urine also.
So, only three non-depolarizing blockers have no elimination through


kidney:- Atracurium, Cisatracurium, Mivacurium
Succinylcholine (delnlarizing blockerl is also independent of renal
excretion for elimination.
It can be safety used in the Presence of renal failure. provided
serum potassium concentration is less than 5 mg/L.


224. True regarding local anesthatics is/are?
a) Prilocaine is longer acting then bupivacaine
b) Tetracaine is more potent than lignocaine
c) Dibucaine is the longest acting local anesthetic
d) Bupivacaine can produce cardiotoxicity
e) Cocaine can produce hypotension
Correct Answer - B:C:D
Ans. is'b'i.e., Tetracaine is more potent than lignocaine,'c' i.e.,
Dibucaine is the longest acting local anesthetic & 'd' i.e.,
Bupivacaine can produce cardiotoxicity
[Ref: Morgan 4th/e p. 266-270, 926; Ajay Yadav 4'h/e p. 118;
Essential of anaesthesia 4th/e p. 116; Goodman & Gilman LLth/e
p. 375]
Chloroprocaine is the shortest actingLA.
Dibucaine is the longest acting. most potent and most toxic LA.
Procaine & chloroprocaine are least potent LAs.
BuQivacaind is the most cardiotoxic LA (Ropivacaine is a newer
bupivacaine congener with less cardiotoxicity).
Levobupivacaine (The S (-) enantiomer of bupivacaine) is less
cardiotoxic and less prone to cause seizure.
Prilocaine and Benzocaine can cause Methemoglobinemia.
Lignocaine is the most commonly used LA. Bupivacaine
has the highest local tissue irritancy
Chloroprocaine is contraindicated in spinal anaesthesia as it can
cause paraplegia due to the presence of neurotoxic
preservative sodium metabisulphite.
Procaine is the LA of choice in malignant hyperthermia.




225. If you are asked to collect 4 serial
samples from lumbar puncture. What
should be done with first sample?

a) Cell counts like differential counts
b) Biochemical tests [protein & glucose etc
c) Bacterial culture and gram staining
d) Mycobacterial & fungal culture and staining
e) None of the above-discard the sample
Correct Answer - A
Ans. is'a'i.e., Cell counts like differential counts
Ref: https://emedicine.medscape.com/article/80773-technique
The classic approach is to send the 4 CSF tubes for
the following studies:

. Tube I - Cell count and differential .
Tube 2 - Glucose and protein levels
. Tube 3 - Gram stain, culture and sensitivity (C&S)
. Tube 4 - Cell count and differential


226. TRUE statement regarding inhalational
anesthesia is/ are?
a) Sevoflurane is the agent of choice for children and
asthma patients
b) Sevoflurane should not be used where the gas flow rate is
less than 2 L/min
c) Desflurane should not be used for induction in children
d) Isoflurane is more potent than sevoflurane
e) Halothane is the agent of choice for day care surgery
Correct Answer - A:B:C:D
Ans. is'a'i.e., Sevoflurane is the agent of choice for children
and asthma patients,'b'i.e., Sevoflurane should not be used
where the gas flow rate is less than 2Llmin,'c' i.e., Desflurane
should not be used for induction in children & 'd' i.e.,
Isoflurane is more potent than sevoflurane

[Ref: Ajay Yadav Sth/e p. 70-87; Morgan Sth/e p. 163-70]
"In June 1995' the Food and Drug Administration (FDAI approved
the clinical use of sevoflurane. but with a warning that not sued at
fresh gas flows less than 2 l/min because sufficient data had not
been presented to establish its safety in that circumstance.
The FDA was concerned that sevoflurane may cause adverse renal
effects at low flows because it is degraded by the strong bases in
CO2 absorbents to fluoromethyl-2,2-difluoro-1-(trifluoromethyl) vinyl
ether (compound A).
-http://anesthesiologlt,pubs.asahq.org/article.aspx?
articleid=2026924




227. In ophthalmology a patient is allergic to
aminoesters. What can be used?
a) Cocaine
b) Procaine
c) Prilocaine
d) Bupivacaine
e) Tetracaine
Correct Answer - C:D
Ans. is'c'i.e., Prilocaine &'d'i.e.,
Bupivacaine
[Ref: Lee's 13th/e p. 486]
Prilocaine & bupivacaine are amides (amcinonide). Other three are
aminoesters.


228. Natural radio-isotopes are?
a) Radium 226
b) Cesium 137
c) Strontium 90
d) Iridium 192
e) Cobalt 60
Correct Answer - A:B:C
Ans. is 'a' i.e., Radium 226,'b' i.e., Cesium 137 &'c'i.e.,
Strontium 90

[Rel https://www.britannica.com/science/radioactive-
isotope; https://www.chemicool.com/examples/natural-
radioactive-elements.htmlf]
Natural sources of radiation (Background radiation)
Hydrogen (H-3), beryllium (Be- l0), carbon (C- 14) , radium (Ra-226).
Radon (Rn-222),cesium (Cs-37), sodium(Na-22), silicon (Si-32),
chlorine (Cl-36), argon (Ar-39), krypton (Kr-81, Kr-78), iodine (I-129,
I-131), potassium (K-40), strontium (Sr 90). calcium (Ca-a8),
germanium (Ge-76), zirconium- (2r-96), samarium, (Sm-147,
I48)selenium (Se-82), rubidium (Ru-87), molybdenum (Mo-100),
cadmium (Cd-113, Cd-l16), xenon (Xe-136), barium (Ba-130),
gadolinium (Gd-152), tungsten (Tn-180), platinum (Pt-190),
bismuth (Bi-209), thorium (Th-232, Th-230) and uranium (u -
23s,236, 237, 238)


229. Posterior urethral valves in
children/infants are diagnosed by?
a) CT
b) HRCT
c) Prenatal ultrasound
d) Postnatal ultrasound
e) Micturating cystourethrography
Correct Answer - C:D:E
Ans. is 'c' i.e., Prenatal ultrasound, 'd' i.e., Postnatal
ultrasound &'e' i.e., Micturating cystourethrography

[Ref Clinical pediatric nephrology 3dlet p 94; Nekon 18th/e p. 2241]
Radiographic investigations for Posterior urethral valves
(PUVs)
Ultrasound
A. Antenatal ultrasound
B. Postnatal ultrasound
C. Voiding cystourethrogram (VCUG\ best imaging technique)


230. Radiological tests which are used to see
white matter of brain are?
a) Skull X-ray
b) CT
c) PET
d) MRI
e) Magnetic Resonance Spectroscopy
Correct Answer - A:B:C:D:E
Ans. is'All'i.e., (a, b, c, d & e)
[Ref www.ncbi.nlm.nih.gov]
'Skull x-rays were historical useful and capable of identifying the
gyriform calcification of the subcortical white matter although they
no longer play a significant role in the diagnosis or management of
this condition.
The finding usually becomes evident between 2 and 7 years of
age" White matter disease imaging
Investigation of choice for white matter disease > MRI (CT is second
choice).


231. Claw sign on radiography is seen in?
a) Ileocoecal TB
b) Ischemic colitis
c) Crohn's disease
d) Ulcerative colitis
e) Intussusceptions
Correct Answer - E
Ans. is'e' i.e., Intussusceptions
[Ref www.ncbi.nlm.nih.gov/]
Radiological signs of intussusceptions
Claw sign
Meniscus sign
Empty right iliac fossa
Coiled spring sign
Pincer sign


232. Radiological signs of NF-1 include?
a) Scoliosis
b) Widening of intercostal space
c) Posterior vertebral scalloping
d) Sphenoid wing dysplasia
e) Narrowing of neural foramina
Correct Answer - A:C:D
Ans. is'a' i.e., Scoliosis, 'c' i.e., Posterior vertebral
scalloping &'d' i.e., Sphenoid wing dysplasia

[Ref: https://radio-p a e dia. o r g/ articles / n eur ofb r o m at o sis
- typ e - 1 ]
Radiographic features of NF 1
Progressive sphenoid. wing dysplasia
Kyphoscoliosis
Posterior vestibular scalloping


233. Teletherapy uses?
a) Electron
b) X-rays
c) Gamma rays
d) Beta rays
e) Protons/neutron
Correct Answer - A:B:C:E
Ans. is 'a' i.e., Electron, 'b' i.e., X-rays, 'c' i.e., Gamma rays &
(e' i.e., Protons/neutrons

[Ref: Radiotherapy & brachyther-apy by Alessandra caner p.
19; Radiation physics p. 210-217; Sumer Sethi &/e p. 177, 184]
External beam radiotherapy (EBRT) or Teletherapy: -
In teletherapy the source of radiation is distant from the patient.
Teletherapy may be given by the following :
. X-rays beams (Linear acceleration)
. Gamma rays: - Cobalt - 60 beam or Cesium-I37.
. Particulate beams


234. Dose rate in linear accelerator is
measured as ?
a) Rads/minute
b) Rads/second
c) Roentgen/second
d) Curie/minute
e) None
Correct Answer - A
Ans. is'a'i.e., Rads/minute
[Ref, Innovation in radiation oncology p. 100)
The dose rate in teletherapy (including linear accelerator) is
measured in monitor unit or rad per minute,
A monitor unit is a measure of machine output from a clinical
accelerator for radiation therapy such as linear accelerator or
an orthovoltage unit.


235. True about sucide attempts are all except

?
a) Hopelessness is one of the important predictor
b) Same as parasuicide
c) Increased risk with substance abuse
d) Commonly seen in young male
e) None
Correct Answer - B:D
Ans. is 'b' i.e., same as parasuicide &'d' i.e., commonly seen
in young male

Feeling of Hopelessness and loneliness are present in most of the
suicide attempts.
Parasuicide is non-suicidal self injury.
Substance abuse is a risk factor for suicide.
Suicidal attempts are more common in females (suicides are more
common in males).


236. A patient with paranoid schizophrenia
c
talks about omnimicro', but he is unable
to explain it in detail and reach a
conclusion. He often repeats last syllable
of one word of last sentence. This can be
related to -

a) Circumstantiality
b) Neologism
c) Preservation
d) Knight's movement
e) Logoclonia
Correct Answer - C:E
Ans. is 'c' i.e., Preservation & 'e' i.e., Logoclonia
Ref: Textbook of Marketing Psychiatry p. 319)
Perseveration:
Continuing with a verbal response which was initially appropriate,
However, thereafter there is persistent and inappropriate repetition
of the same verbal response. There is out of context repetition of
words, phrases, ideas or points even after it has been dealt with
exhaustively or the listener has tried to change the subject.
Perseveration affects speech and it got two forms:
Logoclonia:
Last syllable of the last word is repeated, eg. I am well today ay ay
ay ay

Palilalia:
Repeated word is perseverate with increasing frequency.
There is repetition of words and phrases rather than syllable.


237. Mania does not include?
a) Delusion of grandeur
b) Delusion of guilt
c) Delusion of persecution
d) Delusion of reference
e) Delusion of infidelity
Correct Answer - B:E
Ans. is 'b' i.e., Delusion of guilt & 'e' i.e., Delusion of infidelity
Ref: Niraj Ahuja p. 69-71; Kaplan & Saddock p. 358, 64-71
Symptoms of mania:
Mania is antipodal to depression and its symptoms are a mirror
image of those of depression.
The classical triad of symptoms includes elated mood, pressure of
speech, and increased psychomotor activity.
Important sign and sYmptoms of mania are : -
Elevated mood: - Euphoria (mild elevation), Elation, (moderate
elevation), Exaltation (severe elevation), Ecstasy (Very severe
elevation). Moodmay become irritable, if person is stopped
from doing what he wants.
Thought & speech:-Pressure of speech (rapid talk,over-talkative),
flight of ideas, delusion of grandeur.delusion of persecution
secondary to delusion of grandeur (e.g., person thinks that
people are against me because I am so great), delusion of
reference, delusion of love (erotomania), Distractibility. There is
high self esteem.
Increased psychomotor activity: - Over activeness, restlessness,
increased energy, there is no time for rest.




238. Which of the following is NOT associated
with antisocial personality?
a) No feeling of guilt
b) Unstable and intense interpersonal relationship
c) No care about feeling of other
d) Recurrent suicidal threats
e) Disrupted self image
Correct Answer - B:D
Ans. is 'b' i.e., Unstable and intense interpersonal
relationship &'d' i.e., Recurrent suicidal threats

Ref Niraj Ahuja 6th/e p. 122; Essentials of clinical psychiatry 4th/e
p. 878; Namboodiri/e p. 305)
These two are associated with borderline personality disorder.
Antisocial (Dissocial) personality disorder:
The essential features of antisocial personality disorder are a
disregard for and violation of the rights of the other and the rules
of the society.
It is characterized by repeated violation of the law and rules of the
society:
Patient with antisocial personality disorder may have criminal
behaviour, homicide, sexual offences and drug abuse.
Physical aggressiveness
Reckless disregard for safety of self or others; Consistent
irresponsibility in work and family environment and lack of remorse.
This disorder is synonymous with previously used terms /like
psychopath or sociopath.

Antisocial personality disorder impairs a person's ability to care
about the feelings and needs of others. They may not feel
empathy or guild.


239. Which of the following is not a
paraphilia?
a) Adultery
b) Masochism
c) Exhibitionism
d) Frotteurism
e) Lesbianism
Correct Answer - A:E
Ans. is 'a' i.e., Adultery &'e' i.e., Lesbianism
Ref: Parikh fi/e p. 411-12; Reddy 32th/e p. 411; Niraj Ahuja Vh/e
p. 124; wikipedia.orgl
Paraphilias - Old definition
A paraphilia is a condition in which a person's sexual arousal and
gratification depend on fantasizing about and engaging in sexual
behavior that is atypical and extreme.
Thus all sexual pertersions and unnatural sexual offences (e.g.
homosexuality, bestiality etc.) are paraphilias.
The DSM-5 acknowledges that many dozens of paraphilias exist, but
only has specific listings for eight that are forensically important and
relatively common.
These are voyeuristic disorder, exhibitionistic disorder, frotteuristic
disorder, sexual masochism disorder, sexual.sadism disorder,
pedophilic disorder, fetishistic disorder, and transvestic disorder.
Homosexual behavior gay & lesbianism is longer considered as
"Paraphilias".


240. Most common psychiatric disorder after
trauma/ stress?
a) Major depression
b) Mania
c) Schizophrenia
d) PTSD
e) Acute stress disorder
Correct Answer - D:E
Ans. is'd' i.e., PTSD &'e'i.e., Acute stress reaction
[Ref: Niraj Ahuja p. 111-12; Kaplan & Saddock p.437-40]
Stress disorders:
Stress disorder is the condition marked by the development of
symptoms after exposure to traumatic life evett.
The person reacts to this experience with fear and helplessness,
persistently relives the event, and tries to avoid being reminded of
it.
Stress disorder may be : -
Post - traumatic stress disorder (PTSD): - Symptoms lasts for more
than a month,
Acute stress disorder: - Symptoms last for less than 4 weeks


241. Which vitamin deficiency causes
dementia:
a) Vitamin A
b) Vitamin C
c) Vitamin B12
d) Vitamin B1
e) Nicotinic acid
Correct Answer - C:D:E
Ans. is'c' i.e., Vitamin B 12,'d'i.e., Vitamin B 1 &'e' i.e., Nicotinic
acid

Ref: Dementia by Brown and Hillan I't/e p. 57
https://;www.ncbi.nlm.nih.gov/pmc/articles/PMC 3428233 /
The B-vitamins, including vitamins B12, B6, B1, B2, niacin (B3) and
folate (B9), have been implicated as Protective risk factors against
cognitive decline and Alzheimer's disease.
Of all the B-vitamins, vitamin B12, niacin, and thiamine have the
most clearly established relations with deterioration in mental
state"


242. Defence mechanism in obsessive
compulsive disorder is/are:
a) Undoing
b) Conversion
c) Reaction formation
d) Isolation of defect
e) Projection
Correct Answer - A:C:D
Ans. is'a'i.e., Undoing,'c'i.e., Reaction formation
&'d'i.e., Isolation of affect

Ref: Niraj Ahuja p. 97, 208-210; Kaplan & Sadock p.161-
62 Important diseases and their defence mechanisms
Obsessive compulsive disease: Reaction Formation.
di$tlacement, undoing. isolation of affect. repression inhibition.
Phobia: - Displacement, inhibition.
Conversion disorder (Hysteria): - Conversion.
Persecutory delusions and hallucinations: - Projection.
Neuroses (neurotic reaction) : - Regression to an earlier
state,Failure of repression


243. Beck's cognitive triad of depression
includes?
a) Self
b) Future
c) Past experience
d) World & environment
e) Others
Correct Answer - A:B:D
Ans. is'a' i.e., Self,'b' i.e., Future &'d' i.e., World
and environment

[Ref: Kaplan & Saddock p.355; various sites internet]
Beck 's cognitive theory of depression (1976)
Aaron Beck studied people suffering from depression and found that
they appraised events in a negative way.
Beck identified three mechanisms that he thought were responsible
for depression:
The cognitive triad. (of negative automatic thinking)
Negative self schemas
Errors in Logic (i.e. faulty information processing)
invalid question id

This post was last modified on 11 August 2021